Примириться перед неумолимой судьбой найти ошибку

Допишите окончания, согласуя сказуемое с подлежащим. № 1. 1. Большинство картин художников - передвижников

Допишите окончания, согласуя сказуемое с подлежащим. № 1. 1. Большинство картин художников — передвижников был. . воспринят. . как гимн человеку, прославляющий и утверждающий его высокие духовные ценности. № 2. Часть студентов уже сдал. . экзамены по специальности. № 3. Двадцать человек стоял. . в стороне. № 4. Множество темноватых туч расползал. . сь по небу. Ответы. 1. Большинство картин художников — передвижников было воспринято как гимн человеку, прославляющий и утверждающий его высокие духовные ценности. 2. Часть студентов уже сдала экзамены по специальности. Двадцать человек стояли в стороне. 3. Множество темноватых туч расползалось по небу. Десять студентов окончили институт с отличием.

Б) Исправьте ошибки в употреблении словосочетаний № 1. Отчитаться в получении полученного аванса. №

Б) Исправьте ошибки в употреблении словосочетаний № 1. Отчитаться в получении полученного аванса. № 2. Отзыв на статью. № 3. Уделять внимание на каждую мелочь. № 4. Превосходство новой техники перед старой. № 5. Примириться пред неумолимой судьбой. • Ответы. 1. Отчитаться (о чём? ) о получении полученного аванса. 2. Отзыв (о чём) о статье. 3. Уделять внимание (чему? ) каждой мелочи. 4. Превосходство новой техники (над чем? ) над старой. 5. Примириться (с чем? ) с неумолимой судьбой.

В) Исправьте ошибки в употреблении падежных форм и предлогов. № 1. Об этом мы

В) Исправьте ошибки в употреблении падежных форм и предлогов. № 1. Об этом мы познакомим вас позже. № 2. Докладчик подчеркнул о необходимости строительства. № 3. Решительность парламента свелась лишь в предоставлении полномочий. № 4. Вчера брат приехал с Москвы. № 5. По окончанию переговоров представители сторон подписали совместное заявление. • Ответы. • С этим мы познакомим вас позже. Докладчик подчеркнул необходимость строительства. Решительность парламента свелась лишь к предоставлению полномочий. Вчера брат приехал из Москвы. По окончании переговоров представители сторон подписали совместное заявление.

1. Укажите предложение без пунктуационной ошибки. 1) Наша школа как корабль. 2) Изделия Фаберже

1. Укажите предложение без пунктуационной ошибки. 1) Наша школа как корабль. 2) Изделия Фаберже подлинные произведения искусства. 3) Интеллигентность это способность понимать. 4) Красота — есть обещание счастья. 2. Укажите предложение с пунктуационной ошибкой. 1) Площадь комнаты — двадцать квадратных метров. 2) Жить — Родине служить. 3) Пятью пять — двадцать пять. 4) Любовь — есть источник поэзии, добра и ненависти.

1. Укажите предложение без пунктуационной ошибки. 1) Наша школа как корабль. 2) Изделия Фаберже

1. Укажите предложение без пунктуационной ошибки. 1) Наша школа как корабль. 2) Изделия Фаберже подлинные произведения искусства. 3) Интеллигентность это способность понимать. 4) Красота — есть обещание счастья. 2. Укажите предложение с пунктуационной ошибкой. 1) Площадь комнаты — двадцать квадратных метров. 2) Жить — Родине служить. 3) Пятью пять — двадцать пять. 4) Любовь — есть источник поэзии, добра и ненависти.

• Синтаксис – это • Словосочетание – это • В словосочетании выделяются ….

• Синтаксис – это • Словосочетание – это • В словосочетании выделяются …. слово и …. зависимое слово • Смысловая связь слов в словосочетании устанавливается при помощи… • Грамматическая связь осуществляется при помощи….

В 3 Из предложения 19 выпишите подчинительное словосочетание со связью СОГЛАСОВАНИЕ. (19)У них желания

В 3 Из предложения 19 выпишите подчинительное словосочетание со связью СОГЛАСОВАНИЕ. (19)У них желания сбываются чаще, и, даже если этого не происходит, он не зацикливаются на неудачах, а находят в ситуации светлые стороны

Словосочетаниями не являются: 1 2 3

Словосочетаниями не являются: 1 2 3

Согласование – это такой вид подчинительной связи, при которой зависимое слово уподобляется главному в

Согласование – это такой вид подчинительной связи, при которой зависимое слово уподобляется главному в общих у них грамматических формах (рода, числа, падежа), например: хороший специалист, заветная мечта, любимое дело. При согласовании с изменением форм главного слова соответственно изменяются и формы зависимого слова: важное дело (Им. п. ), важным делом (Р. п. ), о важном деле (П. п. ), важные дела (мн. ч. ). При определении вида словосочетания важно обращать внимание на зависимые слова: какой частью речи они выражены. При согласовании зависимым словом могут быть ПРИЛАГАТЕЛЬНОЕ, ПРИЧАСТИЕ, МЕСТОИМЕНИЕ, ЧИСЛИТЕЛЬНОЕ: частые встречи, летящий мяч, наше право, второй ряд.

Управление - это такой вид подчинительной связи, при которой главное слово требует употребления зависимого

Управление — это такой вид подчинительной связи, при которой главное слово требует употребления зависимого слова в определённом падеже с предлогом или без предлога, например: написание реферата (Р. п. ), работа над проблемой (Т. п. ), интерес к предмету (Д. п. ). Зависимым словом при управлении может быть СУЩЕСТВИТЕЛЬНОЕ или МЕСТОИМЕНИЕ: начало игры, попросить её.

• Примыкание - это такой вид подчинительной связи, при которой зависимое слово, не

• Примыкание — это такой вид подчинительной связи, при которой зависимое слово, не имея форм словоизменения, присоединяется к главному слову по смыслу, например: стараться понять, чтение вслух, очень хорошо. Зависимым словом примыкании могут быть ДЕЕПРИЧАСТИЕ, НЕОПРЕДЕЛЁННАЯ ФОРМА ГЛАГОЛА, НАРЕЧИЕ, СРАВНИТЕЛЬНАЯ СТЕПЕНЬ ПРИЛАГАТЕЛЬНОГО: спать стоя, просил остаться, бежать быстро, стать лучше.

Обратите внимание!!! Особое внимание следует обратить на словосочетания с его, её, их притяжательными местоимениями.

Обратите внимание!!! Особое внимание следует обратить на словосочетания с его, её, их притяжательными местоимениями. Они, в отличие от омонимичных им форм личных местоимений, не изменяются и, соответственно, вступают только в связь примыкание. Сравните: «Назавтра его должны были освободить» — должны были освободить (кого? ) его – его здесь форма родительного падежа личного местоимения он, следовательно, перед нами связь управление. «Его голова была устроена иначе, чем моя» — голова (чья? ) его – это притяжательное местоимение, которое не изменяется: его голова, его головой, его голове, следовательно, примыкает к главному слову.

Обратите внимание!!! • Особый случай примыкания – когда зависимым словом является инфинитив: «Прошу записать

Обратите внимание!!! • Особый случай примыкания – когда зависимым словом является инфинитив: «Прошу записать меня на курсы водителей» . В данном случае сочетание прошу записать не является составным глагольным сказуемым, так как действие осуществляется разными субъектами (действующими лицами): прошу я, а записывать будете вы / они и т. п. , то есть другое лицо / лица, и инфинитив в данном случае является не частью составного сказуемого, а дополнением (прошу (о чем? ) записать). В словосочетании прошу записать инфинитив примыкает к главному слову.

Для выполнения задания В 3 используйте следующий АЛГОРИТМ: 1. При определении нужного словосочетания посмотрите,

Для выполнения задания В 3 используйте следующий АЛГОРИТМ: 1. При определении нужного словосочетания посмотрите, как связаны между собой слова: если оба слова изменяются – согласование; если одно слово требует от другого употребления в каком-либо падеже – управление; если слова связаны только по смыслу и между ними нет грамматической связи, которая осуществляется с помощью окончания и предлога или только окончания – примыкание.

2 Проанализируйте зависимые слова: q если зависимым словом является прилагательное, причастие, местоимение, числительное –

2 Проанализируйте зависимые слова: q если зависимым словом является прилагательное, причастие, местоимение, числительное – это согласование; q если зависимым словом является существительное или местоимение, употреблённое в том или ином падеже – это управление; q если зависимым словом является деепричастие, неопределённая форма глагола, наречие, сравнительная степень прилагательного – это примыкание.

Синтаксический рецепт Если сложить начальные буквы названий всех трех способов подчинительной связи, получится слово

Синтаксический рецепт Если сложить начальные буквы названий всех трех способов подчинительной связи, получится слово СУП. Как известно, суп очень полезен. Но чтобы он получился вкусным, нужно знать, какие ингредиенты необходимы. Итак, варим синтаксический СУП. Первый ингредиент – согласование. Для того, чтобы его приготовить, возьмите самостоятельную часть речи и прибавьте по вкусу прилагательное, причастие, порядковое числительное или местоимение, сходное с ним по форме. Второй ингредиент – управление. Для того, чтобы его приготовить, к самостоятельной части речи нужно добавить существительное или местоимение. Третий ингредиент – примыкание. Чтобы приготовить хорошее примыкание, возьмите самостоятельную часть речи и смешайте ее с неизменяемым словом. После этого аккуратно, не перемешивая ингредиенты, выкладываем их в кастрюлю и варим до полной готовности. Через некоторое время вы получите вкусный, полезный синтаксический СУП под названием «Способы подчинительной связи» . Приятного аппетита!

Выполните задания 2 -10 по тексту: (1)В течение своей жизни Гоголь много путешествовал. (2)Однако

Выполните задания 2 -10 по тексту: (1)В течение своей жизни Гоголь много путешествовал. (2)Однако где бы он ни странствовал, в дороге он чувствовал себя несравненно лучше, чем сидя на одном месте. (3)Он приспособился в дороге думать и работать, между тем одновременно и набираясь впечатлений, и отдыхая душой. (4)У него даже был план объехать все монастыри России, имея в виду, что они располагались, как правило, в красивейших местах. (5)Он собирал в душе своей как бы коллекцию истинно русских пейзажей. (по В. Солоухину)

2. Из предложения 1 выпишите словосочетание со связью СОГЛАСОВАНИЕ. 3. Какой способ связи в

2. Из предложения 1 выпишите словосочетание со связью СОГЛАСОВАНИЕ. 3. Какой способ связи в словосочетании в течение жизни путешествовал (предложение 1)? 4. Из предложения 1 выпишите словосочетание со связью ПРИМЫКАНИЕ. 5. Какое сочетание слов из предложения 2 не является примером примыкания: 1) где бы ни странствовал 2) чувствовал себя лучше 3) сидя на месте 4) несравненно лучше

6. Определите способ связи в словосочетании на одном месте (предложение 2). 7. Определите способ

6. Определите способ связи в словосочетании на одном месте (предложение 2). 7. Определите способ связи в словосочетании набираясь впечатлений (предложение 3). 8. Какое сочетание слов из предложения 4 является словосочетанием: 1)был план 2)план объехать 3)даже был 4)располагались как правило 9. Определите способ связи в словосочетании у него был (предложение 4). 10. Какое сочетание слов из предложения 5 не является словосочетанием: 1)в душе своей 2) как бы коллекцию 3)собирал в душе 4) коллекцию пейзажей •

Проверьте себя. 1. будущее (какое? ) прекрасное - согласование прибавить (к чему? ) к

Проверьте себя. 1. будущее (какое? ) прекрасное — согласование прибавить (к чему? ) к четырём — управление храбрость (чья? кого? ) солдата — управление в пору (какую? ) счастливую — согласование глядеть (куда? вслед чему? ) вслед поезду — управление выть (как? ) по-волчьи — примыкание рублями (сколькими? ) пятьюстами — согласование трудов (чьих? ) моих — согласование работа (чья? ) её — примыкание пришёл, (несмотря на что? ) несмотря на болезнь — управление находиться (где? посредине чего? ) посередине комнаты – управление

2. 3. 4. 5. 6. 7. 8. 9. 10. своей жизни управление много путешествовал

2. 3. 4. 5. 6. 7. 8. 9. 10. своей жизни управление много путешествовал сидя на месте согласование управление 2 Управление 2

Помните: Формальный подход к синтаксическому анализу, который не учитывает логичности установления смысловых отношений между

Помните: Формальный подход к синтаксическому анализу, который не учитывает логичности установления смысловых отношений между словами в предложении, влечёт ошибки как в выделении словосочетаний, так и в определении членов предложения, а также частей речи. Так, в предложении Представьте, вокруг всё сияет: и небо, и солнце, и трава, и река слово вокруг в словосочетании примыкает к главному слову: сияет (где? ) вокруг, обозначает признак действия и является наречием места. В предложении И грибы уже не собираются, а вокруг когда-то прозрачнейшего озера выросли чудовищные лопухи и папоротники смысловые связи того же слова другие: выросли (где? ) вокруг озера. Слово вокруг помогает связать в подчинительном словосочетании главное слово и управляемое им зависимое, выполняя служебную функцию предлога. Итак, в 1 м примере способ связи слов – примыкание, а слово вокруг – наречие; во 2 -м примере способ связи слов – управление, а слово вокруг – производный предлог.

Русский язык представляет собой сложную
систему фонетических, лексических и
грамматических средств. Требование
правильности речи распространяется на
все уровни языка, и всюду требуется
соблюдение нормы. Языковая норма – это
центральное понятие речевой культуры.

Норма литературного языка
это общепринятое употребление языковых
средств, совокупность правил, определяющих
образцовое использование этих средств.
В соответствии с определением, нормативно
– не то, что широко распространено в
речи, а то, что обязательно, что
соответствует требованиям и рекомендациям
словарей и справочной лингвистической
литературы.

Языковые нормы не выдумываются учеными.
Они отражают закономерные процессы и
явления, происходящие в языке, и
поддерживаются речевой практикой. К
основным источникам языковой нормы
относятся произведения писателей-классиков
и современных писателей, анализ языка
средств массовой информации.

Характерные особенности нормы
литературного языка: общеобязательность,
относительная устойчивость и в то же
время историческая изменчивость.

В соответствии с основными уровнями
языка и сферами использования языковых
средств выделяются следующие типы норм:

  1. Орфоэпические
    (произношение), связанные со звуковой
    стороной литературной речи.

  2. Грамматические,
    связанные с правилами словоупотребления,
    употребления словосочетаний и
    синтаксических конструкций.

  3. Лексические,
    связанные с правилами словоупотребления,
    отбора и использования наиболее
    целесообразных лексических единиц.

Нормы помогают литературному языку
сохранять свою целостность и общепонятность.
Они защищают литературный язык от потока
диалектной речи, социальных и
профессиональных арго, просторечия.

Природа языковой нормы очень сложна.
Если бы нормы не были устойчивыми, если
бы они легко подвергались различного
рода воздействиям, языковая связь между
поколениями была бы нарушена. И потому
произведения А.С.Пушкина мы понимаем
свободно, хотя они созданы почти два
века тому назад. В то же время устойчивость
норм не абсолютна, а относительна. Норма,
как и все в языке, медленно, но непрерывно
изменяется под влиянием живой разговорной
речи, местных говоров, просторечья,
социальных и профессиональных жаргонов,
заимствований и т.д.

То, что было нормой в прошлом столетии
и даже 15-20 лет назад, сегодня может стать
отклонением от нее. Например, в начале
19 века нормативными были окончания
-И/-Ы во множественном числе имен
существительных: век – веки, дом –
домы, вино – вины
; окончание –ей в
родительном падеже множественного
числа: афиша – афишей, басня –
басней, капля – каплей
(современные
формы веки, дома, вина; афиш, басен,
капель).

Замена одной нормы другой происходит
очень медленно. Изменению норм предшествует
появление их вариантов, которые реально
существуют в языке.

Вариантность нормы проявляется очень
широко во всех областях языка. Например,
акцентологические варианты:
родился – родился, мышление – мышление;
фонетические варианты: будничный
– буднишний; варианты грамматических
форм: тракторы – трактора, чая – чаю,
искусствен – искусственен; морфологические
и словообразовательные варианты:
спазм – спазма, нагрев – нагревание,
лаконичный – лаконический; синтаксические
варианты: ждать поезда – ждать поезд,
исполненный отвагой – исполненный
отваги и т.п.

Орфоэпическая норма.
Произносительные нормы устной речи
изучает специальный раздел языкознания
– орфоэпия. Соблюдение единообразия в
произношении имеет важное значение.
Орфоэпические ошибки всегда мешают
воспринимать содержание речи: внимание
слушающего отвлекается различными
неправильностями произношения, и
высказывания во всей полноте и с
достаточным вниманием не воспринимается.
Произношение, соответствующее
орфоэпическим нормам, облегчает и
ускоряет процесс общения. Поэтому
социальная роль правильного произношения
очень важна. Особенно в настоящее время
в нашем обществе, где устная речь стала
средством самого широкого общения на
различных собраниях, конференциях,
съездах, симпозиумах.

Варианты норм отражаются в словарях
современного литературного языка.
Например, в «Орфоэпическом словаре»
как равноправные фиксируются акцентные
варианты таких слов: ржаветь и ржаветь,
тефтели и тефтели, штабы и штабы. Варианты
таких слов как нормировать и мышление
даются с пометой «доп.» (допустимо)
нормировать и мышление.

Помета «допустимо устаревающее»
(доп.устар.) показывает, что оцениваемый
вариант постепенно утрачивается:
сливочный – доп.устар. сливошный.

От колебаний нормы следует отличать ее
нарушения. В словарях ненормативные
варианты снабжены пометами «не
рекомендуется» (не рек.), «неправильно»
(неправ.), «грубо неправильно»: ассиметрия,
не рек. ассиметрия; корысть, неправ.
корысть; афера неправ. афёра; документ,
грубо неправ. документ.

Показатели различных нормативных
словарей дают основание говорить о 3
степенях нормативности:

— норма
1 степени – строгая, жесткая, не допускающая
вариантов;

— норма
2 степени – нейтральная, допускает
равнозначные варианты;

— норма
3 степени – более подвижная, допускает
использование разговорных, а также
устаревших форм.

Основные законы произношения
согласных:

— оглушение
и уподобление. В русской речи происходит
обязательное оглушение звонких согласных
в конце слова. Например: хле[п] (хлеб),
са[т] (сад), гора[т] (город) и т.д.

— в
сочетаниях звонкого и глухого согласного
(так же, как и глухого и звонкого) первый
из них уподобляется второму. Например:
ложка – ло[ш]ка, пробка – про[п]ка.

— перед
согласными л, м, н, р, не имеющими
парных глухих, и перед [в] уподобления
не происходит. Следует обратить внимание
на сочетание -ЧН, т.к. при его произношении
нередко допускаются ошибки. В произношении
слов с этим сочетанием наблюдается
колебание, что связано с изменением
правил старого московского произношения.

По нормам современного русского
литературного языка сочетание -ЧН обычно
так и произносится [ЧН], особенно это
относится к словам книжного происхождения
(алчный, античный, единичный, маскировочный).
В соответствии с нормами старого
московского произношения на месте
сочетания –ЧН- произносилось [ШН]:
грешневый, игрушешный, сказошник,
закусошная.

В современном языке можно говорить о
трех категориях слов в зависимости от
рекомендуемого в них произношении
сочетания –ЧН-:

1. Слова,
в которых –ЧН- произносится так, как
пишется: кулачный, уличный, библиотечный,
отличник, заочник.

2. Слова,
в которых допустимо двоякое произношение
–ЧН- и –ШН-: булочная и булошная, молочный
и молошный, порядочно и порядошно,
спичечный и спичешный, сливочный и
сливошный.

3. Слова,
в которых и сейчас орфоэпическая норма
требует произношения –ШН-, а
произношение –ЧН- недопустимо: конечно,
ску[шн]о, наро[шн]о, яи[шн]ица, скворе[шн]ик.
Произношение –ШН- вместо орфографического
–ЧН- в настоящее время требуется и в
женских отчествах на –ИЧНА-: Ильини[шн]а,
Никити[шн]а, Савви[шн]а, Фомини[шн]а,
Лукини[шн]а.

Заимствованные слова, как правило,
подчиняются орфоэпическим нормам
русского литературного языка и только
в некоторых случаях отличаются
особенностями в произношении. Наиболее
существенное из них – сохранение в
произношении звука [О] в безударных
слогах и твердых согласных перед гласным
[Е].

В безударном положении звук [О] сохраняется,
например, в таких словах, как м[о]дель,
м[о]дерн, [о]азис, б[о]а, [о]тель, ф[о]нема
и в иностранных собственных именах:
Фл[о]бер, В[о]льтер, Ш[о]пен, М[о]пассан.
В ряде слов иноязычного происхождения
твердость согласных перед [Е] сохраняется:
ш[тэ]псель, о[тэ]ль, с[тэ]нд, ко[дэ]кс,
[дэ]мпинг, ка[рэ], к[рэ]до и др.

Особенности русского ударения.
Снижает культуру устной речи не только
неправильное произношение, но и
неправильное ударение в словах.

Особенности и функции ударения изучает
раздел языкознания, который называется
акцентологией.

Ударение в русском языке свободно, что
отличает его от некоторых других языков,
в которых ударение закреплено за каким-то
определенным слогом. Например, в
эстонском, латышском, чешском, финском
языках ударным всегда бывает первый
слог. В польском, грузинском –
предпоследний; в армянском, французском
– последний. В русском языке ударение
может падать на любой слог, поэтому его
называют разноместным. Сопоставим
ударение в словах дерево, добыча,
голова, медикамент.
Ударение в русском
языке бывает подвижным и неподвижным.
Если в различных формах слова ударение
падает на одну и ту же часть, то такое
ударение является неподвижным
(берегу, бережешь, бережет, бережем,
бережете). Ударение, меняющее свое место
в разных формах одного и того же слова,
называется подвижным (прав,
правы, права; могу, можешь, могут).

Затруднение иногда вызывает ударение
в винительном падеже единственного
числа имен существительных женского
рода. С одной стороны, ботва – ботву,
десна – десну, стопа – стопу; с другой
стороны, гора – гору, доска – доску,
стена – стену. Конкуренция между
существительными и предлогами (до, из,
за, на, об, по, под) в перетягивании
ударения имеет давнюю историю. В начале
века ударение перетягивали на себя
предлоги: на воду, за ногу, под руки, из
лесу, без вести. В настоящее время для
литературного языка в целом допустимы
оба варианта ударения: по боку, под
боком, за бок, за ворот, на землю, на зиму.
Однако для повседневной речи более
характерны сейчас безударные предлоги:
на воду, год от года, из года в год.

Затруднения вызывает постановка ударения
во многих глаголах прошедшего времени,
имеющих ударение на основе во всех
формах, кроме формы единственного числа
женского рода, в которой оно переносится
на окончание. Например: брал, брала,
брало, брали; взял, взяла, взяло, взяли;
задал, задала, задало, задали; занял,
заняла, заняло, заняли; прожил, прожила,
прожило, прожили и др. То же явление
наблюдается и у многих кратких
страдательных причастий прошедшего
времени. Например: взят, взята, взято,
взяты; начат, начата, начато, начаты;
продан, продана, продано, проданы и др.

Особенно часто нарушается норма ударения
в таких словах как оптовый, начать,
алкоголь, кухонный, жалюзи, задолго,
агент, каталог, намерение
(неправ.
оптовый, начать, алкоголь, кухонный,
жалюзи, задолго, агент, каталог, намерение).

Развитие языка влияет также на место
ударения в слове. Это приводит к
нормативным колебаниям. Для русского
ударения существует понятие вариантности,
означающее наличие в некоторых словах
вариантов ударения, применяемых в разных
ситуациях общения.

Целый ряд вариантов ударения связан с
профессиональной сферой употребления.
Есть слова, специфические ударения в
которых традиционно принято только в
узкопрофессиональной среде. В любой
другой среде такое ударение воспринимается
как ошибочное. Например: шприцы,
эпилепсия (у медиков
шприцы, эпилепсия);
компас, боцманы
(
у моряков – компас, боцмана;
флейтовый (у музыкантов
– флейтовый);
добыча (у горняков
– добыча)
и др.

Ударение в русском языке играет важную
смыслоразличительную роль. С его помощью
различаются омографы (парные
слова, совпадающие по написанию, но не
по произношению): видение – видение,
хлопок – хлопок, атлас – атлас, мука –
мука, проклятый – проклятый.

Об ударении в словах можно справиться
в «Орфоэпическом словаре русского
языка», в «Словаре ударений для работников
радио и телевидения», а также в
орфографических, толковых словарях
русского языка, в различных
словарях-справочниках, в пособиях по
культуре речи.

Грамматические нормы
это правила использования морфологических
форм разных частей речи и синтаксических
конструкций.

Наиболее часты грамматические ошибки,
связанных с употреблением рода имен
существительных. Категория рода, как
правило, весьма стабильна. Однако нередко
наблюдается неправильное отнесение
имен существительных к грамматическому
роду. Так, нарушая литературную норму,
говорят железнодорожная рельса
(вместо железнодорожный рельс),
ароматная шампунь (вместо ароматный
шампунь
), заказной бандероль (вместо
заказная бандероль), лакированный
туфель
(вместо лакированная туфля).
Правильно говорить туфля с ударением
на первом слоге: нет одной туфли; купил
красивые туфли; рада новым туфлям; много
осенних туфель.

— Грамматический род сложносокращенных
слов, составленных из первых букв – СНГ
(эсэнгэ), РТС (эртээс) – или звуков –
ИТАР – определяется по основному слову.
Поэтому говорят: СНГ возник; РТС
заключила договор; СПИД опасен; ИТАР
сообщило.
Однако иногда аббревиатура
получает род, как обычное слово, по
формальному показателю и относится к
мужскому роду в случае нулевого окончания
(ЖЭК, несмотря на то, что это
жилищно-эксплуатационная контора), к
среднему роду в случае окончания –О-
(РОНО, хотя это районный отдел народного
образования).

— Несклоняемые имена существительные,
обозначающие профессию, должность,
звание, традиционно относятся к мужскому
роду: компетентный атташе, молодой
рефери..

— Род несклоняемых существительных,
обозначающих географические названия,
определяется по общему родовому
наименованию. Например: старый Тбилиси
(город); полноводная Миссисипи (река);
Капри спал (остров).

— Несклоняемые существительные,
обозначающие неодушевленные предметы,
относятся к среднему роду. Например:
новое депо, золотое пенсне, разноцветное
кашне ароматное желе.
Исключение
составляет слово кофе (муж.род).
Газеты сообщают о падении цен на
колумбийский кофе.

— В русском языке не склоняются фамилии
на –аго, -яго, -ово, -ых, -их: Живаго,
Дубяго, Дурново, Седых, Чутских.
Не
склоняются также украинские фамилии
на –ко. Например: Стихи Тараса
Шевченко.

— Не склоняются нерусские фамилии,
оканчивающиеся на согласный звук, если
они относятся к женщинам: романы Анны
Зегерс,
но балет Арама Хачатуряна,
игра гроссмейстера Ботвинника, романы
Жюля Верна, история любви Фредерика
Шопена и Жорж Санд.

Для официального наименования рода
занятий, профессий женщин предпочтительнее
существительные мужского рода (аспирант,
геолог, лаборант
). Наименованиям на
–ша, -иха типа инженерша, врачиха,
библиотекарша
присущ сниженный,
иногда подчеркнуто-пренебрежительный
оттенок.

При выборе падежных форм типа в цехе
– в цеху, в отпуске – в отпуску, на холоде
– на холоду
следует исходить из того,
что формы на –Е- присущи литературному
языку, а формы на –У- разговорному.

Соответствуют литературной норме
следующие формы Им.п. множ.числа
существительных муж.рода:

а) с
ударным окончанием –а, -я: адреса,
вечера, глаза, голоса, директора, доктора,
профессора, мастера, номера, ордена,
острова, паспорта;

б) с
безударным окончанием –ы, -и: инженеры,
шоферы, бухгалтеры, редакторы, пекари,
лекторы, тракторы, договоры, приговоры,
выборы, порты, торты.

В ряде случаев наблюдаются вариантные
колебания: инспекторы – инспектора,
джемперы – джемпера, прожекторы –
прожектора, слесари – слесаря, тракторы
– трактора, крейсеры – крейсера.

Следует иметь в виду, что последние
более свойственны бытовой и профессиональной
речи. Некоторые вариантные формы
различаются значением: образы
(художественно-литературное) – образа
(иконы); пояса (части одежды) –
поясы (географические); пропуска
(документы) – пропуски (недосмотры);
цветы (растения) – цвета (окраска)
и т.п.

1. В родительном падеже множественного
числа в литературном языке приняты
следующие формы существительных мужского
рода (одни без окончания, другие с
окончанием):

а)
отдельные названия парных предметов:
ботинок, валенок, погон, сапог, чулок
(но носков);

б)
отдельные названия лиц по национальной
принадлежности: армян, бурят, башкир,
грузин, осетин, лезгин, турок, цыган,
румын, татар
(но: киргизов, арабов,
монголов, таджиков, узбеков, якутов
);

в)
отдельные названия единиц измерения
при указании на их количество: ампер,
аршин, ватт, вольт, герц, ом, микрон,
рентген
(но: граммов, килограммов,
джоулей
).

В случае колебаний форм с нулевым
окончанием и с –ов- первые свойственны
разговорной речи, а последние – строго
литературному языку: (много) сектор –
секторов, апельсин – апельсинов, помидор
– помидоров, томат – томатов.

2. Слова женского рода имеют следующие
окончания в родительном падеже
множественного числа: барж, басен,
вафель, кочерег, свадеб, усадеб, простынь,
яблонь; долей, килей, пригоршней, свечей.

3. Слова среднего рода: верховьев,
доньев (дно – донья), низовьев, устьев,
побережий, коленей, яблок; блюдец,
одеялец, полотенец.

Рекомендуются следующие краткие формы
имен прилагательных на –ен и –енен
(хотя обе формы являются литературными):
бездействен, безнравствен, бессмыслен,
бесчислен, бесчувствен, величествен,
двусмыслен, естествен, злокачествен,
искуствен, легкомыслен
(формы:
бездейственен, безнравственен и
т.д. более свойственны книжной речи).

Синтаксическая норма – это
правила построения словосочетаний и
предложений.

Важную роль в построении предложения
играет правильное согласование
подлежащего со сказуемым.

1. При подлежащем, выраженном собирательным
существительным, имеющим количественное
значение (ряд, большинство, меньшинство,
часть и т.д.) в сочетании с родительным
падежом множественного числа, сказуемое
обычно ставится во множественном числе,
если речь идет о предметах одушевленных
или если подчеркивается активность
действия, и в единственном числе, если
подлежащее обозначает предметы
неодушевленные. Например: Большинство
членов бригады систематически
перевыполняют нормы. Ряд новых домов
стоял в конце деревни.

Такая же форма согласования наблюдается
в тех случаях, когда подлежащее выражено
так называемым счетным оборотом, т.е.
сочетанием количественного числительного
или другого счетного слова (например,
несколько) с существительными в
родительном падеже множественного
числа. Например: Несколько человек
ходили взад и вперед по коридору. Засеяно
сто двадцать гектаров.

2. Если при подлежащем – счетном обороте
имеются слова все, эти, то сказуемое
ставится во множественном числе.
Например: Все пять мальчиков сидели
вокруг костра. Эти восемь книг куплены
недавно.
Наоборот, при наличии слов
всего, только, лишь сказуемое, как
правило, ставится в единственном числе.
Например: На экскурсию пошло всего 15
человек. В секции занимается только 8
студентов.

3. При сложных названиях, состоящих из
двух слов разного грамматического рода,
сказуемое согласуется с тем из них,
которое выражает более широкое понятие
или конкретное обозначение предмета:
витрина-стенд помещена в вестибюле;
книга-справочник переиздана; плащ-палатка
лежала в свернутом виде; кресло-кровать
стояло в углу
и т.п.

4. При существительных мужского рода,
называющих профессию, должность, звание,
но обозначающих женщину, сказуемое в
книжном стиле чаще ставится в форме
мужского рода, а в разговорном – чаще
в женском роде. Например: Ректор
университета регулярно выступал(а)
перед сотрудниками.
Согласование
определений в женском роде с такими
существительными носит разговорный
характер. Например: наша референт

5. При подлежащем, выраженном сочетанием
нарицательного и собственного имени,
сказуемое согласуется с последним:
Референт Иванова помогла директору
составить отчет. Методист Васильева
выдала справку.

6. Часто в предложениях при двух или
нескольких управляющих словах имеется
общее дополнение. Такие конструкции
вполне правильны, если управляющие
слова требуют одинакового падежа и
предлога. Например: читать и
конспектировать книгу; подбирать и
готовить кадры.
Но неверны предложения,
в которых общее зависимое слово имеется
при словах, требующих разного управления.
Например: Трест организовал и руководил
предприятиями
(организовал что?,
руководил чем?); подбор и наблюдение
над фактами
(подбор чего?, наблюдение
над чем?) и т.п. Обычный способ правки в
подобных случаях – добавление ко второму
управляющему слову местоимения,
заменяющего существительное при первом
слове. Например: подбор фактов и
наблюдение над ними.

7. Следует различать конструкции со
словами, близкими по значению или
однокоренными, но требующими различного
управления. Например: беспокоиться о
ком-нибудь,
но тревожиться за
кого-нибудь; обращать внимание на
что-либо,
но уделять внимание
чему-либо; опираться на что-либо,
но
базироваться на чем-либо; отзыв о чем-либо
(отзыв о книге), но рецензия на
что-либо
(рецензия на книгу);
превосходство над чем-либо
(превосходство
над старым), но преимущество перед
чем-либо
(преимущество перед старым).

Нарушение грамматических норм нередко
связано с употреблением в речи предлогов.
Различие предлогов благодаря + Дат.п.
и из-за + Род.п. заключается в том,
что первый используется для обозначения
причины, положительно влияющей на
что-либо (Благодаря помощи населения
последствия аварии были быстро
ликвидированы)
, а второй – для указания
причины, отрицательно влияющей на
что-либо (Из-за отсутствия помощи
населения последствия аварии не могли
быть ликвидированы быстро)
. Имеются
различия и при употреблении целевых
предлогов. В нейтральной речи употребляются
предлоги для, за: Для реализации
программы необходимы соответствующие
ресурсы. Зайди в библиотеку
за
книгами
. Последний предлог
используется только после глаголов
движения. В книжной речи употребляются
предлоги в целях, с целью. Первый
сочетается только с отглагольными
существительными (в целях реализации,
уточнения)
, а второй – с отглагольными
существительными (с целью реализации,
уточнения)
и с инфинитивом (с целью
реализовать, уточнить).

Лексическая норма. Под лексическими
нормами понимают правильность
словоупотребления. Слово должно
использоваться в том значении (в прямом
или переносном), которое оно имеет и
которое зафиксировано в словарях
русского языка. Нарушение лексических
норм приводит к искажению смысла
высказывания.

Нарушение лексических норм приводит к
искажению смысла высказывания. Нарушение
лексических норм порой связано с тем,
что говорящие путают слова, близкие по
звучанию, но различные по значению.
Например, не всегда правильно употребляются
глаголы предоставить и
представить. Иногда мы слышим
неверные выражения типа: «Слово
представляется Петрову», «Разрешите
предоставить вам доктора Петрова».

Глагол предоставить означает
«дать возможность воспользоваться
чем-либо» (предоставить слово, отпуск,
квартиру, права и т.д.), а глагол представить
имеет значение «передать, дать, предъявить
что-либо кому-либо» (представить отчет,
справку, факты, доказательства; представить
к награде, к ордену, к званию и т.д.).

Приведенные выше предложения с этими
глаголами правильно звучат так: «Слово
предоставляется Петрову», «Разрешите
представить вам доктора Петрова».

Порой неверно употребляют такие паронимы:
исполнительный – исполнительский
(исполнительный – старательный,
имеющий своей целью осуществление
чего-либо: исполнительный орган;
исполнительский – относящийся к
исполнителю: исполнительская манера);
логический – логичный (логический
– относящийся к логике: логическое
мышление;
логичный – правильный,
разумный, последовательный: логичное
рассуждение
); отчетный – отчетливый
(отчетный – относящийся к отчету:
отчетный период; отчетливый – хорошо
различимый: отчетливый звук) и т.п.

Лексика языка обогащается постоянно.
Под влиянием тех процессов, которые
произошли в нашем обществе в перестроечный
и постперестроечный периоды наш язык
стал интенсивно пополняться новыми
словами.

Среди общественно значимых политических
терминов, рожденных перестройкой, можно
назвать такие: новое политическое
мышление, общеевропейский дом, всеобщая
система международной безопасности,
либерализация, поименное голосование,
общечеловеческие ценности, арендаторство,
демонополизация, договорная цена, единое
экономическое пространство, зона
свободного предпринимательства, дельцы
теневой экономики, «деревянные» рубли
и т.д.

Особую трудность составляет для говорящих
незнание или неточное знание значений
заимствованных слов. Например: аквапарк,
андерграунд, анимационный, армрестлер,
армрестлинг, аудит, аудитор, аэрогриль,
толинг, толингер, дилер, дистрибьютер,
клиринг, ноу-хау, паблисити, маркетинг,
лизинг.

Говоря о лексических нормах русского
языка наших дней нельзя не указать на
увлечение словами и словечками,
находящимися далеко за пределами
культурного общения, независимо от
степени их новизны в языке. К сожалению,
средства массовой информации захлестнул
поток просторечных, жаргонных, а нередко
и откровенно непристойных слов и
выражений. Для уточнения лексических
норм современного литературного языка
рекомендуется использовать толковые
словари русского языка, специальную
справочную литературу.

Таким образом, говоря о новых словах в
русском языке и трудностях их употребления,
следует еще раз подчеркнуть необходимость
более точного знания семантики
новообразований, особенно терминов,
умения пользоваться заимствованиями,
что требует особой осторожности. Следует
быть также внимательными при употреблении
наименований, имеющих обиходное и
правовое значение и не смешивать их.

Нужно также иметь в виду, что многие
слова изменили свои оценочные свойства
и стилистическую окраску, сферу
употребления, приобрели необычайную
частотность, даже стали «модными», или
утратили былую употребляемость. Наконец,
постарайтесь в своей речи избавиться
от просторечных слов и жаргонизмов.

Контрольные вопросы:

  1. Дайте
    определение понятия «норма литературного
    языка». Перечислите характерные
    особенности нормы.

  2. Расскажите
    о вариантах норм литературного языка.

  3. Охарактеризуйте
    орфоэпические нормы. Расскажите об
    особенностях русского ударения.

  4. Охарактеризуйте
    грамматические нормы. Употребление
    каких морфологических форм вызывает
    у вас затруднения?

  5. В
    чем состоит наибольшая трудность в
    употреблении новых слов?

  6. Как
    вы относитесь к словам, которые находятся
    за пределами русского литературного
    языка (инвективная и нецензурная
    лексика)? Употребляете ли вы их и в каких
    случаях?

Практические
задания

Задание
1.
Какой звук
([о] или [э]) произносится под ударением
в следующих словах? Там, где необходимо,
поставьте две точки над Е. Пользуйтесь
справочными материалами.

Опека, безнадежный,
блеклый, наемник, атлет, маневр, никчемный,
одновременный, разновременный,
одноименный, преемник, бытие, затекший,
острие, обнесший, принесший, местоименный,
новорожденный, оседлый, платежеспособный,
афера, дареный, свекла, наперченный,
щелка, зев, отцветший.

Задание
2.
Найдите
ошибки, которые появились в связи с
неправильным образованием слов. Исправьте
предложения.

1. Змеи относятся
к животным класса пресмыкающих.

2. При гололеде
будьте внимательны на дорогах, чтобы
не подскользнуться и не упасть прямо
на проезжей части.

3. Эта история
может стать самым смотрибельным сериалом.

4. У него был
вспыхчивый характер.

  1. Его предсмертельные
    слова произвели большое впечатление.

Задание
3.
Исправьте
ошибки, вызванные неправильным
образованием падежной формы количественных
числительных.

1. Библиотека
института ежемесячно пополняется триста
книгами. 2. Во время полета была
выполнена широкая программа
медико-биологических исследований,
включавшая в себя около пятьдесят
экспериментов. 3. С семи русских кораблей
успели выпустить по неприятелю более
восемьсот снарядов. 4. Протяженность
столичной подземной дороги достигла
сто семьдесят три километра, а автобусных
и троллейбусных линий четыре тысячи
триста километров. 5. Вместе с новыми
тысяча двухсот тремя словами учебник
немецкого языка будет насчитывать свыше
четыре с половиной тысячи слов.

Задание
4.
Выберите
из скобок один из вариантов; подчеркните
его.

  1. Водить автобусы
    по горным дорогам могут только опытные
    (шоферы – шофера).

  2. (Выборы – выбора)
    губернатора области показывают, что
    многие политические противники заключают
    (договоры – договора) о сотрудничестве.

  3. В летнее время
    у многих людей начинаются (отпуски –
    отпуска).

  4. В нашем кондитерском
    отделе всегда свежие (торты – торта).

  5. Многие (цехи –
    цеха) завода переориентированы на
    выпуск новой продукции.

Задание
5.
Образуйте
от приведенных ниже глаголов формы
1-го лица единственного числа настоящего
времени; 3 лица единственного числа
настоящего времени.

Образец:
люблю – любит – любят.

Бегать, бежать,
бриться, ехать, ездить, класть, махать,
победить

сыпать, хотеть

Задание
6.
Назовите
и запишите слова, употребляющиеся в
русском языке со следующими
интернациональными словообразовательными
элементами.

Авиа…(лат.avis
— птица), авт(о)… (греч.
autos
— сам), агр(о)…(греч.
agros
— поле), аква…(лат.
aqua
— вода), анти…(греч.
anti
— против), био…(греч.
bios
— жизнь), дем(о)… (греч.
demos
— народ), интер…(греч.между), …крат
(греч.
kratos
— власть), микр(о)…(греч.
mikros
-малый), мон(о)…(греч.
monos
— один), орф(о)…(греч.
orthos
— прямой, правильный), поли…(греч.
poli
— много), …скоп (греч.
skopeo
— смотрю), …тек(а) (греч.
theke
— вместилище), тип…(греч.
typos
— отпечаток, образ), ультра…(лат.
ultra
— сверх), фил(о)…(греч.
philos
— люблю), экстра…(лат.
extra
-вне).

Задание
7.
Знакомы
ли вам значения этих иностранных слов?
Употребите их в предложениях.

Абракадабра,
авантюризм, ажиотаж, бедлам, блеф,
вакансия, варьировать, виртуальный,
вульгарный, дебютант, деликатес,
анонимный, конфиденциальный, экстренный,
экстремальный.

Задание
8.
Установите
различия в экспрессивно-стилистической
окраске синонимов. Составьте предложения
с каждым из данных слов.

Сообщать,
возвещать, доводить до сведения,
выкладывать; вкушать, есть, уплетать,
принимать пищу; глаза, зенки, очи, глазки;
уста, губы, губки; книга, книжечка,
фолиант, книжонка; послание, письмо,
письмецо; зачинатель, инициатор, зачинщик.

Задание
9.
Установите
разницу в эмоционально-экспрессивной
окраске слов.

Старец – старик;
ложе – постель; поведать – сообщить;
возложить – положить; стяг – знамя; дар
– подарок; кончина – смерть; дерзание
– стремление.

Задание
10.
Найдите
ошибки в употреблении следующих
словосочетаний. Исправьте их.

  1. Отчитаться в
    израсходовании полученного аванса.

  2. Отзыв на статью.

  3. Уделять внимание
    на каждую мелочь.

  4. Превосходство
    новой техники перед старой.

  5. Примириться
    перед неумолимой судьбой.

  6. Уверенность в
    торжество справедливости.

Задание
11.
Исправьте
ошибки в употреблении падежных форм и
предлогов.

1. Об этом мы познакомим вас позже.

2. Докладчик
подчеркнул о необходимости строительства.

3. Высказывается
критика о том, что мы делаем.

4. Решимость
парламента свелась лишь в предоставлении
полномочий.

5. Магазин,
ориентированный специально для
школьников.

6. Уверенность
каждого за свой завтрашний день.

7. Контролировать
за ходом лечения.

8. Озабоченность
за безопасность границ.

Задание
12.
Исправьте
ошибки в предложениях.

1. Он учился в школе хорошо, благодаря
своих хороших способностей.

2. Девушка очень
тосковала по брате.

3. Первый год по
приезду его жизнь складывалась нормально.

4. Он выехал из
города согласно предписания.

5. На этот вечер
смогли приехать престарелые люди со
всех районов города.

6. В отделе критики
и библиографии «толстых» журналов
систематические публикуются обзоры о
выходящих новинках художественной
литературы.

7. Представитель
строительной организации вновь заверил
заказчика о своей готовности выполнить
работу в срок.

  1. Факты, о которых
    изложил автор письма, при проверке
    полностью подтвердились.

9. Студенты уделяют
внимание на записи во время лекций.

10. Преподаватель
указал о том, что в работе имеется ряд
ошибок.

1) Глаголы с разным управлением в качестве однородных членов предложения:

Слова с разным управлением могут использоваться в качестве однородных членов, если каждое из них имеет свои зависимые слова, употреблённые в нужном падеже. Например:

Она ждала (кого?) его (В.п.) и звонила (кому?) ему (Д.п.) весь вечер.

Часто построить такие предложения правильно помогают местоимения:

Дети редко прислушиваются  (к чему?) к советам (Д.п. с предлогом к) взрослых и следуют (чему?)им  (Д.п.).
Люди были измучены, но верили 
(во что?) в победу (В.п. с предлогом в) и надеялись (на что?) на неё (Д.п. с предлогом на).

2)Разное управление у глаголов и  существительных:

интересоваться искусством (Т.п.) — интерес к искусству (Д.п.)
любить литературу (В.п.) — любовь к литературе (Д.п.)
ненавидеть беспорядок (В.п.) — ненависть к беспорядку (Д.п.)
уважать коллег (В.п.) — уважение к коллегам (Д.п.)
доверять другу (Д.п.) — доверие к друзьям (Д.п.)
сочувствовать слабому (Д.п.) — сочувствие к слабому (Д.п. с предлогом)

3). Разное управление у слов со сходным значением:

беспокоиться о чём-либо (П.п.) — тревожиться за что-либо (В.п.)
оплатить что-либо (В.п.) — заплатить за что-либо (В.п.  с предлогом 
за)
уплатить за проезд (В.п. с предлогом 
за) – оплатить проезд (В.п.)
уделить внимание чему-либо (Д.п.) — обратить внимание на что-либо (В.п. с предлогом 
на)
сообщать кому-либо (Д.п.) — информировать кого-либо (В.п.) 
упрекать в чём-либо (П.п. с предлогом 
в) — осуждать за что-либо (В.п. с предлогом за)
отзыв о чём-либо (П.п. с предлогом 
о) — рецензия на что-либо (В.п. с предлогом на)
вера во что-либо (В.п. с предлогом
 в (во)) — уверенность в чём-либо (В.п. с предлогом в)
свойственный чему-либо (Д.п.) — характерный для чего-либо (Р.п. с предлогом 
для)  

4) Разное управление у глаголов с отрицанием и без него:

Замечать реакцию (В.п.) – не заметить реакции (Р.п.)
Он замечал (что?) реакцию зала на его слова. — От волнения он не замечал (чего?) реакции зала на его слова.

5) Употребление названий произведений литературы и искусства.

В «Войне и мире» актёр В. Тихонов сыграл роль князя Андрея. В кинофильме «Война и мир» актёр В. Тихонов сыграл роль князя Андрея. 
После слов — родовых обозначений, например, сказка, роман, повесть, рассказ, картина, кинофильм и других подобных, называющих жанр произведения литературы или искусства, имя собственное ставится в именительном падеже. Если такие слова в предложении отсутствуют, то названия произведений литературы и искусства используются в тех падежах, каких необходимо для контекста предложения.

В сказке «Репка» либо: В «Репке»
В кинофильме «Война и мир» либо: В «Войне и мире»
В балете «Щелкунчик» либо: В «Щелкунчике

ЗАПОМНИ
Многие глаголы требуют после себя определённого падежа существительного.

Родительного падежа требуют глаголы:

Достигать, добиваться, желать, жаждать, хотеть, ожидать, наделать, опасаться, остерегаться, бояться, избегать, лишаться, пугаться, стыдиться, сторониться, стоить, искать, просить, требовать  и др. (кого? чего?) 

Глаголы с отрицанием: не видеть, не замечать, не слышать и др. (кого? чего?) 

Дательного падежа требуют глаголы:

кому? чему?

Давать, верить, доверять, угрождать, потакать, учиться, радоваться, улыбаться, говорить, отвечать, грозить, угрожать, возражать, кланяться, кивать, махать, сигналить, звонить, писать, говорить, рассказывать, объявлять, отвечать, объяснять, сообщать, нравиться, казаться, мешать, вредить, изменять,  мстить, надоедать, опротиветь, дарить, покупать, приносить, посылать, показывать, помогать, обещать, сниться и др. (кому? чему?)

Винительного падежа требуют все переходные глаголы:

кого? что?

Давать, дарить, продавать, покупать, посылать, показывать, обещать, строить, шить, убирать, мыть, стирать, брать, класть, ставить, вешать, видеть, смотреть, слышать, слушать, чувствовать, испытывать, замечать, любить, ненавидеть, презирать, уважать, ценить, помнить, понимать, изучать, решать, учить, рассказывать, объяснять, сообщать, говорить, благодарить, поздравлять, вспоминать, встречать, ругать, ждать  и др. (кого? что?) 

Творительного падежа требуют глаголы:

кем? чем?

Править, руководить, распоряжаться, командовать, управлять, верховодить, заведовать, увлекаться, интересоваться, заниматься, любоваться, восхищаться, восторгаться, наслаждаться, гордиться, восторгаться, восхищаться, пленяться, дорожить, владеть, пользоваться, обладать, овладевать, хвалиться, гордиться, хвастаться, клясться, торговать, жертвовать, рисковать, быть, стать, становиться, явиться, оказаться, остаться, считаться, слыть, называться и др. (кем? чем?)

Для многих глаголов характерно двойное управление:

Дать, передать, вручить, выдать, продать, возвратить, дарить, сдать, предоставить, вверить, уступить, оставить кому-либо что-либо
Сказать, объяснить, объявить, внушить, рассказать, заявить, ответить, обещать, рекомендовать кому-либо что-либо
Обещать, гарантировать кому-либо что-либо
Учить кого-либо чему-либо
Считать, вообразить, признать, представить, называть, изображать, обругать, объявить кого-либо кем-либо

Варианты норм

Хотеть, желать, жаждать, просить, заслуживать награду — награды (В.п. и Р.п), но: заслужить награду (В.п.)
Спрашивать совета, разрешения – совет, разрешение (Р.п. и В.п.)
Ждать поезда, звонка – поезд, звонок (Р.п. и В.п.), но ждать бабушку, сестру (В.п.)
Дать, брать, добыть, получить, прислать, купить, положить, налить, насыпать, выпить, глотнуть, попробовать воду, сахар – воды, сахара (В.п. и Р.п.)

ВНИМАНИЕ

 Скучать (по чему? кому?) по работе, по дому, по матери, по мужу

Но с местоимениямискучать (по ком?) по нас, по вас.

2. Конструкции с предлогами

 1) предложное  управление:

благодаря, согласно, вопреки, наперекор, подобно + Д.п. существительного,

 Например: вопреки указанию, правилам, мнению близких, согласно приказу…

по (в значении «после чего-либо») + П.п. существительного,

 Например: по приезде, по возвращении, по завершении эксперимента…

в меру, в силу, в течение, в продолжение, в заключение, по причине, по завершении, наподобие, посредством + Р.п. существительного,

Например: в продолжение разговора, в течение недели...

2) использование предлогов в беспредложных конструкциях:

Статья послужила толчком мыслям (нельзя: к мыслям).
Заказчик выставил рекламацию заводу-изготовителю (нельзя: к заводу-изготовителю)

3) предлоги при однородных членах предложения:

Мне нужно на вокзал, на почту и в магазин.
Занятия проходят на стадионе, в парке и в зале.

Если требуются разные предлоги с разными существительными, они должны быть использованы. Пропуск предлогов в таких случаях недопустим.

4) предлоги в, на, с, из:

В Камергерском переулке, магазин в Камергерском — на улице, магазин на Тверской

Из Ростова, из театра, из магазина, из парка, из ссылки, из цирка, из клуба, из консерватории, из ресторана, из школы, из класса, из аэропорта, из порта, из института, из университета, из библиотеки, из больницы — с юга, с площади, с бульвара, с почты, с рынка, с лекции, с представления, с концерта, со станции, с вокзала

Задание №1: Найдите ошибки в употреблении словосочетаний, исправьте их:

1. Отчитаться в израсходовании полученного аванса.

2. Отзыв на статью.

3. Уделять внимание на каждую мелочь.

4. Превосходство новых технологий перед старыми.

5. Примириться перед неумолимой судьбой.

6. Рассержен за выходку приятеля.

7. Уверенность в торжество справедливости.

Задание №2. Составить словосочетания, поставив заключенные в скобках слова в нужном падеже:

Удивляться (результаты)

Преклонение (талант)

Платить (квартира)

Упрекать (грубость)

Уверенность (победа)

Оплатить (проезд)

Заведовать (отделение)

Управляющий  (филиал)

Согласно (приказ, распоряжение)

Вера (победа)

Стремление (деятельность)

Тема урока: Обобщение и систематизация изученного по теме «Синтаксис и пунктуация простого предложения»

Цели урока:

I. Познавательные:

1)  Повторить сведения об особенностях простого предложения, пунктуацию простого осложнённого предложения, повторить попутно типы речи, стили речи.

2)  Повторить орфоэпические и грамматические нормы русского языка.

II. Практические:

1)  Обобщить и систематизировать ранее полученные знания по теме «Синтаксис и пунктуация простого предложения».

2)  Закрепить умение расстановки знаков препинания в простом осложнённом предложении.

3)  Формировать умение применять знание норм языка в речи.

4)  Учить выявлять и формулировать проблему на основе исходного текста — высказывания писателя.

III. Общепредметные:

1)  Воспитывать:

а) интерес к предмету в ходе освоения лингвистической темы и размышления над содержанием дидактического материала о языке и о вреде жаргонизмов (высказывания о языке, аудиофрагмент разговора, записанного на улице);

б) бережное отношение к родному языку;

в) языковую личность учащегося, развивающего разные виды компетентности (лингвистической, языковой, речевой, коммуникативной);

г) нравственные качества учащихся (понимание значимости языка как величайшего дара и осознание необходимости бережного отношения к языку);

способность выработать определённый взгляд на нравственные понятия, определённое отношение к ним, эстетическое восприятие высказываний писателей о языке.

2)  Развивать:

а) логическое мышление (мыслительные операции: анализ, синтез, сравнение, обобщение, классификация, систематизация); речь, эмоциональную сферу.

3)  Формировать:

а) умение строить высказывание по заданным критериям;

б) оценивать степень трудности предложенных заданий, уровень собственных знаний, умений.

Тип урока: урок обобщения и систематизации знаний, умений, навыков

Лингвистическая тема: Синтаксис и пунктуация простого предложения.

Коммуникативная тема: коммуникативно-организационный дидактический материал на тему «Русский язык».

Технология урока: комплексная (работа над орфографическими и грамматическими нормами в группах, систематизация изученного с составлением обобщающей схемы и поэлементной отработкой навыков и умений, тестовый текущий контроль, продуцирование собственного высказывания).

Средства обучения:

  • Учебник (Бабайцева В.В. Русский язык. 10-11 кл.: учебник для общеобразоват. Учреждений филол. Профиля. – М.: Дрофа, 2011).

  • Дидактический материал для работы в группах над орфоэпическими и грамматическими нормами (на карточках и слайдах презентации).

  • Аудиозапись речевого образца (разговор, записанный на улице).

  • Обобщающая таблица «Простое осложненное предложение».

  • Таблицы «Речевые и грамматические ошибки».

  • Карточки с заданиями и бланк ответов для итогового теста урока «Пунктуация в простом предложении».

  • Высказывания о русском языке русских писателей (слайды презентации к уроку).

  • Распечатка текста и заданий к нему для выполнения домашней работы.

  • Лист учёта знаний для осуществления рефлексии на уроке.

  • Мультимедийная презентация к уроку.

Используемые технологии и методики: технология проблемного обучения, критического мышления, информационные технологии (ЭОР в виде мультимедийной презентации и видеосюжета), научно-поисковый и наглядный методы.

Ход урока.

  1. Организационный момент (проверка подготовки класса, учащихся к уроку, отметка отсутствующих).

II. Повторение.

Эпиграф.

Язык открывает свои чертоги и кладовые только

людям с раскрытой душой, тем, у кого сердце не

зачерствело в однообразии покоя и борьбы, кто

искренен не только с другими, но и с самим собою

В. И. Белов.

Вопрос.

Как вы думаете, что будет темой нашего сегодняшнего разговора? (Русский язык).

Послушайте диалог, записанный на улице.

Вопрос.

− На какие невесёлые размышления наводит разговор на улице?

− Допустимо ли употребление жаргонизмов в литературной речи? Почему?

(Затрудняет понимание речи и ограничивает возможности языка).

Вывод:

Соблюдение норм литературного языка является необходимым условием его сохранения и развития.

Задание № 1.Работа по группам над орфоэпическими нормами.

Произнесите следующие слова:

Группа № 1.

Афера, затекший, опека, оседлый, маневры.

Группа № 2.

Новорожденный, острие, платежеспособный, дареный, одноименный.

Группа № 3.

Свекла, многоженство, блеклый, никчемный, безнадежный.

Группа № 4.

Белесый, заем, двоеженец, договоренность, истекший.

Группа № 5.

Житие, забредший, испекший, щелка, бытие.

Ответы:

  1. Афера, затекший, опека, оседлый, маневры.

  2. Новорождённый, остриё, платёжеспособный, дарёный, одноимённый.

  3. Свёкла, многожёнство, блёклый, никчемный, безнадёжный.

  4. Белёсый, заём, двоежёнец, договорённость, истёкший.

  5. Житие, забредший, испекший, щёлка, бытие.

Задание № 2. Работа над грамматическими нормами.

А) Допишите окончания, согласуя сказуемое с подлежащим.

Группа № 1.

1. Большинство картин художников — передвижников был.. воспринят.. как гимн человеку, прославляющий и утверждающий его высокие духовные ценности.

Группа № 2.

2.  Часть студентов уже сдал.. экзамены по специальности.

Группа № 3.

3.  Двадцать человек стоял.. в стороне.

Группа № 4.

4.  Множество темноватых туч расползал..сь по небу.

Группа № 5.

5.  Десять студентов окончил.. институт с отличием.

Ответы.

  1. Большинство картин художников — передвижников было воспринято как гимн человеку, прославляющий и утверждающий его высокие духовные ценности.

  2. Часть студентов уже сдала экзамены по специальности. 

  3. Двадцать человек стояли в стороне. 

  4. Множество темноватых туч расползалось по небу. 

  5. Десять студентов окончили институт с отличием.

Б) Исправьте ошибки в употреблении словосочетаний.

Группа № 1.

1. Отчитаться в получении полученного аванса.

Группа № 2.

2. Отзыв на статью.

Группа № 3.

3. Уделять внимание на каждую мелочь.

Группа № 4.

4. Превосходство новой техники перед старой.

Группа № 5.

5. Примириться пред неумолимой судьбой.

Ответы.

  1. Отчитаться (о чём?) о получении полученного аванса.

  2. Отзыв (о чём?) о статье.

  3. Уделять внимание (чему?) каждой мелочи.

  4. Превосходство новой техники (над чем?) над старой.

  5. Примириться (с чем?) с неумолимой судьбой.

В) Исправьте ошибки в употреблении падежных форм и предлогов.

Группа № 1.

Об этом мы познакомим вас позже.

Группа № 2.

Докладчик подчеркнул о необходимости строительства.

Группа № 3.

Решительность парламента свелась лишь в предоставлении полномочий.

Группа № 4.

Вчера брат приехал с Москвы.

Группа № 5.

По окончанию переговоров представители сторон подписали совместное заявление.

Ответы.

  1. С этим мы познакомим вас позже. 

  2. Докладчик подчеркнул необходимость строительства. 

  3. Решительность парламента свелась лишь к предоставлению полномочий. 

  4. Вчера брат приехал из Москвы. 

  5. По окончании переговоров представители сторон подписали совместное заявление.

Г) Исправьте ошибки в употреблении деепричастного оборота.

Группа № 1.

Объяснение этих явлений может быть найдено, взяв в качестве иллюстрации последние события.

Группа № 2.

Проработав всего два месяца, у него возникли осложнения с начальником цеха.

Группа № 3.

Приняв смену, дежурные водители направляются диспетчером по объектам.

Группа № 4.

Торговый зал был очищен от людей, опасаясь, что рухнет потолок.

Группа № 5.

Изучая процесс разгосударствления, обнаруживается определённая закономерность.

Ответы.

  1. Объяснение этих явлений можно найти, взяв в качестве иллюстрации последние события. 

  2. После двух месяцев работы у него возникли осложнения с начальником цеха.

  3.  Приняв смену, диспетчер направляет дежурных водителей по объектам. 

  4. Торговый зал был очищен от людей из-за опасений, что рухнет потолок. 

  5. Изучая процесс разгосударствления, можно обнаружить определённую закономерность.

Д) Внесите стилистическую правку в предложения с однородными членами.

Группа № 1.

Магазину требуются продавцы овощей и картофеля.

Группа № 2.

ООО Севастопольский приглашает на постоянную работу мастера по переработке овощей и одиноких рабочих (мужчин и женщин).

Группа № 3.

В работу нужно включить как можно больше людей и лошадей, которые управлялись бы с уборкой картофеля.

Группа № 4.

Рабочие завода поддерживают и голосуют за кандидатуры, выдвинутые участниками митинга.

Группа № 5.

Автобус имеет высокие подножки и низкие динамические показатели.

Ответы.

  1. Магазину требуются продавцы овощей: картофеля и др.

  2. ООО Севастопольский приглашает на постоянную работу мастера по переработке овощей и рабочих (мужчин и женщин). 

  3. В работу нужно включить как можно больше людей, которые управлялись бы с уборкой картофеля; необходимо использовать лошадей для облегчения труда работников. 

  4. Рабочие завода поддерживают кандидатуры, выдвинутые участниками митинга, и голосуют за них. 

  5. Автобус имеет высокие подножки, у него низкие динамические показатели.

  1. Проверка домашнего задания.

Прочитайте высказывания о русском языке, принадлежащие великим мастерам слова.

Какое высказывание кажется вам наиболее актуальным в наше время и почему?

Во дни сомнений, во дни тягостных раздумий о судьбах моей родины,- ты один мне поддержка и опора, о великий, могучий, правдивый и свободный русский язык! Не будь тебя — как не впасть в отчаяние при виде всего, что совершается дома? Но нельзя думать, чтобы такой язык не был дан великому народу! (И. С. Тургенев).

Нет слова, которое было бы так замашисто, бойко, так вырывалось бы из — под самого сердца, так бы кипело и животрепало, как метко сказанное русское слово. (Н. В. Гоголь).

Язык — это история народа. Язык — это путь цивилизации и культуры. Поэтому — то изучение и сбережение русского языка является не праздным занятием от нечего делать, но насущной необходимостью. (А. И. Куприн).

I. Прочитайте микротекст и выполните задания к нему.

(1)  Русский язык (как любой другой национальный язык) представляет собой систему знаков, с помощью которой мы общаемся с представителями своего народа. (2) И, таким образом, он становится неотъемлемой частью нашего коллективного сознания, нашего менталитета. (3) Поэтому вместе с такими культурными элементами, как обычаи, нравы, национальное искусство, язык определяет нацию как выделенный среди иных народов объект.

Какую цель, по-вашему, ставит автор этого текста:

1)  описать картину, нарисовать образ,

2)  передать последовательность действий, событий,

3)  донести до читателя свои мысли, рассуждения,

4)  познакомить читателя с научными сведениями и сделать умозаключение.

Укажите тип речи.______________________________

Укажите стиль речи. ____________________________

Ответы.

1. 4.

2. Рассуждение.

3. Научно — популярный.

II.  Систематизация изученного по теме «Синтаксис и пунктуация простого предложения».

  1. Конструирование предложений.

Синтаксический разбор предложения.

Мы из всех исторических катастроф вынесли и сохранили в чистоте великий русский язык, он передан нам нашими дедами и отцами. (В. Шукшин).

.

Перестройте его дважды таким образом, чтобы изменилась его характеристика. Дайте общую характеристику получившимся предложениям.

Ответы.

А) Мы из всех исторических катастроф вынесли и сохранили в чистоте великий русский язык, переданный нам нашими дедами и отцами. (Простое, осложнённое).

Б) Мы из всех исторических катастроф вынесли и сохранили в чистоте великий русский язык, который передан нам нашими дедами и отцами. (Сложноподчинённое с придаточным определительным).

  1. Составление обобщающей схемы «Простое осложнённое предложение».

Вопрос.

Чем может быть осложнено простое предложение?

(- словами, грамматически связанными с членами предложения:

— однородными, обособленными и уточняющими членами;

— словами, грамматически не связанными с членами предложения: обращениями и междометиями, вводными словами и предложениями, вставными конструкциями).

3. Повторение сведений и выполнение тренировочных упражнений.

А) Однородные члены предложения.

Дайте определение однородных членов предложения. СЛАЙД

  • Однородные члены  это ряд одинаковых членов предложения, соединённых между собой сочинительной связью, которая выражается союзами или только интонацион­но.

  • Между однородными членами при отсутствии союза ставится запятая.

Вопрос.

Когда между однородными членами запятая:

— ставится,

— не ставится?

Ответ. СЛАЙД.

Запятая ставится:

  • перед противительными союзами (не марки, а открытки);

  • перед повторяющимися союзами (и марки, и открытки);

  • перед второй частью двойных союзов (как марки, так и открытки);

  •  перед союзом да и, обозначающим добавление к сказанному раньше.

Запятая не ставится:

  • перед однородными членами, соединёнными одиночными союзами (соединительными или разделительными);

  • запятая не ставится во фразеологических оборотах (и так и сяк).

Задание № 1. СЛАЙД

А) Составьте схемы предложений (знаки препинания не расставлены):

1.  Рыбалка успокаивает нервы настраивает на философский лад.

2.  Схватил он его за ноги и руки спеленал поправил все перевязки завернул его в воловью кожу увязал и помчался с ним в дорогу.

3.  Острым ножом мы вырезали узоры то колечко то спираль то шахматную клеточку.

4.  Летом и зимой весной и осенью лес хорош во все времена года.

5.  В каждом отсеке школы на лестницах на этажах в кабинетах везде стояли дежурные.

Б) Сравнительный оборот.

Расскажите о знаках препинания при сравнительных оборотах. СЛАЙД

  • Сравнительный оборот является частью простого распространённого предложения.

  • Сравнительные обороты выделяются запятыми.

  • Если оборот входит в состав сказуемого или тесно связан с ним по смыслу, то запятыми он не выделяется.

  • Не обособляется оборот, если он входит в состав устойчивого словосочетания.

Задание № 2. СЛАЙД

Тест 1. Укажите предложение с пунктуационной ошибкой, объясните свой выбор:

Озеро, как море, шумит при сильном ветре. Перед нами расстилались пёстрые, как ковёр, луга. Осыпался с берёзы лист и, как ковёр, устлал дорогу. Они были похожи, как две капли воды.

Ответ. 4.

Тест 2. Укажите предложение, в котором на месте всех пропусков ставятся запятые.

1.  Внизу _как зеркало _стальное синеют озера струи.

2.  На чёрном_как тушь_небе не мелькает ни одна звёздочка.

3.  Как парус_чайка_там белеет в высоте.

4. Вода в реке была чёрной_словно_дёготь.

Ответ.2.

В) Обособленные члены предложения.

Ø  Обособленные определения и приложения.

Задание № 3.

Запишите предложения. Объясните знаки препинания.

Группа № 1. Незримый, ты мне был уж мил.

Люди, злые, голодные, двинулись по улице.

Группа № 2. А он, мятежный, ищет бури.

Привлечённые светом, бабочки летели к свече.

Группа № 3. Он видел небо, покрытое тучами.

Иван, усталый, всё же не мог заснуть.

Группа № 4. На небе стыли облака, ещё розовые от заката.

Пёстро раскрашенный, стоял катер у причала.

Группа № 5. Анна Ивановна, в шляпе и шали, вернулась с прогулки.

Он ушёл сильно расстроенный.

Задание№ 4. СЛАЙД

Запишите предложения. Замените обособленные определения необособленными (знаки препинания не расставлены). Какой вариант, по — вашему, более уместен? Какой приём используют писатели? С какой целью? (Инверсия).

Только люди способные сильно любить могут испытывать и сильные огорчения; но та же способность любить служит для них противодействием горести и исцеляет их. (Л.Н. Толстой). 

Песня тихая тягучая и заунывная похожая на плач и едва уловимая слухом слышалась то справа то слева то сверху то из — под земли. (А.П. Чехов).

Задание № 5.

Из прочитанных предложений выпишите приложения вместе со словами, которые связывают приложения с другими словами. Составьте предложение с одним из записанных приложений.

Мы, экономисты, народ увлекающийся. Зашёл сосед Иван, поклонник Высоцкого. Всё дорожает белое золото, или хлопок. Иванову, как художнику, поручили оформить холл. Мой пёс, по кличке Сват, спит.

Ответ.

Экономисты, поклонник Высоцкого, или хлопок, как художнику, по кличке Сват.

Ø  Обособленные обстоятельства.

Расскажите об обособлении обстоятельств. СЛАЙД

  • Обособляются обстоятельства, выраженные деепричастными оборотами и одиночными деепричастиями.

  • Всегда обособляются обстоятельства с предлогом несмотря на, которые имеют уступительное значение.

  • Не обособляются фразеологические обороты и деепричастия, непосредственно примыкающие к сказуемому и близкие по функции к наречиям (стоя, сидя, лёжа, молча, нехотя и др.).

  • Могут обособляться обстоятельства, выраженные существительными с предлогами благодаря, согласно, вопреки, в силу, в случае, при наличии, при отсутствии, по причине, ввиду, вследствие, если они распространены и стоят перед сказуемым.

Задание № 6. Тесты

1. Укажите правильное объяснение пунктуации в предложении. СЛАЙД

Весело махая хвостом ( ) и тихо повизгивая, Каштанка ждала своего хозяина.

1. Здесь один деепричастный оборот, запятая ставится.

2. Причастный оборот перед определяемым словом, запятая не ставится.

3. Предложение сложное, запятая ставится.

4. Два деепричастных оборота, соединённых союзом и, запятая не ставится.

2. Укажите предложение, в котором ставится одна запятая. (Знаки препинания не расставлены.) СЛАЙД

На севере слабо мерцая подымались какие-то белые облака. Он всегда краснел вспоминая эту историю. Воробей заметив кота мгновенно взлетел вверх. Часто он читал лёжа.

3. Укажите предложение, в котором ставится одна запятая. (Знаки препинания не расставлены.) СЛАЙД

Несмотря на запрещение она пошла к реке надеясь развеять свою печаль. Он женился и вопреки пословице не переменился. Расправив крылья и вытянув шею гусь шёл прямо на меня. Садовник подрезав ветки яблонь закрыл калитку сада и ушёл в дом.

Ø  Обособленные дополнения.

Расскажите об обособлении дополнений. СЛАЙД

Вопрос.

В каком значении употреблены дополнения с предлогами в следующих предложениях? СЛАЙД

1. Все включая новичков очень старались.

2. Он не видел ничего кроме её смеющихся глаз.

3. Иван в отличие от Вани человек солидный.

4. Вместо бритых скул была у него теперь бородка.

5. Я работал вместо него.

Задание № 7.

Сделайте вывод об обособлении дополнений. Запишите предложения, расставьте знаки препинания.

Г) Уточняющие члены предложения

Вопрос.

Какие члены предложения называются уточняющими? Какими словами могут присоединяться уточняющие члены? СЛАЙД

  • Уточняться могут все члены предложения: и главные, и второстепенные.

  • Уточняющие члены предложения могут быть присоединены к уточняемым словам союзами: то есть, или (в значении «то есть»), иначе, именно и др., а также словами: особенно, даже, в частности, в том числе, например и др.

Задание № 8.

Запишите предложение, расставив знаки препинания:

Впереди нас шагах в тридцати видна полынья из глубины которой валит морозный пар.

Задание № 9.

Составьте предложения с уточняющими членами:

Несмотря на утренний туман, шагах в тридцати от нас, в частности учащиеся физмата, в том числе русский язык, то есть наш сосед.

Д) Вводные слова и предложения.

Вопрос.

На какие группы делятся вводные слова по значению? Как они могут выделяться в предложении?

  • Степень уверенности (большая или меньшая)

  • Различные чувства

  • Источник сообщения

  • Порядок мыслей и их связь

  • Способы оформления мыслей

Задание № 10.

Разграничьте в следующих парах предложений вводные слова и синонимичные им члены предложения. СЛАЙД

Наконец он приехал.- Наконец можно обратиться к врачу за помощью. Выбор варианта с моей точки зрения удачен.- Я знаю, что совершено преступление, если смотреть на вещи с точки зрения общей морали.

IV. Подведение итогов.

Итоговый тест по теме «Пунктуация простого предложения»

__________________________________________________________

1. Укажите предложение без пунктуационной ошибки.

1)  Наша школа как корабль.

2)  Изделия Фаберже подлинные произведения искусства.

3)  Интеллигентность это способность понимать.

4)  Красота — есть обещание счастья.

2. Укажите предложение с пунктуационной ошибкой.

1)  Площадь комнаты — двадцать квадратных метров.

2)  Жить — Родине служить.

3)  Пятью пять — двадцать пять.

4)  Любовь — есть источник поэзии, добра и ненависти.

3. Укажите правильное объяснение пунктуации в предложении.

Незаметно зажглась одна звезда () и тихо дрожит в тёмном сумраке небес.

1)  В этом предложении союзом и соединены однородные члены, перед и ставится запятая.

2)  Это предложение сложное, перед и ставится запятая.

3)  В этом предложении союз и соединяет однородные члены, запятая перед и не ставится.

4)  Это предложение сложное, запятая перед и не ставится.

4. Укажите, на месте каких цифр в предложении должны стоять запятые.

Няня повествовала (1) с пылом (2) живописно (3) с увлечением (4) и местами вдохновенно.

1)  1,2,3 2) 3,4 3) 2,3 4) 2,3,4

5. Укажите, какие знаки препинания нужны в предложении на месте цифр.

Всеобщее признание принесли Поленову его картины (1) «Московский дворик» (2) «Бабушкин сад» (3) «Заросший пруд».

1)  1 — запятая,2 — запятая, 3 – запятая;

2) 1 — двоеточие, 2 — запятая, 3 – запятая;

3) 1 — тире, 2 — запятая, 3 – запятая;

4)1 — запятая, 2 — запятая, 3 – тире

6. Укажите, в каком предложении ставится тире. (Знаки препинания не расставлены.)

1)  Ближе к осени все опустело и дом и усадьба.

2)  Острым ножом мы вырезали узоры то колечко то спираль то шахматную клеточку.

3)  В одной из комнат жили птицы Черноголовки малиновки и даже соловей.

4)  Летом и зимой весной и осенью лес хорош во все времена года.

7. Укажите предложение с пунктуационной ошибкой.

1)  Своего начальника он боялся как огня.

2)  Двор как плац.

3)  С утра поползли серые как дым, облака.

4)  Морозило ещё сильнее, чем вчера.

8.Укажите, в каком предложении на месте всех пропусков ставятся запятые.

1)  Ужинали _ не спеша и почти_ молча.

2)  Несмотря на_ плохую погоду_ соревнования состоялись.

3)  Он обернулся_ и_не поднимая головы_ и _ не показывая ей лица_ушёл.

4)  Заметив людей_конь сердито фыркнул и пустился прочь_ломая кусты.

9. Укажите предложение, в котором ставится одна запятая. (Знаки препинания не расставлены.)

1)  На севере слабо мерцая подымались какие-то белые облака.

2)  Он всегда краснел вспоминая эту историю.

3)  Воробей заметив кота мгновенно взлетел вверх.

4)  Часто он читал лёжа.

10. В каком варианте ответа правильно указаны все цифры, на месте которых в пред­ложении должны стоять знаки препинания?

Оба неразлучные друга (1) глухой и слепой (2) приносили каждую весну глухарей (3) много больше (4) чем все обыкновенные охотники.

1)  1,2,3,4 2) 1,2,3 3) 2,3,4 4) 2,4

11. В каком варианте ответа правильно указаны все цифры, на месте которых в пред­ложении должны стоять знаки препинания?

Много есть на свете (1) кроме нашей страны (2) всяких государств и земель (3) но одна у человека мать (4) одна у него и Родина.

1)  1,2,3,4 2) 3, 4 3) 1,3,4 4) 2, 3, 4

12. В каком варианте ответа правильно указаны все цифры, на месте которых в пред­ложении должны стоять знаки препинания?

Слепой музыкант (1) как это постоянно бывает со слепыми (2) обладал тонким слухом (3) а глухой повар (4) замечательным зрением.

1)  1,2,3,4 2) 1,3,4 3) 1,2,3 4) 3,4

13. На месте каких цифр в предложении нужно поставить скобки?

Дикий барин (1) так его прозвали (2) настоящее же его имя было Перевлесов (3) пользовался огромным влиянием (4) во всей округе.

1)  1,2 2) 2,4 3) 1,3 4) 2,3

14. На месте каких цифр в предложении нужно поставить тире?

Пригибаясь (1) Юргин бросился к своему окошку (2) где у него (3) он помнил (4) лежала связка гранат.

1)  3,4 2) 1,2 3) 2,3 4) 2,4

15. Укажите, в каком предложении пропущена запятая.

1)  Храбрый от природы человек, он не любил никаких рассуждений на эту тему.

2) Роща на противоположном берегу реки, ещё час назад освещённая майским солнцем вдруг потемнела.

3)  Тяжелыми шагами ходили по паркету люди, выносившие вещи.

4)  Сорванная с деревьев листва закружилась в воздухе.

16. Укажите, на месте каких цифр в предложении должны быть запятые.

Это была икона (1) вывезенная из Смоленска (2) и с того времени возимая (3) за армией.

1)  1 2) 1,2 3) 1,3 4) 2,3

17. Укажите предложение, в котором на месте всех пропусков ставятся запятые.

1)  Изумлённый и растерянный_он не сразу нашёл_подходящий_ ответ.

2)  Вдоль железной дороги простирались_ невспаханные поля_ и нескошенные луга.

3)  Над рекой повис густой туман_белый_страшный.

4)  Простой народ здесь носит_ плетённые из лёгкого тростника _шляпы.

18. Укажите, на месте каких цифр должны быть запятые в предложении.

С необычайной силой и убедительностью художник умел передать ярость (1) грозно бушующего (2) моря (3) и блеск солнечных лучей (4) сверкающих в белоснежной пене (5) волн.

1)  1,3 2) 1,3,4 3) 3, 4, 5 4) 4

19. Укажите, на месте каких цифр в предложении должны стоять запятые.

Раскрыв (1) рот от восхищения (2) забыв (3) страх и одиночество (4) Павлик смотрел кругом (5) машинально отвечая на вопросы зоолога.

1)  2,4,5 2) 1,3,4 3) 3,4,5 4) 2,3,4,5

2. Домашнее задание.

А) Актуализация домашнего задания.

Прочитайте размышления о русском языке и его судьбе, принадлежащие замечательным русским писателям XX века. СЛАЙД

Русский народ за свою историю отобрал, сохранил, возвёл в степень уважения такие человеческие качества, которые не подлежат пересмотру: честность, трудолюбие, совестливость, доброту… Мы из всех исторических катастроф вынесли и сохранили в чистоте великий русский язык, он передан гам нашими дедами и отцами… (В. М. Шукшин).

Язык открывает свои чертоги и кладовые только людям с раскрытой душой, тем, у кого сердце не зачерствело в однообразии покоя и борьбы, кто искренен не только с другими, но и с самим собою… (В. Белов).

Согласны ли вы с писателями?

Выпишите наиболее актуальные проблемы, связанные с современным состоянием русского языка, Почему вы так читаете?

Домашнее задание.

Прочитайте текст и выполните задания к нему.

(1)Все метафоры, применимые к языку, верны по-сво­ему. (2)Язык — это тюрьма, из которой нам никогда не уда­стся сбежать. (З)Язык — это очки, без которых нам не раз­глядеть окружающий мир. (4)Язык — слуга и господин. (5)Язык — наш друг и враг одновременно. (б)Любой язык изменяется под влиянием различных факторов: внешних или внутренних. (7)Он как будто бы следит за нами и фик­сирует все самые важные наши проблемы и больные ме­ста. (8) Он не даёт ни соврать, ни обмануть самих себя. (9) О состоянии здоровья русского народа довольно красноре­чиво говорит статистика, утверждающая, что смертность русского населения России намного выше рождаемости, а это весьма тревожный сигнал, информирующий о вы­рождении нации, и об этом следует срочно звонить во все колокола.

(10)В полном соответствии со здоровьем народа находится состояние русского языка, терпящего сокрушительное фиаско, усугубляющееся резкой деформацией политической и экономической систем в России.

(11)На гребне реформаторской волны возникла как бы новая общность людей, не обременённых высоким обра­зованием, но с преувеличенными претензиями на веду­щую роль в бизнесе, средствах массовой информации, очагах культуры и т. д.

(12)Они создали свои нравы, свой язык, копирующий «блатную феню». (13)Заявив о себе русским фольклором в форме анекдотов о делах своих и помышлениях, − назва­ли себя «новыми русскими».

(14) На сочной ниве их антирусской лексики ярким бу­кетом расцвели слова, не отмеченные академическими словарями классического русского языка, а именно: не­пруха, неудобняк, нал, бабки, кидало, наезд, прикол, про­кол, по барабану и несть им числа.

(15)Язык «новых русских» проник в жизнеутвержда­ющие речи депутатов Госдумы, достиг эстрады и поразил кинематограф.

(16)Австралийцы, с большой любовью изучающие рус­ский язык и культуру русскую, не могут взять в толк, напри­мер, какое лексическое значение имеет слово «блин» в филь­ме «Особенности национальной охоты», так как оно грамма­тически не связано в фразе, произнесённой героем фильма. (17)Это типичное слово-паразит от «новых русских»; имея примитивно-ругательный смысл, оно жирной кляк­сой ложится на прекрасный русский язык.

(18)Чтобы хоть как-то обогатить свою скудную речь, «новые русские» вкрапляют в неё слова иностранного происхождения (чаще английские), к примеру: эксклюзив, бартер, брокер и уйму других. (19) Слова эти на фоне са­мобытного и богатого фонетически русского языка вы­глядят, извините, как на корове седло.

(20)Увы, мы никогда не будем говорить на языке Турге­нева. (21 ) Что Тургенева — на языке Пришвина и Паустов­ского никогда мы уже не будем говорить вместе с нашими детьми! (22)Так что живём мы сейчас в стрессовых усло­виях языкового разрыва поколений и многоязычия, и един­ственное, что можно посоветовать соотечественникам, так это терпения и терпимости. (23)Ещё через десяток-другой лет наступит период стабилизации, и мы наконец без вся­ких законов обретём наш единый общий русский язык, без которого невозможна русская культура.

(По Е. Чепурову)

1. Определите тип и стиль речи текста.

1)  описание, художественный

2)  повествование, публицистический

3)  рассуждение, публицистический

4)  рассуждение, научно-популярный

2. Укажите предложения, в которых сформулирована позиция автора.

1)  1,9, 13, 18

2)  6, 10, 22, 23

3)  2, 8, 14, 20

4)  3, И, 15, 19

3. Укажите средства выразительности, которые не используют­ся в предложениях 14-19 для передачи особенностей совре­менной языковой ситуации.

1)  эпитет

2)  метафора

3)  сравнение

4)  риторическое восклицание

4. Укажите предложения, в которых содержатся аргументы к тезису, сформулированному в предложении 15.

1)  12-14

2)  13

3)  16-17

4)  18-19

5. Среди предложений 15-21 найдите предложение, которое связано с предыдущим с помощью лексического повтора и указательного местоимения. Запишите его номер.

6. Напишите по прочитанному тексту сочинение-рассуждение.

Методические рекомендации.

Урок проводится на 1 курсе СПО на этапе систематического повторения изученного за курс средней школы. Повторение разделов «Синтаксис и пунктуация» считаю целесообразным проводить в несколько этапов:

  • Синтаксис и пунктуация простого предложения (осложнённого).

  • Синтаксис и пунктуация сложного предложения.

  • Сложное синтаксическое целое.

Данный урок позволяет привести в систему знания по указанной теме, полученные в школьном курсе изучения русского языка, использовать их при решении практических лингвистических задач, в том числе, при подготовке к ЕГЭ.

Организация повторения на уроке организована с использованием беседы о языке, построенной на основе высказывания писателя В. Белова, что даёт возможность учащимся эмоционально настроиться на предмет последующего разговора и самим сформулировать коммуникативную тему урока, целевые установки. Повторение орфоэпических и грамматических норм проводится в форме групповой работы, что позволяет в ходе обсуждения в группах вспомнить нормы, найти правильное решение, учит деловому общению, а также способствует здоровьесбережению (снимается напряжение, вызванное индивидуальной и фронтальной формой работы). В качестве заданий предложены:

  • работа над орфоэпическими нормами (произнести указанные слова);

  • работа над грамматическими нормами (согласовать формы подлежащего и сказуемого, исправить ошибки в словосочетаниях, в употреблении падежных форм и предлогов, деепричастных оборотов);

  • работа над стилистическими нормами в предложениях с однородными членами.

Дидактический материал взят из домашних заданий, творческих работ учащихся, поэтому составляет элемент работы над ошибками учащихся. При выполнении подобных заданий учащиеся могут обращаться к таблице «Речевые и грамматические ошибки», таким образом, закрепляется умение квалифицировать ошибки, что способствует предупреждению и развитию навыка исправления подобных ошибок. Содержание повторения является также элементом подготовки к ЕГЭ, так как задания сформулированы аналогично заданиям А4, А5. Для проверки правильности после выполнения заданий учащимся предлагается правильный ответ на слайде презентации.

Материал домашнего задания к данному уроку (беседа об актуальности проблем каждого из текстов о языке) также способствует подготовке к ЕГЭ. Учащиеся строят свой ответ как рассуждение (называют тезис, приводят доказательства своей позиции, делают вывод). Высказывания о русском языке учащиеся могут использовать при формулировании проблемы, комментария или подборе аргументов, доказательств по теме «Язык» при работе над текстом в части С.

Повторяются также типы и стили речи.

Материал домашнего задания используется для перехода к лингвистической теме урока: перестроив высказывание В. Шукшина дважды, обращаемся к теме «Простое осложнённое предложение. Синтаксис и пунктуация».

Составление опорной схемы «Простое осложнённое предложение» позволяет актуализировать знания учащихся.

Повторение сведений и отработка пунктуационных навыков осуществляются с использованием мультимедийной презентации, в которую включены сведения об осложнённом предложении и задания различных типов:

  • Однородные члены предложения и пунктуация при них (задания: составить схемы предложений).

  • Сравнительный оборот и знаки препинания при сравнительных оборотах (тестовые задания типа А 21).

  • Обособленные члены предложения (задания: конструирование предложений, тест типа А 19, А22).

  • Уточняющие члены предложения (задания: расставить знаки препинания в предложении с уточняющими членами, сконструировать предложения с данными уточняющими членами).

  • Вводные слова и вводные предложения (задание на разграничение вводных слов и синонимичных им членов предложения).

Таким образом, задания предусматривают различные формы работы на уроке, что позволяет эффективно отработать пунктуационные, речевые навыки, навык работы с тестовыми заданиями, в том числе конкретными заданиями ЕГЭ, сохранить внимание учащихся, избежать утомляемости на уроке. Задания различны по степени сложности, что позволяет учащимся оценить свои силы и сделать выводы о необходимости корректировки собственных знаний. В ходе урока по мере выполнения заданий учащиеся отмечают успешность и трудности при выполнении заданий в листке учёта знаний (это могут быть различные отметки: плюсы или минусы, баллы по определённой шкале). Рефлексия на уроке способствует самооценке знаний учащегося. Обучающимся необходимо развивать навыки самоконтроля. Считаю целесообразным после проверки итогового теста урока соотнести результаты оценки учителя и самооценки обучающегося, что продемонстрирует объективную картину знаний и умений обучающегося.

После систематизации и повторения проводится текущий контроль — тест по теме с включением заданий, аналогичных заданиям урока и заданиям ЕГЭ. Эта работа предваряется комментарием учителя о необходимости правильно рассчитать время работы: на каждый тест отводится 1-2 минуты (рекомендации ФИПИ по распределению времени на часть А ЕГЭ — 1-3 мин).

Актуализация домашнего задания проводится на материале высказываний русских писателей XX века. Вопросы учителя возвращают учащихся к разговору о проблемах языка, затронутых в начале урока. Учащиеся формулируют актуальные проблемы современного языка, что позволяет подготовить их к восприятию домашнего задания: работа по тексту о языке, написание сочинения — рассуждения в соответствии с критериями ЕГЭ. Содержание задания предполагает использование при его выполнении материала урока (лингвистические и коммуникативные знания, умения и навыки).

Итак, данный урок может использоваться на 1 курсе с целью повышения пунктуационной грамотности обучающихся, а также для подготовки учащихся к итоговой аттестации в форме ЕГЭ.

1

Меркушенкова Людмила Александровна Учитель русского языка и литературы МБОУ «Лицей 9 имени К.Э. Циолковского» города Калуги

4

Нормы – принятые в обществсенно- языковой практике образованных людей правила употребления слов, форм слов и предложений. Норма – единствсенно возможный или предпочтительный вариант правильного, образцового употребления слова, формы, конструкции.

5

НОРМЫ Орфоэпи- ческие Лексические Стилисти- ческие Граммати- ческие Словообразо- вательные Морфологи- ческие Синтакси- ческие Пунктуаци- онные Орфогра фи- ческие Правопи- сания

6

Нормы произношения ши[н’]ель, а не ши[на]ль Нормы ударения дос Уг, а не д Осуг Интонационные нормы Нормы использования интонации Орфоэпические нормы устной речи изучает орфоэпия. Звуковые средства языка изучает фонетика.

7

1)ненадолго 2)проживший 3)прожитый 4)собралась отвсет: 2) проживший В каком словсе ударение падает на второй слог?

8

1)пространство конечно 2)это, конечно, шутка 3)полуночный час 4)послать письмо с нарочным отвсет: 2) это, коне [шон] о, шутка В каком примере в выделенном словсе произносится [шон]?

9

1)проект 2)проекция 3)кариес 4)всеер отвсет: 4) все[jь]р В каком словсе есть звук [j]?

10

1)побл..кла 2)оп..ка 3)быт и.. 4)а ф..ра отвсет: 1) поблёкла В каком словсе на месте пропуска пишется Ё?

12

Речевые ошибки Вид ошибок Примеры

13

Речевые ошибки Вид ошибок Примеры 1Мы были шокированы прекрасной игрой актеров

14

Речевые ошибки Вид ошибок Примеры 1Мы были шокированы прекрасной игрой актеров

15

Речевые ошибки Вид ошибок Примеры 1Употребление слова в несвойствсенном ему значении Мы были шокированы прекрасной игрой актеров

16

Речевые ошибки Вид ошибок Примеры 1Употребление слова в несвойствсенном ему значении Мы были шокированы прекрасной игрой актеров. 2Были приняты эффектные меры

17

Речевые ошибки Вид ошибок Примеры 1Употребление слова в несвойствсенном ему значении Мы были шокированы прекрасной игрой актеров. 2Были приняты эффектные меры

18

Речевые ошибки Вид ошибок Примеры 1Употребление слова в несвойствсенном ему значении Мы были шокированы прекрасной игрой актеров. 2Неразличение оттенков значения, вносимых в слово приставкой и суффиксом Были приняты эффектные меры

19

Речевые ошибки Вид ошибок Примеры 1Употребление слова в несвойствсенном ему значении Мы были шокированы прекрасной игрой актеров. 2Неразличение оттенков значения, вносимых в слово приставкой и суффиксом Были приняты эффектные меры. 3В конечном предложении автор применяет градацию

20

Речевые ошибки Вид ошибок Примеры 1Употребление слова в несвойствсенном ему значении Мы были шокированы прекрасной игрой актеров. 2Неразличение оттенков значения, вносимых в слово приставкой и суффиксом Были приняты эффектные меры. 3В конечном предложении автор применяет градацию

21

Речевые ошибки Вид ошибок Примеры 1Употребление слова в несвойствсенном ему значении Мы были шокированы прекрасной игрой актеров. 2Неразличение оттенков значения, вносимых в слово приставкой и суффиксом Были приняты эффектные меры. 3Неразличение синонимичных словВ конечном предложении автор применяет градацию

22

Речевые ошибки Вид ошибок Примеры 1Употребление слова в несвойствсенном ему значении Мы были шокированы прекрасной игрой актеров. 2Неразличение оттенков значения, вносимых в слово приставкой и суффиксом Были приняты эффектные меры. 3Неразличение синонимичных словВ конечном предложении автор применяет градацию. 4Автор увселичивает впечатление

23

Речевые ошибки Вид ошибок Примеры 1Употребление слова в несвойствсенном ему значении Мы были шокированы прекрасной игрой актеров. 2Неразличение оттенков значения, вносимых в слово приставкой и суффиксом Были приняты эффектные меры. 3Неразличение синонимичных словВ конечном предложении автор применяет градацию. 4Автор увселичивает впечатление

24

Речевые ошибки Вид ошибок Примеры 1Употребление слова в несвойствсенном ему значении Мы были шокированы прекрасной игрой актеров. 2Неразличение оттенков значения, вносимых в слово приставкой и суффиксом Были приняты эффектные меры. 3Неразличение синонимичных словВ конечном предложении автор применяет градацию. 4Нарушение лексической сочетаемости Автор увселичивает впечатление

25

Речевые ошибки Вид ошибок Примеры 1Употребление слова в несвойствсенном ему значении Мы были шокированы прекрасной игрой актеров. 2Неразличение оттенков значения, вносимых в слово приставкой и суффиксом Были приняты эффектные меры. 3Неразличение синонимичных словВ конечном предложении автор применяет градацию. 4Нарушение лексической сочетаемости Автор увселичивает впечатление. 5Молодой юноша, очень прекрасный. 6 7

26

Речевые ошибки Вид ошибок Примеры 1Употребление слова в несвойствсенном ему значении Мы были шокированы прекрасной игрой актеров. 2Неразличение оттенков значения, вносимых в слово приставкой и суффиксом Были приняты эффектные меры. 3Неразличение синонимичных словВ конечном предложении автор применяет градацию. 4Нарушение лексической сочетаемости Автор увселичивает впечатление. 5Молодой юноша, очень прекрасный. 6 7

27

Речевые ошибки Вид ошибок Примеры 1Употребление слова в несвойствсенном ему значении Мы были шокированы прекрасной игрой актеров. 2Неразличение оттенков значения, вносимых в слово приставкой и суффиксом Были приняты эффектные меры. 3Неразличение синонимичных словВ конечном предложении автор применяет градацию. 4Нарушение лексической сочетаемости Автор увселичивает впечатление. 5Употребление лишоних слов, в том числе плеоназм Молодой юноша, очень прекрасный. 6 7

28

Речевые ошибки Вид ошибок Примеры 1Употребление слова в несвойствсенном ему значении Мы были шокированы прекрасной игрой актеров. 2Неразличение оттенков значения, вносимых в слово приставкой и суффиксом Были приняты эффектные меры. 3Неразличение синонимичных словВ конечном предложении автор применяет градацию. 4Нарушение лексической сочетаемости Автор увселичивает впечатление. 5Употребление лишоних слов, в том числе плеоназм Молодой юноша, очень прекрасный. 6В этом рассказе рассказывается о реальных событ иях. 7

29

Речевые ошибки Вид ошибок Примеры 1Употребление слова в несвойствсенном ему значении Мы были шокированы прекрасной игрой актеров. 2Неразличение оттенков значения, вносимых в слово приставкой и суффиксом Были приняты эффектные меры. 3Неразличение синонимичных словВ конечном предложении автор применяет градацию. 4Нарушение лексической сочетаемости Автор увселичивает впечатление. 5Употребление лишоних слов, в том числе плеоназм Молодой юноша, очень прекрасный. 6В этом рассказе рассказывается о реальных событ иях. 7

30

Речевые ошибки Вид ошибок Примеры 1Употребление слова в несвойствсенном ему значении Мы были шокированы прекрасной игрой актеров. 2Неразличение оттенков значения, вносимых в слово приставкой и суффиксом Были приняты эффектные меры. 3Неразличение синонимичных словВ конечном предложении автор применяет градацию. 4Нарушение лексической сочетаемости Автор увселичивает впечатление. 5Употребление лишоних слов, в том числе плеоназм Молодой юноша, очень прекрасный. 6Употребление рядом или близко однокоренных слов (тавтология) В этом рассказе рассказывается о реальных событ иях. 7

31

Речевые ошибки Вид ошибок Примеры 1Употребление слова в несвойствсенном ему значении Мы были шокированы прекрасной игрой актеров. 2Неразличение оттенков значения, вносимых в слово приставкой и суффиксом Были приняты эффектные меры. 3Неразличение синонимичных словВ конечном предложении автор применяет градацию. 4Нарушение лексической сочетаемости Автор увселичивает впечатление. 5Употребление лишоних слов, в том числе плеоназм Молодой юноша, очень прекрасный. 6Употребление рядом или близко однокоренных слов (тавтология) В этом рассказе рассказывается о реальных событ иях. 7Герой рассказа не задумывается над своим поступком. Герой даже не понимает всей глубины содеянного.

32

Речевые ошибки Вид ошибок Примеры 1Употребление слова в несвойствсенном ему значении Мы были шокированы прекрасной игрой актеров. 2Неразличение оттенков значения, вносимых в слово приставкой и суффиксом Были приняты эффектные меры. 3Неразличение синонимичных словВ конечном предложении автор применяет градацию. 4Нарушение лексической сочетаемости Автор увселичивает впечатление. 5Употребление лишоних слов, в том числе плеоназм Молодой юноша, очень прекрасный. 6Употребление рядом или близко однокоренных слов (тавтология) В этом рассказе рассказывается о реальных событ иях. 7Герой рассказа не задумывается над своим поступком. Герой даже не понимает всей глубины содеянного.

33

Речевые ошибки Вид ошибок Примеры 1Употребление слова в несвойствсенном ему значении Мы были шокированы прекрасной игрой актеров. 2Неразличение оттенков значения, вносимых в слово приставкой и суффиксом Были приняты эффектные меры. 3Неразличение синонимичных словВ конечном предложении автор применяет градацию. 4Нарушение лексической сочетаемости Автор увселичивает впечатление. 5Употребление лишоних слов, в том числе плеоназм Молодой юноша, очень прекрасный. 6Употребление рядом или близко однокоренных слов (тавтология) В этом рассказе рассказывается о реальных событ иях. 7Неоправданное повторение слова Герой рассказа не задумывается над своим поступком. Герой даже не понимает всей глубины содеянного.

34

1)АФОРИЗМ – краткое выразительное изречение 2)ДОБЛЕСТЬ – мужество, отвага, самоотвсерженность 3)СКУДНЫЙ – недостаточный по количеству, бедный 4)ДЕКЛАРИРОВАТЬ – давать обещание отвсет: декларировать – выступать с программным заявлением, провозглашать что-либо Значение какого слова определено невсерно?

35

1. В парке было заложено тридцать два дерева. 2. Бабушка ласково кивает добрым морщинистым лицом. 3. Оратор говорил очень конспективно. 4. В поступившей в библиотеку партии книг оказалось несколько дефективных экземпляров. 5. Результаты следствия ясны всем. Осталось резюмировать приговор. Найдите случаи нарушения лексических норм, связанных с употреблением слов в несвойствсенных им значениях. Исправьте ошибки.

36

Молодые люди удобно расположились на сиденьях и с показным пренебрежением посматривают на стоящих вокруг. 1)наглядным 2)показательным 3)демонстрационным 4)демонстративным Какое слово может быть использовано вместо выделенного в предложении?

37

1)Ежедневные прогулки по лесу сделались для нас потребностью. 2)Жизнь спортсмена – это прежде всего упорные, ежедневные тренировки. 3)Модельеры не очень любят работать над ежедневной одеждой, их больше интересуют всечерние туалеты. 4)В каждом большом городе издаются свои ежедневные газеты. В каком предложении вместо прилагательного ЕЖЕДНЕВНЫЙ нужно употребить ПОВСЕДНЕВНЫЙ?

38

1)Ежедневные прогулки по лесу сделались для нас потребностью. 2)Жизнь спортсмена – это прежде всего упорные, ежедневные тренировки. 3)Модельеры не очень любят работать над ежедневной одеждой, их больше интересуют всечерние туалеты. 4)В каждом большом городе издаются свои ежедневные газеты. В каком предложении вместо прилагательного ЕЖЕДНЕВНЫЙ нужно употребить ПОВСЕДНЕВНЫЙ?

39

1)превосходить 2)преобладать 3)доминировать 4)превалировать Какое слово лишонее в ряду синонимов?

40

1)Существсенный признак чего-либо 2)Обсуждение чего-либо с целью дать оценку, выявить достоинства 3)Принципы, убеждения какого-либо человсека 4)Основание для оценки Какое слово имеет значение КРИТЕРИЙ?

41

1)Существсенный признак чего-либо 2)Обсуждение чего-либо с целью дать оценку, выявить достоинства 3)Принципы, убеждения какого-либо человсека 4)Основание для оценки Какое слово имеет значение КРИТЕРИЙ?

42

1)дестабилизирующий 2)деградирующий 3)реакционный 4)резонёрствующий Какое слово имеет значение «приходящий в упадок»?

43

1)знаток 2)эксперт 3)профессионал 4)любитель Какое слово имеет значение, противоположное слову ДИЛЕТАНТ?

44

Пыль наводнила воздух, уделить серьезное значение, тяжелый поступок, представить слово для доклада, ухудшение уровня жизни, повысить кругозор. Исправьте ошибки, связанные с нарушением лексической сочетаемости

45

Практическая работа по теме «Лексическая норма» Задание 1. Определите значение следующих слов. Составьте словосочетания. Абонемент, абонент, абстракция, аксиома, аллегория, альманах, амплуа, апогей, апостроф, апофеоз, ассоциация, баллотироваться, беллетристика, вираж, виртуоз, витраж, габарит, гамма, гарантия, гипотеза, глашатай, гравсер, гравюра. Задание 2. Правильно ли употреблены слова иноязычного происхождения в привседенных ниже предложениях? Исправьте предложения, в которых допущены ошибки. 1. Для свободных игр выносим на площадку много различных а т р и б у т о в: вожжи, санки, шапочки. 2. Обилие а к с е с с у а р о в отягощает сюжет, отвлекая внимание от главного. 3. В классе нерегулярно в е н т и л и р у ю т помещение. 4. Выступающий о п е р и р у е т положительными примерами из жизни группы. 5. Важным ф а к т о р о м хорошего качества знаний являются систематические знания. Задание 3. Устраните ошибки, связанные с употреблением слов без учета их семантики в контексте и с неточным словоупотреблением. 1. Один поступок Чацкого привсел меня в неясность. 2. Сейчас многие писатели пристально занимаются политикой. 3. Ученики пристально слушали выступление артиста. 4. Но перед тем, как использовать материал и легко вибрировать им, я хочу изложить свои мысли о Базаровсе. 5. Идея этого произвседения заключается в призывсе к русским князьям воплотиться в единое княжество и встать на защиту Русской земли. 6. Все это и рисует представление о «Словсе о полку Игоревсе». 7. Зарисовки также помогают читателю лучше понять текст. Это своенравное объяснение. 8. Ученики сами могут поставить вопросы выступающему. 9. «Гранатовый браслет» — одно из самых подтвсерждающих произвседений Куприна.

47

ЛЕКСИКА РУССКОГО ЯЗЫКА Стилистически нейтральные слова литературные книжные разговорные Стилистически окрашенные слова нелитературные просторечные диалектные жаргонные

48

Сгруппируйте привседенные слова по типу функционально- стилистической окраски (нейтр., книжн., разг.) Ахнуть, актуальный, анонс, бартер, взыскание, всещий, вкалывать, довсеренность, дефект, зачетка, иждивсение, забота, вакуоль, иск, книжка, косинус, квартиросъемщик, картошка, начертать, норма, нормальный, отлынивать, пневмония, работа, рот, трудиться, усыновитель, уста, эксперимент. Сгруппируйте привседенные слова по типу функционально- стилистической окраски (нейтр., книжн., разг.)

49

Богатей, горбач, промеж, смуглявый, супротивник, теперича, вдогон, спечь, убивсец, упредь, манатки, потра фить, сохатый, барахло, тутошоний, аккурат, небось, умаяться, сызмала, смотаться, шибко, пузо, брюхо, сдрейфить, зубодер, облапошить, расфуфыриться. Определите значение просторечных слов, замените их литературными

50

Работу начали с первыми кочетами. К всечеру ненадолго развседрилось. Ребята набрали целую корзину пуховок. Утирка осталась в кармане пальто. Отец с братьями охотились на всекшу. Найдите в предложениях диалектизмы, определите их значение. Укажите случаи неоправданного их употребления.

51

Высокие (торжествсенные) слова Сниженные слова Книжные слова литературного языка, употребляющиеся в публицистическом и художествсенных текстах, например: отечество, родина, дерзновсение, кончина Разговорные, просторечные и жаргонные слова, употребляющиеся в бытовой сфере общения и недопустимые в официальных ситуациях общения, например: старикашка, сыграть в ящик

52

Взывать, глазеть, головомойка, грядущий, дерзновсенный, легавый, мазила, мольба, нагоняй, начертать, лоботряс, почин, растянуться, содружество, соратник, чаять, нянчиться. Сгруппируйте данные ниже слова следующим образом: 1) высокие, торжествсенные слова; 2) просторечия.

55

Найдите ошибки, которые появились в связи с неправильным образованием слов. Исправьте предложения. 1. Змеи относятся к животным класса пресмыкающих. 2. При гололеде будьте внимательны на дорогах, чтобы не подскользнуться и не упасть прямо на проезжей части. 3. Эта история может стать самым смотрибельным сериалом. 4. У него был вспыхчивый характер. 5. Его предсмертельные слова произвсели большое впечатление. 6. Значение этого английского слова необходимо провсерить по двухязычному словарю.

58

В следующих парах слов найдите формы, не соотвсетствующие литературной норме. Клавиш – клавиша, ставсень – ставня, плацкарт – плацкарта, спазм – спазма, зал – зала, туфля – туфель, бронх – бронха, банкнот – банкнота, жира ф – жира фа, ката фалк – ката фалка, бакенбард — бакенбарда, ласт – ласта, скирд – скирда, рельс – рельса.

59

Найдите все неправильные формы степеней сравнения имен прилагательных. Более шире – более широко – шире – ширее – ширше. Кратчайший – самый кратчайший – самый короткий. Худший – самый худший. Лучший – самый хороший – очень хороший — лучше всех.

60

Исправьте ошибки в предложениях. 1. Главная бухгалтерия обслуживает сегодня тридцать два яслей. 2. Двадцать три суток прошло с момента отправки очередной научной экспедиции на станцию «Мир». 3. Трое бракованных плоскогубцев были проданы по сниженной цене. 4. За день торговый центр продал тридцать трое брюк. 5. В гонках собачьих упряжек участвовало двадцать четыре саней.

63

Беспокоиться о ком Идентичный чему Надеть что на что Обращать внимание на что Отзыв о чем Превосходство над чем Предостеречь от чего Препятствовать чему Увсеренность в чем Уплатить за что Тревожиться за кого Сходный с чем Одеть кого во что Уделять внимание чему Рецензия на что Преимущество перед чем Предупредить о чем Тормозить что Вера во что Оплатить что

64

Найдите ошибки в употреблении следующих словосочетаний. Исправьте их. 1. Отчитаться в израсходовании полученного аванса. 2. Отзыв на статью. 3. Уделять внимание на каждую мелочь. 4. Превосходство новой техники перед старой. 5. Примириться перед неумолимой судьбой. 6. Рассержен за выходку приятеля. 7. Увсеренность в торжество справседливости.

65

Наблюдая за прыжком Павла Ольшанского, у вас может возникнуть вопрос… Жаль, что у меня нет просто возможности было полететь туда. Мы должны действовать вопреки тому, что действуют со стороны НАТО. Никому не секрет, что главные виновники в развале хозяйства – чиновники. Время слушать классику. На сей раз вашему вниманию ария тореадора из оперы «Кармен» Жоржа Бизе. Мы объявили о подорожании цен на метро. Россия будет совсершать рыночные, тем не менее, преобразования. Найдите случаи нарушения синтаксических норм. В чем, на ваш взгляд, ошибка? Обратите внимание на предложения, в которых нарушены лексические нормы современного русского языка.

66

Виды ошибок Примеры 1. Ошибочное словообразование Трудолюбимый, надсмехаться 2 Ошибочное образование формы существительного Многие чуда техники, не хватает время 3 Ошибочное образование формы прилагательного Более интереснее, красивше 4 Ошибочное образование формы числительного С пятистами рублями 5 Ошибочное образование формы местоимения Ихнего па фоса, ихи дети 6 Ошибочное образование формы глагола Они ездиют, хочут 7Нарушение согласованияЯ знаком с группой ребят, серьезно увлекающимися джазом 8Нарушение управления Нужно сделать свою природу более красивую. 9Нарушение связи между подлежащим и сказуемым Большинство возражали против такой оценки его творчества. 10Ошибки в построении предложения с однородными членами предложения Страна любила и гордилась поэтом. 11Ошибки в построении предложения с причастным оборотом Узкая дорожка была покрыта проваливающимся снегом под ногами. 12Ошибки в построении предложения с деепричастным оборотом Читая текст, возникает такое чувство… 13Ошибки в построении сложного предложения Эта книга научила меня ценить и уважать друзей, которую я прочитал ещё в детствсе. 14Смешение прямой и косвсенной речи Автор сказал, что я не согласен с мнением рецензента. 15Нарушение границ предложения Когда герой опомнился. Было уже поздно. 16Нарушение видовременной соотнесенности глагольных форм Замирает не мгновсение сердце и вдруг застучит вновь. Грамматические ошибки

67

Правила расстановки знаков препинания Правила написания слов

68

«Учиться хорошей, спокойной, интеллигентной речи надо долго и внимательно — прислушиваясь, запоминая, замечая, читая и изучая. Но хоть и трудно — это надо, надо. Наша речь — важнейшая часть не только нашего повседения, но и нашей личности, наших души, ума, нашей способности не поддаваться влияниям среды, если она «затягивает». Д.С. Лихачев

«Синтаксис и пунктуация простого предложения» в 11 классе.

I.  Повторение.

Эпиграф.

Язык открывает свои чертоги
и кладовые только людям с раскрытой душой, тем, у кого сердце не зачерствело в однообразии покоя и борьбы, кто искренен не только с другими,
но и самим собою
…В. И. Белов.

Вопрос.

Как вы думаете, что будет темой нашего сегодняшнего разговора?

(-русский язык).

Послушайте диалог, записанный на улице.

На какие невесёлые размышления наводит разговор на улице?

Допустимо ли употребление жаргонизмов в литературной речи? Почему?

(-затрудняет понимание и ограничивает возможности языка).

Соблюдение норм литературного языка является необходимым условием его сохранения и развития.

Задание № 1.Работа над орфоэпическими нормами.

Произнесите следующие слова:

Группа № 1.

Афера, затекший, опека, оседлый, маневры.

Группа № 2.

Новорожденный, острие, платежеспособный, дареный, одноименный.

Группа № 3.

Свекла, многоженство, блеклый, никчемный, безнадежный.

Группа № 4.

Белесый, заем, двоеженец, договоренность, истекший.

Группа № 5.

Житие, забредший, испекший, щелка, бытие.

Ответы:

Афера, затекший, опека, оседлый, маневры.

Новорождённый, остриё, платёжеспособный, дарёный, одноимённый.

Свёкла, многожёнство, блёклый, никчемный, безнадёжный.

Белёсый, заём, двоежёнец, договорённость, истёкший.

Житие, забредший, испекший, щёлка, бытие.

Задание № 2. Работа над грамматическими нормами.

А) Допишите окончания, согласуя сказуемое с подлежащим.

Группа № 1.

1. Большинство картин художников — передвижников был.. воспринят.. как гимн человеку, прославляющий и утверждающий его высокие духовные ценности.

Группа № 2.

2.  Часть студентов уже сдал.. экзамены по специальности.

Группа № 3.

3.  Двадцать человек стоял.. в стороне.

Группа № 4.

4.  Множество темноватых туч расползал..сь по небу.

Группа № 5.

5.  Десять студентов окончил.. институт с отличием.

Ответ.

1. Большинство картин художников — передвижников было воспринято как гимн человеку, прославляющий и утверждающий его высокие духовные ценности.

Часть студентов уже сдала экзамены по специальности. Двадцать человек стояли в стороне. Множество темноватых туч расползалось по небу. Десять студентов окончили институт с отличием.

Б) Исправьте ошибки в употреблении словосочетаний.

Группа № 1.

1. Отчитаться в получении полученного аванса.

Группа № 2.

2. Отзыв на статью.

Группа № 3.

3. Уделять внимание на каждую мелочь.

Группа № 4.

4. Превосходство новой техники перед старой.

Группа № 5.

5. Примириться пред неумолимой судьбой.

Ответ.

1. Отчитаться (о чём?) о получении полученного аванса.

2. Отзыв (о чём) о статье.

3. Уделять внимание (чему?) каждой мелочи.

4. Превосходство новой техники (над чем?) над старой.

5. Примириться (с чем?) с неумолимой судьбой.

В) Исправьте ошибки в употреблении падежных форм и предлогов.

Группа № 1.

Об этом мы познакомим вас позже.

Группа № 2.

Докладчик подчеркнул о необходимости строительства.

Группа № 3.

Решительность парламента свелась лишь в предоставлении полномочий.

Группа № 4.

Вчера брат приехал с Москвы.

Группа № 5.

По окончанию переговоров представители сторон подписали совместное заявление.

Ответ.

С этим мы познакомим вас позже. Докладчик подчеркнул необходимость строительства. Решительность парламента свелась лишь к предоставлению полномочий. Вчера брат приехал из Москвы. По окончании переговоров представители сторон подписали совместное заявление.

Г) Исправьте ошибки в употреблении деепричастного оборота.

Группа № 1.

Объяснение этих явлений может быть найдено, взяв в качестве иллюстрации последние события.

Группа № 2.

Проработав всего два месяца, у него возникли осложнения с начальником цеха.

Группа № 3.

Приняв смену, дежурные водители направляются диспетчером по объектам.

Группа № 4.

Торговый зал был очищен от людей, опасаясь, что рухнет потолок.

Группа № 5.

Изучая процесс разгосударствления, обнаруживается определённая закономерность.

Ответ.

Объяснение этих явлений можно найти, взяв в качестве иллюстрации последние события. После двух месяцев работы у него возникли осложнения с начальником цеха. Приняв смену, диспетчер направляет дежурных водителей по объектам. Торговый зал был очищен от людей из-за опасений, что рухнет потолок. Изучая процесс разгосударствления, можно обнаружить определённую закономерность.

Группа № 5.

Д) Внесите стилистическую правку в предложения с однородными членами.

Группа № 1.

Магазину требуются продавцы овощей и картофеля.

Группа № 2.

ООО Севастопольский приглашает на постоянную работу мастера по переработке овощей и одиноких рабочих (мужчин и женщин).

Группа № 3.

В работу нужно включить как можно больше людей и лошадей, которые управлялись бы с уборкой картофеля.

Группа № 4.

Рабочие завода поддерживают и голосуют за кандидатуры, выдвинутые участниками митинга.

Группа № 5.

Автобус имеет высокие подножки и низкие динамические показатели.

Ответ.

1. Магазину требуются продавцы овощей: картофеля и др.

ООО Севастопольский приглашает на постоянную работу мастера по переработке овощей и рабочих (мужчин и женщин). В работу нужно включить как можно больше людей, которые управлялись бы с уборкой картофеля; необходимо использовать лошадей для облегчения труда работников. Рабочие завода поддерживают кандидатуры, выдвинутые участниками митинга, и голосуют за них. Автобус имеет высокие подножки, у него низкие динамические показатели.

II. Проверка домашнего задания.

Прочитайте высказывания о русском языке, принадлежащие великим мастерам слова.

Какое высказывание кажется вам наиболее актуальным в наше время и почему?

Во дни сомнений, во дни тягостных раздумий о судьбах моей родины,- ты один мне поддержка и опора, о великий, могучий, правдивый и свободный русский язык! Не будь тебя — как не впасть в отчаяние при виде всего, что совершается дома? Но нельзя думать, чтобы такой язык не был дан великому народу! И. С. Тургенев.

Нет слова, которое было бы так замашисто, бойко, так вырывалось бы из — под самого сердца, так бы кипело и животрепало, как метко сказанное русское слово. Н. В. Гоголь.

Язык — это история народа. Язык — это путь цивилизации и культуры. Поэтому — то изучение и сбережение русского языка является не праздным занятием от нечего делать, но насущной необходимостью. А. И. Куприн.

Прочитайте микротекст и выполните задания к нему.

(1)  Русский язык (как любой другой национальный язык) представляет собой систему знаков, с помощью которой мы общаемся с представителями своего народа. (2) И, таким образом, он становится неотъемлемой частью нашего коллективного сознания, нашего менталитета. (3) Поэтому вместе с такими культурными элементами, как обычаи, нравы, национальное искусство, язык определяет нацию как выделенный среди иных народов объект.

Какую цель, по-вашему, ставит автор этого текста:

1)  описать картину, нарисовать образ,

2)  передать последовательность действий, событий,

3)  донести до читателя свои мысли, рассуждения,

4)  познакомить читателя с научными сведениями и сделать умозаключение.

Укажите тип речи.______________________________ Укажите стиль речи. _____________________________

Ответы.

1.4).

2. Рассуждение.

3. Научно — популярный.

II.  Систематизация изученного по теме «Синтаксис и пунктуация простого предложения».

1.  Конструирование предложений.

Синтаксический разбор предложения.

Мы из всех исторических катастроф вынесли и сохранили в чистоте великий русский язык, он передан нам нашими дедами и отцами. В. Шукшин.

Перестройте его дважды таким образом, чтобы изменилась его характеристика. Дайте общую характеристику получившимся предложениям.

Ответ.

А) Мы из всех исторических катастроф вынесли и сохранили в чистоте великий русский язык, переданный нам нашими дедами и отцами. (простое, осложнённое).

Б) Мы из всех исторических катастроф вынесли и сохранили в чистоте великий русский язык, который передан нам нашими дедами и отцами. (сложноподчинённое с придаточным определительным).

2.  Составление обобщающей схемы «Простое осложнённое предложение».

Вопрос.

Чем может быть осложнено простое предложение?

(- словами, грамматически связанными с членами предложения:

— однородными, обособленными и уточняющими членами;

— словами, грамматически не связанными с членами предложения: обращениями и междометиями, вводными словами и предложениями, вставными конструкциями)

3. Повторение сведений и выполнение тренировочных упражнений.

А) Однородные члены предложения.

Дайте определение однородных членов предложения. СЛАЙД

ü  Однородные членыэто ряд одинаковых членов предложения, соединённых между собой сочинительной связью, которая выражается союзами или только интонацион­но.

ü  Между однородными членами при отсутствии союза ставится запятая.

Вопрос.

Когда между однородными членами запятая:

— ставится,

— не ставится.

Ответ. СЛАЙД

Запятая ставится:

ü  перед противительными союзами (не марки, а открытки);

ü  перед повторяющимися союзами (и марки, и открытки);

ü  перед второй частью двойных союзов (как марки, так и открытки);

ü  перед союзом да и, обозначающим добавление к сказанному раньше.

Запятая не ставится:

ü  перед однородными членами, соединёнными одиночными союзами (соединительными или разделительными);

ü  запятая не ставится во фразеологических оборотах (и так и сяк).

Задание № 1. СЛАЙД

Составьте схемы предложений (знаки препинания не расставлены):

1.  Рыбалка успокаивает нервы настраивает на философский лад.

2.  Схватил он его за ноги и руки спеленал поправил все перевязки завернул его в воловью кожу увязал и помчался с ним в дорогу.

3.  Острым ножом мы вырезали узоры то колечко то спираль то шахматную клеточку.

4.  Летом и зимой весной и осенью лес хорош во все времена года.

5.  В каждом отсеке школы на лестницах на этажах в кабинетах везде стояли дежурные.

Б) Сравнительный оборот.

Расскажите о знаках препинания при сравнительных оборотах. СЛАЙД

ü  Сравнительный оборот является частью простого распространённого предложения.

ü  Сравнительные обороты выделяются запятыми.

ü  Если оборот входит в состав сказуемого или тесно связан с ним по смыслу, то запятыми он не выделяется.

ü  Не обособляется оборот, если он входит в состав устойчивого словосочетания.

Задание № 2. СЛАЙД

Тест 1. Укажите предложение с пунктуационной ошибкой, объясните свой выбор:

Озеро, как море, шумит при сильном ветре. Перед нами расстилались пёстрые, как ковёр, луга. Осыпался с берёзы лист и, как ковёр, устлал дорогу. Они были похожи, как две капли воды.

Ответ. 4.

Тест 2. Укажите предложение, в котором на месте всех пропусков ставятся запятые.

1.  Внизу _как зеркало _стальное синеют озера струи.

2.  На чёрном_как тушь_небе не мелькает ни одна звёздочка.

3.  Как парус_чайка_там белеет в высоте.

4. Вода в реке была чёрной_словно_дёготь.

Ответ.2.

В) Обособленные члены предложения.

Ø  Обособленные определения и приложения.

Задание № 3.

Запишите предложения. Объясните знаки препинания.

Группа № 1. Незримый, ты мне был уж мил.

Люди, злые, голодные, двинулись по улице.

Группа № 2. А он, мятежный, ищет бури.

Привлечённые светом, бабочки летели к свече.

Группа № 3. Он видел небо, покрытое тучами.

Иван, усталый, всё же не мог заснуть.

Группа № 4. На небе стыли облака, ещё розовые от заката.

Пёстро раскрашенный, стоял катер у причала.

Группа № 5.Анна Ивановна, в шляпе и шали, вернулась с прогулки.

Он ушёл сильно расстроенный.

Задание№ 4. СЛАЙД

Запишите предложения. Замените обособленные определения необособленными (знаки препинания не расставлены). Какой вариант, по — вашему, более уместен? Какой приём используют писатели? С какой целью? (- инверсия)

Только люди способные сильно любить могут испытывать и сильные огорчения; но та же способность любить служит для них противодействием горести и исцеляет их. Л. Н. Толстой. Песня тихая тягучая и заунывная похожая на плач и едва уловимая слухом слышалась то справа то слева то сверху то из — под земли. А. П. Чехов.

Задание № 5.

Из прочитанных предложений выпишите приложения вместе со словами, которые связывают приложения с другими словами. Составьте предложение с одним из записанных приложений.

Мы, экономисты, народ увлекающийся. Зашёл сосед Иван, поклонник Высоцкого. Всё дорожает белое золото, или хлопок. Иванову, как художнику, поручили оформить холл. Мой пёс, по кличке Сват, спит.

Ответ.

Экономисты, поклонник Высоцкого, или хлопок, как художнику, по кличке Сват.

Ø  Обособленные обстоятельства.

Расскажите об обособлении обстоятельств. СЛАЙД

ü  Обособляются обстоятельства, выраженные деепричастными оборотами и одиночными деепричастиями.

ü  Всегда обособляются обстоятельства с предлогом несмотря на, которые имеют уступительное значение.

ü  Не обособляются фразеологические обороты и деепричастия, непосредственно примыкающие к сказуемому и близкие по функции к наречиям (стоя, сидя, лёжа, молча, нехотя и др.).

ü  Могут обособляться обстоятельства, выраженные существительными с предлогами благодаря, согласно, вопреки, в силу, в случае, при наличии, при отсутствии, по причине, ввиду, вследствие, если они распространены и стоят перед сказуемым.

ü   

Задание № 6. Тесты

1. Укажите правильное объяснение пунктуации в предложении. СЛАЙД

Весело махая хвостом ( ) и тихо повизгивая, Каштанка ждала своего хозяина.

здесь один деепричастный оборот, запятая ставится. причастный оборот перед определяемым словом, запятая не ставится. предложение сложное, запятая ставится. два деепричастных оборота, соединённых союзом и, запятая не ставится.

2. Укажите предложение, в котором ставится одна запятая. (Знаки препинания не расставлены.) СЛАЙД

На севере слабо мерцая подымались какие-то белые облака. Он всегда краснел вспоминая эту историю. Воробей заметив кота мгновенно взлетел вверх. Часто он читал лёжа.

3. Укажите предложение, в котором ставится одна запятая. (Знаки препинания не расставлены.) СЛАЙД

Несмотря на запрещение она пошла к реке надеясь развеять свою печаль. Он женился и вопреки пословице не переменился. Расправив крылья и вытянув шею гусь шёл прямо на меня. Садовник подрезав ветки яблонь закрыл калитку сада и ушёл в дом.

Ø  Обособленные дополнения.

Вопрос.

В каком значении употреблены дополнения с предлогами в следующих предложениях? СЛАЙД

1. Все включая новичков очень старались.

2. Он не видел ничего кроме её смеющихся глаз.

3. Иван в отличие от Вани человек солидный.

4. Вместо бритых скул была у него теперь бородка.

5. Я работал вместо него.

Задание № 7.

Сделайте вывод об обособлении дополнений. Запишите предложения, расставьте знаки препинания.

Г) Уточняющие члены предложения

Вопрос.

Какие члены предложения называются уточняющими? Какими словами могут присоединяться уточняющие члены? СЛАЙД

ü  Уточняться могут все члены предложения:

и главные, и второстепенные.

ü  Уточняющие члены предложения могут быть присоединены к уточняемым словам союзами: то есть, или (в значении «то есть»), иначе, именно и др.,

а также словами: особенно, даже, в частности, в том числе, например и др.

Задание № 8.

Запишите предложение, расставив знаки препинания:

Впереди нас шагах в тридцати видна полынья из глубины которой валит морозный пар.

Задание № 9.

Составьте предложения с уточняющими членами:

несмотря на утренний туман шагах в тридцати от нас в частности учащиеся физмата в том числе русский язык то есть наш сосед

Д) Вводные слова и предложения.

Вопрос.

На какие группы делятся вводные слова по значению? Как они могут выделяться в предложении?

ü  степень уверенности (большая или меньшая)

ü  различные чувства

ü  источник сообщения

ü  порядок мыслей и их связь

ü  способы оформления мыслей

Задание № 10.

Разграничьте в следующих парах предложений вводные слова и синонимичные им члены предложения. СЛАЙД

Наконец он приехал.- Наконец можно обратиться к врачу за помощью. Выбор варианта с моей точки зрения удачен.- Я знаю, что совершено преступление, если смотреть на вещи с точки зрения общей морали.

IV. Подведение итогов. Подведение итогов.

Итоговый тест по теме урока.

Тест по теме «Пунктуация простого предложения»

__________________________________________________

1. Укажите предложение без пунктуационной ошибки.

1)  Наша школа как корабль.

2)  Изделия Фаберже подлинные произведения искусства.

3)  Интеллигентность это способность понимать.

4)  Красота — есть обещание счастья.

2. Укажите предложение с пунктуационной ошибкой.

1)  Площадь комнаты — двадцать квадратных метров.

2)  Жить — Родине служить.

3)  Пятью пять — двадцать пять.

4)  Любовь — есть источник поэзии, добра и ненависти.

3. Укажите правильное объяснение пунктуации в предложении.

Незаметно зажглась одна звезда () и тихо дрожит в тёмном сумраке небес.

1)  В этом предложении союзом и соединены однородные члены, перед и ставится запятая.

2)  Это предложение сложное, перед и ставится запятая.

3)  В этом предложении союз и соединяет однородные члены, запятая перед и не ставится.

4)  Это предложение сложное, запятая перед и не ставится.

4. Укажите, на месте каких цифр в предложении должны стоять запятые.

Няня повествовала (1) с пылом (2) живописно (3) с увлечением (4) и местами вдохновенно.

1)  1,2,3 2) 3,4 3) 2,3 4) 2,3,4

5. Укажите, какие знаки препинания нужны в предложении на месте цифр.

Всеобщее признание принесли Поленову его картины (1) «Московский дворик» (2) «Бабушкин сад» (3) «Заросший пруд».

1)  1 — запятая,2 — запятая, 3 – запятая; 2)1 — двоеточие, 2 — запятая, 3 – запятая;

3) 1 — тире, 2 — запятая, 3 – запятая; 4)1 — запятая, 2 — запятая, 3 – тире

6. Укажите, в каком предложении ставится тире. (Знаки препинания не расставлены.)

1)  Ближе к осени все опустело и дом и усадьба.

2)  Острым ножом мы вырезали узоры то колечко то спираль то шахматную клеточку.

3)  В одной из комнат жили птицы Черноголовки малиновки и даже соловей.

4)  Летом и зимой весной и осенью лес хорош во все времена года.

7. Укажите предложение с пунктуационной ошибкой.

1)  Своего начальника он боялся как огня.

2)  Двор как плац.

3)  С утра поползли серые как дым, облака.

4)  Морозило ещё сильнее, чем вчера.

8.Укажите, в каком предложении на месте всех пропусков ставятся запятые.

1)  Ужинали _ не спеша и почти_ молча.

2)  Несмотря на_ плохую погоду_ соревнования состоялись.

3)  Он обернулся_ и_не поднимая головы_ и _ не показывая ей лица_ушёл.

4)  Заметив людей_конь сердито фыркнул и пустился прочь_ломая кусты.

9. Укажите предложение, в котором ставится одна запятая. (Знаки препинания не расставлены.)

1)  На севере слабо мерцая подымались какие-то белые облака.

2)  Он всегда краснел вспоминая эту историю.

3)  Воробей заметив кота мгновенно взлетел вверх.

4)  Часто он читал лёжа.

10. В каком варианте ответа правильно указаны все цифры, на месте которых в пред­ложении должны стоять знаки препинания?

Оба неразлучные друга (1) глухой и слепой (2) приносили каждую весну глухарей (3) много больше (4) чем все обыкновенные охотники.

1)  1,2,3,4 2) 1,2,3 3) 2,3,4 4) 2,4

11. В каком варианте ответа правильно указаны все цифры, на месте которых в пред­ложении должны стоять знаки препинания?

Много есть на свете (1) кроме нашей страны (2) всяких государств и земель (3) но одна у человека мать (4) одна у него и Родина.

1)  1,2,3,4 2) 3, 4 3) 1,3,4 4) 2, 3, 4

12. В каком варианте ответа правильно указаны все цифры, на месте которых в пред­ложении должны стоять знаки препинания?

Слепой музыкант (1) как это постоянно бывает со слепыми (2) обладал тонким слухом (3) а глухой повар (4) замечательным зрением.

1)  1,2,3,4 2) 1,3,4 3) 1,2,3 4) 3,4

13. На месте каких цифр в предложении нужно поставить скобки?

Дикий барин (1) так его прозвали (2) настоящее же его имя было Перевлесов (3) пользовался огромным влиянием (4) во всей округе.

1)  1,2 2) 2,4 3) 1,3 4) 2,3

14. На месте каких цифр в предложении нужно поставить тире?

Пригибаясь (1) Юргин бросился к своему окошку (2) где у него (3) он помнил (4) лежала связка гранат.

1)  3,4 2) 1,2 3) 2,3 4) 2,4

15. Укажите, в каком предложении пропущена запятая.

1)  Храбрый от природы человек, он не любил никаких рассуждений на эту тему.

2)  Роща на противоположном берегу реки, ещё час назад освещенная майским солнцем
вдруг потемнела.

3)  Тяжелыми шагами ходили по паркету люди, выносившие вещи.

4)  Сорванная с деревьев листва закружилась в воздухе.

16. Укажите, на месте каких цифр в предложении должны быть запятые.

Это была икона (1) вывезенная из Смоленска (2) и с того времени возимая (3) за армией.

1)  1 2) 1,2 3) 1,3 4) 2,3

17. Укажите предложение, в котором на месте всех пропусков ставятся запятые.

1)  Изумлённый и растерянный_он не сразу нашёл_подходящий_ ответ.

2)  Вдоль железной дороги простирались_ невспаханные поля_ и нескошенные луга.

3)  Над рекой повис густой туман_белый_страшный.

4)  Простой народ здесь носит_ плетённые из лёгкого тростника _шляпы.

18. Укажите, на месте каких цифр должны быть запятые в предложении.

С необычайной силой и убедительностью художник умел передать ярость (1) грозно бушующего (2) моря (3) и блеск солнечных лучей (4) сверкающих в белоснежной пене (5) волн.

1)  1,3 2) 1,3,4 3) 3, 4, 5

19. Укажите, на месте каких цифр в предложении должны стоять запятые.

Раскрыв (1) рот от восхищения (2) забыв (3) страх и одиночество (4) Павлик смотрел кругом (5) машинально отвечая на вопросы зоолога.

1)  2,4,5 2) 1,3,4 3) 3,4,5 4) 2,3,4,5

2. Домашнее задание.

А) Актуализация домашнего задания.

Прочитайте размышления о русском языке и его судьбе, принадлежащие замечательным русским писателям XX века. СЛАЙД

Русский народ за свою историю отобрал, сохранил, возвёл в степень уважения такие человеческие качества, которые не подлежат пересмотру: честность, трудолюбие, совестливость, доброту… Мы из всех исторических катастроф вынесли и сохранили в чистоте великий русский язык, он передан гам нашими дедами и отцами… В. М. Шукшин.

Язык открывает свои чертоги
и кладовые только людям с раскрытой душой, тем, у кого сердце не зачерствело в однообразии покоя и борьбы, кто искренен не только с другими,
но и самим собою
…В. И. Белов.

Согласны ли вы с писателями?

Какие проблемы, связанные с современным состоянием русского языка вы считаете наиболее актуальными? Почему?

Домашнее задание.

Прочитайте текст и выполните задания к нему.

(1)Все метафоры, применимые к языку, верны по-сво­ему. (2)Язык — это тюрьма, из которой нам никогда не уда­стся сбежать. (З)Язык — это очки, без которых нам не раз­глядеть окружающий мир. (4)Язык — слуга и господин. (5)Язык — наш друг и враг одновременно. (б)Любой язык изменяется под влиянием различных факторов: внешних или внутренних. (7)Он как будто бы следит за нами и фик­сирует все самые важные наши проблемы и больные ме­ста. (8) Он не даёт ни соврать, ни обмануть самих себя. (9) О состоянии здоровья русского народа довольно красноре­чиво говорит статистика, утверждающая, что смертность русского населения России намного выше рождаемости, а это весьма тревожный сигнал, информирующий о вы­рождении нации, и об этом следует срочно звонить во все колокола.

(10)В полном соответствии со здоровьем народа находится состояние русского языка, терпящего сокрушительное фиаско, усугубляющееся резкой деформацией политической и экономической систем в России.

(11)На гребне реформаторской волны возникла как бы новая общность людей, не обременённых высоким обра­зованием, но с преувеличенными претензиями на веду­щую роль в бизнесе, средствах массовой информации, очагах культуры и т. д.

(12)Они создали свои нравы, свой язык, копирующий «блатную феню». (13)Заявив о себе русским фольклором в форме анекдотов о делах своих и помышлениях-, назва­ли себя «новыми русскими».

(14) На сочной ниве их антирусской лексики ярким бу­кетом расцвели слова, не отмеченные академическими словарями классического русского языка, а именно: не­пруха, неудобняк, нал, бабки, кидало, наезд, прикол, про­кол, по барабану и несть им числа.

(15)Язык «новых русских» проник в жизнеутвержда­ющие речи депутатов Госдумы, достиг эстрады и поразил кинематограф.

(16)Австралийцы, с большой любовью изучающие рус­ский язык и культуру русскую, не могут взять в толк, напри­мер, какое лексическое значение имеет слово «блин» в филь­ме «Особенности национальной охоты», так как оно грамма­тически не связано в фразе, произнесённой героем фильма. (17)Это типичное слово-паразит от «новых русских»; имея примитивно-ругательный смысл, оно жирной кляк­сой ложится на прекрасный русский язык.

(18)Чтобы хоть как-то обогатить свою скудную речь, «новые русские» вкрапляют в неё слова иностранного происхождения (чаще английские), к примеру: эксклюзив, бартер, брокер и уйму других. (19) Слова эти на фоне са­мобытного и богатого фонетически русского языка вы­глядят, извините, как на корове седло.

(20)Увы, мы никогда не будем говорить на языке Турге­нева. (21 ) Что Тургенева — на языке Пришвина и Паустов­ского никогда мы уже не будем говорить вместе с нашими детьми! (22)Так что живём мы сейчас в стрессовых усло­виях языкового разрыва поколений и многоязычия, и един­ственное, что можно посоветовать соотечественникам, так это терпения и терпимости. (23)Ещё через десяток-другой лет наступит период стабилизации, и мы наконец без вся­ких законов обретём наш единый общий русский язык, без которого невозможна русская культура.

(По Е. Чепурову)

1. Определите тип и стиль речи текста.

1)  описание, художественный

2)  повествование, публицистический

3)  рассуждение, публицистический

4)  рассуждение, научно-популярный

2. Укажите предложения, в которых сформулирована позиция автора.

1)  1,9, 13, 18

2)  6, 10, 22, 23

3)  2, 8, 14, 20

4)  3, И, 15, 19

3. Укажите средства выразительности, которые не используют­ся в предложениях 14-19 для передачи особенностей совре­менной языковой ситуации.

1)  эпитет

2)  метафора

3)  сравнение

4)  риторическое восклицание

4. Укажите предложения, в которых содержатся аргументы к тезису, сформулированному в предложении 15.

1)  12-14

2)  13

3)  16-17

4)  18-19

5. Среди предложений 15-21 найдите предложение, которое связано с предыдущим с помощью лексического повтора и указательного местоимения. Запишите его номер.

6. Напишите по прочитанному тексту сочинение-рассуждение.

Урок русского языка по теме:

«Синтаксис и пунктуация простого предложения» в 11 классе.

Тип урока: обобщение и систематизация изученного.

Лингвистическая тема: Синтаксис и пунктуация простого предложения.

Коммуникативная тема: коммуникативно — организационный дидактический материал на тему «Русский язык».

Технология урока: комплексная (работа над орфографическими и грамматическими нормами в группах, систематизация изученного с составлением обобщающей схемы и поэлементной отработкой навыков и умений, тестовый текущий контроль, продуцирование собственного высказывания).

Средства обучения:

ü  Учебник

ü  Дидактический материал для работы в группах над орфоэпическими и грамматическими нормами (на карточках и слайдах презентации)

ü  Аудиозапись речевого образца (разговор, записанный на улице).

ü  Обобщающая таблица «Простое осложненное предложение

ü  Таблицы «Речевые и грамматические ошибки».

ü  Карточки с заданиями и бланк ответов для итогового теста урока «Пунктуация в простом предложении».

ü  Высказывания о русском языке русских писателей (слайды презентации к уроку).

ü  Распечатка текста и заданий к нему для выполнения домашней работы.

ü  Лист учёта знаний для осуществления рефлексии на уроке.

ü  Мультимедийная презентация к уроку.

Цели урока.

I. Познавательные:

1)  Повторить сведения об особенностях простого предложения, пунктуацию простого осложнённого предложения, повторить попутно типы речи, стили речи.

2)  Повторить орфоэпические и грамматические нормы русского языка.

II. Практические:

1)  Обобщить и систематизировать ранее полученные знания по теме «Синтаксис и пунктуация простого предложения».

2)  Закрепить умение расстановки знаков препинания в простом осложнённом предложении.

3)  Формировать умение применять знание норм языка в речи.

4)  Учить выявлять и формулировать проблему на основе исходного текста — высказывания писателя.

III. Общепредметные:

1)  Воспитывать:

а) интерес к предмету в ходе освоения лингвистической темы и размышления над содержанием дидактического материала о языке и о вреде жаргонизмов (высказывания о языке, аудиофрагмент разговора, записанного на улице);

б) бережное отношение к родному языку;

в) языковую личность учащегося, развивающего разные виды компетентности (лингвистической, языковой, речевой, коммуникативной);

г) нравственные качества учащихся (понимание значимости языка как величайшего дара и осознание необходимости бережного отношения к языку);

способность выработать определённый взгляд на нравственные понятия, определённое отношение к ним, эстетическое восприятие высказываний писателей о языке.

2)  Развивать:

а) логическое мышление (мыслительные операции: анализ, синтез, сравнение, обобщение, классификация, систематизация); речь, эмоциональную сферу.

3)  Формировать:

а) умение строить высказывание по заданным критериям;

б) оценивать степень трудности предложенных заданий, уровень собственных знаний, умений.

Урок проводится в 11 классе на этапе систематического повторения изученного за курс средней школы.. Повторение разделов «Синтаксис и пунктуация» считаю целесообразным проводить в несколько этапов:

ü  Синтаксис и пунктуация простого предложения (осложнённого).

ü  Синтаксис и пунктуация сложного предложения.

ü  Сложное синтаксическое целое.

Данный урок позволяет привести в систему знания по указанной теме, полученные в школьном курсе изучения русского языка, использовать их при решении практических лингвистических задач, в том числе, при подготовке к ЕГЭ.

Организация повторения на уроке организована с использованием беседы о языке, построенной на основе высказывания писателя В. Белова, что даёт возможность учащимся эмоционально настроиться на предмет последующего разговора и самим сформулировать коммуникативную тему урока, целевые установки. Повторение орфоэпических и грамматических норм проводится в форме групповой работы, что позволяет в ходе обсуждения в группах вспомнить нормы, найти правильное решение, учит деловому общению, а также способствует здоровьесбережению (снимается напряжение, вызванное индивидуальной и фронтальной формой работы). В качестве заданий предложены:

ü  работа над орфоэпическими нормами (произнести указанные слова);

ü  работа над грамматическими нормами ( согласовать формы подлежащего и сказуемого, исправить ошибки в словосочетаниях, в употреблении падежных форм и предлогов, деепричастных оборотов);

ü  работа над стилистическими нормами в предложениях с однородными членами.

Материал заданий взят из домашних заданий, творческих работ учащихся, поэтому составляет элемент работы над ошибками учащихся. При выполнении подобных заданий учащиеся могут обращаться к таблице «Речевые и грамматические ошибки», таким образом закрепляется умение квалифицировать ошибки, что способствует предупреждению и развитию навыка исправления подобных ошибок. Содержание повторения позволяет также элементом подготовки к ЕГЭ, так как задания сформулированы аналогично заданиям А4, А5. Для проверки правильности после выполнения заданий учащимся предлагается правильный ответ на слайде презентации.

Материал домашнего задания к данному уроку (беседа об актуальности проблем каждого из текстов о языке) также способствует подготовке к ЕГЭ. Учащиеся строят свой ответ как рассуждение (называют тезис, приводят доказательства своей позиции, делают вывод). Высказывания о русском языке учащиеся могут использовать при формулировании проблемы, комментария или подборе аргументов, доказательств по теме «Язык» при работе над текстом в части С.

Повторяются также типы и стили речи.

Материал домашнего задания используется для перехода к лингвистической теме урока: перестроив высказывание В. Шукшина дважды, обращаемся к тема «Простое осложнённое предложение. Синтаксис и пунктуация».

Составление опорной схемы «Простое осложнённое предложение» позволяет актуализировать знания учащихся.

Повторение сведений и отработка пунктуационных навыков осуществляются с использованием мультимедийной презентации, в которую включены сведения об осложнённом предложении и задания различных типов:

ü  Однородные члены предложения и пунктуация при них (задания: составить схемы предложений).

ü  Сравнительный оборот и знаки препинания при сравнительных оборотах (тестовые задания типа А 21).

ü  Обособленные члены предложения (задания: конструирование предложений, тест типа А 19, А22).

ü  Уточняющие члены предложения (задания: расставить знаки препинания в предложении с уточняющими членами, сконструировать предложения с данными уточняющими членами).

ü  Вводные слова и вводные предложения (задание на разграничение вводных слов и синонимичных им членов предложения).

Таким образом, задания предусматривают различные формы работы на уроке, что позволяет эффективно отработать пунктуационные, речевые навыки, навык работы с тестовыми заданиями, в том числе конкретными заданиями ЕГЭ, сохранить внимание учащихся, избежать утомляемости на уроке. Задания различны по степени сложности, что позволяет учащимся оценить свои силы и сделать выводы о необходимости корректировки собственных знаний. В ходе урока по мере выполнения заданий учащиеся отмечают успешность и трудности при выполнении заданий в листке учёта знаний (это могут быть различные отметки: плюсы или минусы, баллы по определённой шкале). Рефлексия на уроке способствует самооценке знаний учащегося. Одиннадцатиклассникам необходимо развивать навыки самоконтроля. Считаю целесообразным после проверки итогового теста урока соотнести результаты оценки учителя и самооценки ученика, что продемонстрирует объективную картину знаний и умений учащегося.

После систематизации и повторения проводится текущий контроль — тест по теме с включением заданий, аналогичных заданиям урока и заданиям ЕГЭ. Эта работа предваряется комментарием учителя о необходимости правильно рассчитать время работы: на каждый тест отводится 1-2 минуты (рекомендации ФИПИ по распределению времени на часть А ЕГЭ — 1-3 мин).

Актуализация домашнего задания проводится на материале высказываний русских писателей XX века. Вопросы учителя возвращают учащихся к разговору о проблемах языка, затронутых в начале урока. Учащиеся формулируют актуальные проблемы современного языка, что позволяет подготовить их к восприятию домашнего задания: работа по тексту о языке, написание сочинения — рассуждения в соответствии с критериями ЕГЭ. Содержание задания предполагает использование при его выполнении материала урока (лингвистические и коммуникативные знания, умения и навыки).

Итак, данный урок может использоваться в 11 классе с целью повышения пунктуационной грамотности учащихся, а также для подготовки учащихся к итоговой аттестации в форме ЕГЭ.

В связи с недостаточным количеством уроков русского языка в учебном плане (1 ч в неделю) урок может использоваться в рамках ПО как занятие элективного курса «Говорим и пишем грамотно» по указанной теме. Материалы можно использовать как на одном, так и на нескольких занятиях в качестве отдельных элементов повторения по указанной теме. Презентационный материал позволяет неоднократно возвращаться к необходимым для повторения сведениям, упражнениям, заданиям, т. е. является универсальным средством для различных видов повторения.

Допишите окончания, согласуя сказуемое с подлежащим. № 1. 1. Большинство картин художников - передвижников

Допишите окончания, согласуя сказуемое с подлежащим. № 1. 1. Большинство картин художников — передвижников был. . воспринят. . как гимн человеку, прославляющий и утверждающий его высокие духовные ценности. № 2. Часть студентов уже сдал. . экзамены по специальности. № 3. Двадцать человек стоял. . в стороне. № 4. Множество темноватых туч расползал. . сь по небу. Ответы. 1. Большинство картин художников — передвижников было воспринято как гимн человеку, прославляющий и утверждающий его высокие духовные ценности. 2. Часть студентов уже сдала экзамены по специальности. Двадцать человек стояли в стороне. 3. Множество темноватых туч расползалось по небу. Десять студентов окончили институт с отличием.

Б) Исправьте ошибки в употреблении словосочетаний № 1. Отчитаться в получении полученного аванса. №

Б) Исправьте ошибки в употреблении словосочетаний № 1. Отчитаться в получении полученного аванса. № 2. Отзыв на статью. № 3. Уделять внимание на каждую мелочь. № 4. Превосходство новой техники перед старой. № 5. Примириться пред неумолимой судьбой. • Ответы. 1. Отчитаться (о чём? ) о получении полученного аванса. 2. Отзыв (о чём) о статье. 3. Уделять внимание (чему? ) каждой мелочи. 4. Превосходство новой техники (над чем? ) над старой. 5. Примириться (с чем? ) с неумолимой судьбой.

В) Исправьте ошибки в употреблении падежных форм и предлогов. № 1. Об этом мы

В) Исправьте ошибки в употреблении падежных форм и предлогов. № 1. Об этом мы познакомим вас позже. № 2. Докладчик подчеркнул о необходимости строительства. № 3. Решительность парламента свелась лишь в предоставлении полномочий. № 4. Вчера брат приехал с Москвы. № 5. По окончанию переговоров представители сторон подписали совместное заявление. • Ответы. • С этим мы познакомим вас позже. Докладчик подчеркнул необходимость строительства. Решительность парламента свелась лишь к предоставлению полномочий. Вчера брат приехал из Москвы. По окончании переговоров представители сторон подписали совместное заявление.

1. Укажите предложение без пунктуационной ошибки. 1) Наша школа как корабль. 2) Изделия Фаберже

1. Укажите предложение без пунктуационной ошибки. 1) Наша школа как корабль. 2) Изделия Фаберже подлинные произведения искусства. 3) Интеллигентность это способность понимать. 4) Красота — есть обещание счастья. 2. Укажите предложение с пунктуационной ошибкой. 1) Площадь комнаты — двадцать квадратных метров. 2) Жить — Родине служить. 3) Пятью пять — двадцать пять. 4) Любовь — есть источник поэзии, добра и ненависти.

1. Укажите предложение без пунктуационной ошибки. 1) Наша школа как корабль. 2) Изделия Фаберже

1. Укажите предложение без пунктуационной ошибки. 1) Наша школа как корабль. 2) Изделия Фаберже подлинные произведения искусства. 3) Интеллигентность это способность понимать. 4) Красота — есть обещание счастья. 2. Укажите предложение с пунктуационной ошибкой. 1) Площадь комнаты — двадцать квадратных метров. 2) Жить — Родине служить. 3) Пятью пять — двадцать пять. 4) Любовь — есть источник поэзии, добра и ненависти.

 • Синтаксис – это • Словосочетание – это • В словосочетании выделяются ….

• Синтаксис – это • Словосочетание – это • В словосочетании выделяются …. слово и …. зависимое слово • Смысловая связь слов в словосочетании устанавливается при помощи… • Грамматическая связь осуществляется при помощи….

В 3 Из предложения 19 выпишите подчинительное словосочетание со связью СОГЛАСОВАНИЕ. (19)У них желания

В 3 Из предложения 19 выпишите подчинительное словосочетание со связью СОГЛАСОВАНИЕ. (19)У них желания сбываются чаще, и, даже если этого не происходит, он не зацикливаются на неудачах, а находят в ситуации светлые стороны

Словосочетаниями не являются: 1 2 3

Словосочетаниями не являются: 1 2 3

Согласование – это такой вид подчинительной связи, при которой зависимое слово уподобляется главному в

Согласование – это такой вид подчинительной связи, при которой зависимое слово уподобляется главному в общих у них грамматических формах (рода, числа, падежа), например: хороший специалист, заветная мечта, любимое дело. При согласовании с изменением форм главного слова соответственно изменяются и формы зависимого слова: важное дело (Им. п. ), важным делом (Р. п. ), о важном деле (П. п. ), важные дела (мн. ч. ). При определении вида словосочетания важно обращать внимание на зависимые слова: какой частью речи они выражены. При согласовании зависимым словом могут быть ПРИЛАГАТЕЛЬНОЕ, ПРИЧАСТИЕ, МЕСТОИМЕНИЕ, ЧИСЛИТЕЛЬНОЕ: частые встречи, летящий мяч, наше право, второй ряд.

Управление - это такой вид подчинительной связи, при которой главное слово требует употребления зависимого

Управление — это такой вид подчинительной связи, при которой главное слово требует употребления зависимого слова в определённом падеже с предлогом или без предлога, например: написание реферата (Р. п. ), работа над проблемой (Т. п. ), интерес к предмету (Д. п. ). Зависимым словом при управлении может быть СУЩЕСТВИТЕЛЬНОЕ или МЕСТОИМЕНИЕ: начало игры, попросить её.

 • Примыкание - это такой вид подчинительной связи, при которой зависимое слово, не

• Примыкание — это такой вид подчинительной связи, при которой зависимое слово, не имея форм словоизменения, присоединяется к главному слову по смыслу, например: стараться понять, чтение вслух, очень хорошо. Зависимым словом примыкании могут быть ДЕЕПРИЧАСТИЕ, НЕОПРЕДЕЛЁННАЯ ФОРМА ГЛАГОЛА, НАРЕЧИЕ, СРАВНИТЕЛЬНАЯ СТЕПЕНЬ ПРИЛАГАТЕЛЬНОГО: спать стоя, просил остаться, бежать быстро, стать лучше.

Обратите внимание!!! Особое внимание следует обратить на словосочетания с его, её, их притяжательными местоимениями.

Обратите внимание!!! Особое внимание следует обратить на словосочетания с его, её, их притяжательными местоимениями. Они, в отличие от омонимичных им форм личных местоимений, не изменяются и, соответственно, вступают только в связь примыкание. Сравните: «Назавтра его должны были освободить» — должны были освободить (кого? ) его – его здесь форма родительного падежа личного местоимения он, следовательно, перед нами связь управление. «Его голова была устроена иначе, чем моя» — голова (чья? ) его – это притяжательное местоимение, которое не изменяется: его голова, его головой, его голове, следовательно, примыкает к главному слову.

Обратите внимание!!! • Особый случай примыкания – когда зависимым словом является инфинитив: «Прошу записать

Обратите внимание!!! • Особый случай примыкания – когда зависимым словом является инфинитив: «Прошу записать меня на курсы водителей» . В данном случае сочетание прошу записать не является составным глагольным сказуемым, так как действие осуществляется разными субъектами (действующими лицами): прошу я, а записывать будете вы / они и т. п. , то есть другое лицо / лица, и инфинитив в данном случае является не частью составного сказуемого, а дополнением (прошу (о чем? ) записать). В словосочетании прошу записать инфинитив примыкает к главному слову.

Для выполнения задания В 3 используйте следующий АЛГОРИТМ: 1. При определении нужного словосочетания посмотрите,

Для выполнения задания В 3 используйте следующий АЛГОРИТМ: 1. При определении нужного словосочетания посмотрите, как связаны между собой слова: если оба слова изменяются – согласование; если одно слово требует от другого употребления в каком-либо падеже – управление; если слова связаны только по смыслу и между ними нет грамматической связи, которая осуществляется с помощью окончания и предлога или только окончания – примыкание.

2 Проанализируйте зависимые слова: q если зависимым словом является прилагательное, причастие, местоимение, числительное –

2 Проанализируйте зависимые слова: q если зависимым словом является прилагательное, причастие, местоимение, числительное – это согласование; q если зависимым словом является существительное или местоимение, употреблённое в том или ином падеже – это управление; q если зависимым словом является деепричастие, неопределённая форма глагола, наречие, сравнительная степень прилагательного – это примыкание.

Синтаксический рецепт Если сложить начальные буквы названий всех трех способов подчинительной связи, получится слово

Синтаксический рецепт Если сложить начальные буквы названий всех трех способов подчинительной связи, получится слово СУП. Как известно, суп очень полезен. Но чтобы он получился вкусным, нужно знать, какие ингредиенты необходимы. Итак, варим синтаксический СУП. Первый ингредиент – согласование. Для того, чтобы его приготовить, возьмите самостоятельную часть речи и прибавьте по вкусу прилагательное, причастие, порядковое числительное или местоимение, сходное с ним по форме. Второй ингредиент – управление. Для того, чтобы его приготовить, к самостоятельной части речи нужно добавить существительное или местоимение. Третий ингредиент – примыкание. Чтобы приготовить хорошее примыкание, возьмите самостоятельную часть речи и смешайте ее с неизменяемым словом. После этого аккуратно, не перемешивая ингредиенты, выкладываем их в кастрюлю и варим до полной готовности. Через некоторое время вы получите вкусный, полезный синтаксический СУП под названием «Способы подчинительной связи» . Приятного аппетита!

Выполните задания 2 -10 по тексту: (1)В течение своей жизни Гоголь много путешествовал. (2)Однако

Выполните задания 2 -10 по тексту: (1)В течение своей жизни Гоголь много путешествовал. (2)Однако где бы он ни странствовал, в дороге он чувствовал себя несравненно лучше, чем сидя на одном месте. (3)Он приспособился в дороге думать и работать, между тем одновременно и набираясь впечатлений, и отдыхая душой. (4)У него даже был план объехать все монастыри России, имея в виду, что они располагались, как правило, в красивейших местах. (5)Он собирал в душе своей как бы коллекцию истинно русских пейзажей. (по В. Солоухину)

2. Из предложения 1 выпишите словосочетание со связью СОГЛАСОВАНИЕ. 3. Какой способ связи в

2. Из предложения 1 выпишите словосочетание со связью СОГЛАСОВАНИЕ. 3. Какой способ связи в словосочетании в течение жизни путешествовал (предложение 1)? 4. Из предложения 1 выпишите словосочетание со связью ПРИМЫКАНИЕ. 5. Какое сочетание слов из предложения 2 не является примером примыкания: 1) где бы ни странствовал 2) чувствовал себя лучше 3) сидя на месте 4) несравненно лучше

6. Определите способ связи в словосочетании на одном месте (предложение 2). 7. Определите способ

6. Определите способ связи в словосочетании на одном месте (предложение 2). 7. Определите способ связи в словосочетании набираясь впечатлений (предложение 3). 8. Какое сочетание слов из предложения 4 является словосочетанием: 1)был план 2)план объехать 3)даже был 4)располагались как правило 9. Определите способ связи в словосочетании у него был (предложение 4). 10. Какое сочетание слов из предложения 5 не является словосочетанием: 1)в душе своей 2) как бы коллекцию 3)собирал в душе 4) коллекцию пейзажей •

Проверьте себя. 1. будущее (какое? ) прекрасное - согласование прибавить (к чему? ) к

Проверьте себя. 1. будущее (какое? ) прекрасное — согласование прибавить (к чему? ) к четырём — управление храбрость (чья? кого? ) солдата — управление в пору (какую? ) счастливую — согласование глядеть (куда? вслед чему? ) вслед поезду — управление выть (как? ) по-волчьи — примыкание рублями (сколькими? ) пятьюстами — согласование трудов (чьих? ) моих — согласование работа (чья? ) её — примыкание пришёл, (несмотря на что? ) несмотря на болезнь — управление находиться (где? посредине чего? ) посередине комнаты – управление

2. 3. 4. 5. 6. 7. 8. 9. 10. своей жизни управление много путешествовал

2. 3. 4. 5. 6. 7. 8. 9. 10. своей жизни управление много путешествовал сидя на месте согласование управление 2 Управление 2

Помните: Формальный подход к синтаксическому анализу, который не учитывает логичности установления смысловых отношений между

Помните: Формальный подход к синтаксическому анализу, который не учитывает логичности установления смысловых отношений между словами в предложении, влечёт ошибки как в выделении словосочетаний, так и в определении членов предложения, а также частей речи. Так, в предложении Представьте, вокруг всё сияет: и небо, и солнце, и трава, и река слово вокруг в словосочетании примыкает к главному слову: сияет (где? ) вокруг, обозначает признак действия и является наречием места. В предложении И грибы уже не собираются, а вокруг когда-то прозрачнейшего озера выросли чудовищные лопухи и папоротники смысловые связи того же слова другие: выросли (где? ) вокруг озера. Слово вокруг помогает связать в подчинительном словосочетании главное слово и управляемое им зависимое, выполняя служебную функцию предлога. Итак, в 1 м примере способ связи слов – примыкание, а слово вокруг – наречие; во 2 -м примере способ связи слов – управление, а слово вокруг – производный предлог.

1) Глаголы с разным управлением в качестве однородных
членов предложения:

Слова с
разным управлением могут использоваться в качестве однородных членов, если
каждое из них имеет свои зависимые слова, употреблённые в нужном падеже.
Например:

Она ждала (кого?) его (В.п.) и звонила (кому?) ему
(Д.п.) весь вечер.

Часто
построить такие предложения правильно помогают местоимения:

Дети редко прислушиваются  (к чему?) к советам (Д.п. с предлогом
к) взрослых и следуют 
(чему?)им  (Д.п.).
Люди были измучены, но верили (во что?) в
победу (В.п. с предлогом в) и надеялись 
(на что?) на
неё (Д.п. с предлогом на).

2)Разное управление у глаголов и  существительных:

интересоваться искусством (Т.п.) — интерес к искусству (Д.п.)
любить литературу (В.п.) — любовь к литературе (Д.п.)
ненавидеть беспорядок (В.п.) — ненависть к беспорядку (Д.п.)
уважать коллег (В.п.) — уважение к коллегам (Д.п.)
доверять другу (Д.п.) — доверие к друзьям (Д.п.)
сочувствовать слабому (Д.п.) — сочувствие к слабому (Д.п. с предлогом)

3). Разное управление у слов со сходным значением:

беспокоиться о чём-либо (П.п.) — тревожиться за что-либо
(В.п.)
оплатить что-либо (В.п.) — заплатить за что-либо (В.п.  с предлогом за)
уплатить за проезд (В.п. с предлогом за) – оплатить проезд (В.п.)
уделить внимание чему-либо (Д.п.) — обратить внимание на что-либо (В.п. с
предлогом на)
сообщать кому-либо (Д.п.) — информировать кого-либо (В.п.) 
упрекать в чём-либо (П.п. с предлогом в) — осуждать за что-либо
(В.п. с предлогом за)
отзыв о чём-либо (П.п. с предлогом о) — рецензия на что-либо
(В.п. с предлогом на)
вера во что-либо (В.п. с предлогом в (во)) — уверенность в
чём-либо (В.п. с предлогом в)
свойственный чему-либо (Д.п.) — характерный для чего-либо (Р.п. с
предлогом для)  

4) Разное управление у глаголов с отрицанием и без него:

Замечать реакцию (В.п.) – не заметить реакции (Р.п.)
Он замечал (что?) реакцию зала на его слова. — От волнения он не замечал
(чего?) реакции зала на его слова.

5) Употребление названий произведений литературы и
искусства.

В «Войне и мире» актёр В. Тихонов сыграл
роль князя Андрея. В кинофильме «Война и мир» актёр В.
Тихонов сыграл роль князя Андрея. 
После слов — родовых обозначений, например, сказка, роман,
повесть, рассказ, картина, кинофильм 
и других подобных, называющих жанр
произведения литературы или искусства, имя собственное ставится в именительном
падеже. Если такие слова в предложении отсутствуют, то названия произведений
литературы и искусства используются в тех падежах, каких необходимо для
контекста предложения.

В сказке «Репка» либо: В «Репке»
В кинофильме «Война и мир» либо: В «Войне и мире»
В балете «Щелкунчик» либо: В «Щелкунчике

ЗАПОМНИ
Многие
глаголы требуют после себя определённого падежа существительного.

Родительного падежа требуют глаголы:

Достигать, добиваться, желать, жаждать, хотеть,
ожидать, наделать, опасаться, остерегаться, бояться, избегать,
лишаться, пугаться, стыдиться, сторониться, стоить, искать, просить,
требовать  и др. (кого? чего?) 

Глаголы с отрицанием: не видеть, не
замечать, не слышать и др. (кого? чего?) 

Дательного падежа требуют глаголы:

кому? чему?

Давать, верить,
доверять, угрождать, потакать, учиться, радоваться,
улыбаться, говорить, отвечать, грозить,
угрожать, возражать, кланяться, кивать, махать, сигналить, звонить, писать,
говорить, рассказывать, объявлять, отвечать, объяснять,
сообщать, нравиться, казаться, мешать, вредить, изменять,  мстить, надоедать,
опротиветь, дарить, покупать, приносить, посылать, показывать, помогать,
обещать, сниться и др. (кому? чему?)

Винительного падежа требуют все переходные глаголы:

кого? что?

Давать, дарить,
продавать, покупать, посылать, показывать, обещать, строить, шить, убирать,
мыть, стирать, брать, класть, ставить, вешать, видеть, смотреть, слышать,
слушать, чувствовать, испытывать, замечать, любить, ненавидеть, презирать,
уважать, ценить, помнить, понимать, изучать, решать, учить, рассказывать,
объяснять, сообщать, говорить, благодарить, поздравлять, вспоминать, встречать,
ругать, ждать  и др. (кого? что?) 

Творительного падежа требуют глаголы:

кем? чем?

Править, руководить, распоряжаться, командовать, управлять,
верховодить, заведовать, увлекаться, интересоваться, заниматься,
любоваться, восхищаться, восторгаться, наслаждаться, гордиться, восторгаться,
восхищаться, пленяться, дорожить, владеть, пользоваться, обладать,
овладевать, хвалиться, гордиться, хвастаться,
клясться, торговать, жертвовать, рисковать, быть, стать, становиться,
явиться, оказаться, остаться, считаться, слыть, называться и др. (кем? чем?)

Для многих глаголов характерно двойное управление:

Дать, передать, вручить, выдать, продать, возвратить, дарить,
сдать, предоставить, вверить, уступить, оставить кому-либо что-либо

Сказать, объяснить, объявить, внушить, рассказать, заявить, ответить,
обещать, рекомендовать кому-либо что-либо

Обещать, гарантировать кому-либо что-либо
Учить кого-либо чему-либо
Считать, вообразить, признать, представить, называть, изображать, обругать,
объявить кого-либо кем-либо

Варианты норм

Хотеть, желать, жаждать, просить, заслуживать награду —
награды (
В.п. и Р.п), но: заслужить
награду (
В.п.)
Спрашивать совета, разрешения – совет, разрешение (Р.п. и В.п.)
Ждать поезда, звонка – поезд, звонок (Р.п. и В.п.),
но ждать бабушку, сестру (
В.п.)
Дать, брать, добыть, получить, прислать, купить, положить, налить,
насыпать, выпить, глотнуть, попробовать воду, сахар – воды, сахара 
(В.п.
и Р.п.)

ВНИМАНИЕ

 Скучать (по чему? кому?)
по работе, по дому, по матери, по мужу

Но с местоимениямискучать (по ком?)
по нас, по вас
.

2. Конструкции с предлогами

 1) предложное
 управление:

благодаря, согласно, вопреки,
наперекор, подобно
 + Д.п. существительного,

 Например: вопреки указанию, правилам, мнению
близких, согласно приказу…

по (в значении «после
чего-либо»)
 + П.п. существительного,

 Например: по приезде, по возвращении, по
завершении эксперимента…

в меру, в силу, в течение, в
продолжение, в заключение, по причине, по завершении, наподобие, посредством
 + Р.п. существительного,

Например: в продолжение разговора, в течение недели...

2) использование предлогов
в беспредложных конструкциях:

Статья послужила толчком мыслям (нельзя: к
мыслям).

Заказчик выставил рекламацию заводу-изготовителю (нельзя: к
заводу-изготовителю)

3) предлоги при однородных
членах предложения:

Мне нужно на вокзал, на почту и в магазин.
Занятия проходят на стадионе, в парке и в зале.

Если требуются разные предлоги с разными существительными, они
должны быть использованы. Пропуск предлогов в таких случаях недопустим.

4) предлоги в, на, с, из:

В Камергерском переулке, магазин в
Камергерском — на улице, магазин на Тверской

Из Ростова, из театра, из магазина, из
парка, из ссылки, из цирка, из клуба, из консерватории, из ресторана, из школы,
из класса, из аэропорта, из порта, из института, из университета, из
библиотеки, из больницы — с юга, с площади, с бульвара, с почты, с рынка, с
лекции, с представления, с концерта, со станции, с вокзала

Задание №1: Найдите
ошибки в употреблении словосочетаний, исправьте их:

1. Отчитаться в
израсходовании полученного аванса.

2. Отзыв на статью.

3. Уделять внимание
на каждую мелочь.

4. Превосходство
новых технологий перед старыми.

5. Примириться перед
неумолимой судьбой.

6. Рассержен за
выходку приятеля.

7. Уверенность в
торжество справедливости.

Задание №2. Составить
словосочетания, поставив заключенные в скобках слова в нужном падеже:

Удивляться
(результаты)

Преклонение (талант)

Платить (квартира)

Упрекать (грубость)

Уверенность (победа)

Оплатить (проезд)

Заведовать
(отделение)

Управляющий  (филиал)

Согласно (приказ,
распоряжение)

Вера (победа)

Стремление
(деятельность)


Подборка по базе: Русский язык и культура речи Практическое занятие 3 Мишина Полин, русский язык 9кл.docx, Задание к семинару №2 по дисциплине Иностранный язык (для уровня, Задание к семинару №1 по дисциплине Иностранный язык (для уровня, Сербский язык.docx, 10 кл Урок 1 Введение в Модуль1 Strong ties.docx, русский язык задание 3.docx, Анализ контрольной работы по русскому языку в 7 классе за 4 четв, Роль и значение Державина в русском языке.docx, Лаб1. Основы языка VHDL Койчубаев КИ20-05Б.docx


СТИЛИСТИЧЕСКИЕ НОРМЫ

Стилистические нормы – это правила употребления языковых единиц в соответствии с их стилистическими качествами и характеристиками, т.е. стилистической окраской.

Стилистическая окраска языковой единицы представляет собой дополнительную к лексическому значения слова, словосочетания или предложения информацию

 о преимущественной сфере их употребления и принадлежности к той или иной форме существования русского национального языка (литературному языку, диалектам, просторечию, жаргонам);

 об отнесенности языкового знака к функциональной разновидности литературного языка (научному, официально-деловому, публицистическому стилю, языку художественной литературы или разговорной речи);

 об отнесенности к тому или иному временному или экспрессивному разряду языковых средств (явлениям устаревшим или современным, новым; высоким, торжественным или сниженным.

Ср.: голова (литер.) – башка (прост., снижен.); глаза (литер.) – очи (книжн., устар., высокое) – орган зрения (книжн., спец.); полотенце (литер.) – утирка (диал.); картофель (литер., офиц.-дел.)  картошка (литер., разг.).

Стилистическая окраска определяет уместность использования того или иного языкового средства в определенной речевой ситуации. Употребление в речи слов, словосочетаний и предложений с несвойственной данной ситуации общения и данному тексту стилистической окраской воспринимается как речевая ошибка.
Упр. 15. В приведенных предложениях найдите чужеродные, иностилевые элементы. Исправьте ошибки. Запишите правильный вариант.

1. Перед принятием пищи надо вымыть руки.

Перед ПРИЕМОМ пищи надо вымыть руки.

2. Нас встретил седой величественный старичок.

Нас встретил седой ВЕЛИЧАВЫЙ старичок. 3. Князь оседлал боевую лошадь и двинулся в путь. Князь оседлал боевого КОНЯ и двинулся в путь. 4. Завод выпускает специальные агрегаты для уборки картошки. Завод выпускает специальную ТЕХНИКУ для уборки картошки. 5. Грядущие каникулы я проведу у моря. ПРЕДСТОЯЩИЕ каникулы я проведу у моря. 6. Поданная мною справка была заверена секретаршей деканата. Поданная мною справка была заверена СЕКРЕТАРЕМ деканата.

Упр. 16. Определите экспрессивно-стилистическую окраску слов и фразеологизмов (высок., сниж., нейтр.).

Принять в дар(В), здоровяк(СН), попадать на крючок(Н), держава(В), норма(Н), до последней капли крови(В), отлично(Н), грязнуля(СН), соратник(СН), глас вопиющего в пустыне(В).

Упр. 17. Определите, к какой разновидности национального русского языка (к литературному языку, просторечию, территориальным диалектам или социальным диалектам) относятся следующие фонетические варианты и грамматические формы.

Килóметр(ТД), намерéние(П), эксплатировать(П), транвай(П), дермантин(ЛЯ), психиатор(П), туфель(П), мыш(П), с повидлой(П), под роялью(П), делов(ТД), с людями(ТД), на пляжу(ТД), нет время(П), к Марии Васильевной(П), мате[р’]я́(П), стака[н’]ы́(ТД), наиболее чаще(П), у ней(П), хочут(ТД), берегёт(ТД), броюсь(П), плотит(ТД), ездию(П), стонает(ТД), ляжь(П), ехай(П), поломатый(П), достигать до цели(П).

ГРАММАТИЧЕСКИЕ НОРМЫ

К грамматическим нормам относятся нормы словообразовательные, морфологические и синтаксические.

Словообразовательныенормы  это правила образования слов. Ср.: насмешка (не надсмешка), поскользнуться (не подскользнуться).

Упр. 18. Найдите ошибки, которые появились в связи с неправильным образованием слов. Исправьте предложения.

1. Змеи относятся к животным класса пресмыкающих.

Змеи относятся к животным класса пресмыкающихся.

2. При гололеде будьте внимательны на дорогах, чтобы не подскользнуться и не упасть прямо на проезжей части.

При гололеде будьте внимательны на дорогах, чтобы не поскользнуться и не упасть прямо на проезжей части.

3. Эта история может стать самым смотрибельным сериалом.

Эта история может стать наиболее просматриваемым сериалом. 4. У него был вспыхчивый характер. У него был вспыльчивый характер. 5. Его предсмертельные слова произвели большое впечатление. Его предсмертные слова произвели большое впечатление.

6. Рецензист был слишком резок в своей оценке.

Рецензент был слишком резок в своей оценке.

7. На физкультуре я его не только догнал, но и передогнал.

На физкультуре я его не только догнал, но и обогнал.

Упр. 19. Разграничьте случаи намеренного (словообразовательная игра) и ненамеренного (ошибка) нарушения словообразовательных норм. Предложения с ошибками отредактируйте.

1. Обытоживающим было выступление его заместителя.

Подытоживающим было выступление его заместителя.

2. В результате испугивания некоторые студенты на контрольной работе были в недоумевании.

В результате испуга некоторые студенты на контрольной работе были в недоумении.

3. Хлебозавод предлагает бублы, ватрухи, прянища и кренделища. Надоело заниматься мелочёвкой.

Хлебозавод предлагает бублики, ватрушки, пряники и крендели. Надоело заниматься мелочёвкой.

4. Здесь пляжатся и курортятся несколько тысяч человек.

Здесь отдыхают на пляже и находятся на курорте несколько тысяч человек.

5. ЦСКАзка с печальным концом.

ЦСКА с печальным концом.

6. Жена сына претензирует на всё то, что было ими не нажито в совместной жизни.

Жена сына претендует на всё то, что было ими не нажито в совместной жизни.

Морфологические нормы– это правила образования форм слов разных частей речи. Ср.: бухгалтеры (не бухгалтера), слезай (не слазь), красивее (не красивше и не более красивее), три студентки (не трое студенток).

Упр. 20. К приведенным словам подберите определения. Правильно согласуйте определения с существительными.

Ядовитый аэрозоль, Яблочное повидло, Старинное бра, Бешеная мышь, Вареное щупальце, Ржавый рельс, Эксклюзивный тюль, Продвинутая молодежь, Гнилой толь, Трудовая мозоль, Срочная бандероль, Яичная вермишель, Старая вуаль, Полный тезка, Мелкая фасоль, Тихий тамада.
Упр. 21. Вставьте пропущенные буквы, раскройте скобки.

1. Необходимо вымыть волосы шампун

ем от перхоти. 2. Домик на пригорке был черный и крыт (толь) толем 3. Наконец я смогла купить пять метров красивого (тюль) тюля. 4. (Ставень  ставня) Ставня слетела с петли. 5. Снова разболелась мозоль. 6. Покупатель попросил показать ему левый (туфель  туфля) туфель 7. Заказную бандероль можно получить в окне № 5. 8. Манжеты платья оформлены красивой аппликацией из бархата. 9. В доме есть большой (зал  зала) зал.

Упр. 22. Образуйте форму единственного числа следующих существительных.

Туфли –

туфля, тапки – тапок, босоножки – босоножка, погоны – погон, клипсы – клипса, плацкарты – плацкарт, щупальца – щупальце, бакенбарды – бакенбарда, чучела – чучело, ласты – ласт.

Упр. 23. Определите род следующих несклоняемых существительных. Мотивируйте свой ответ.

Алиби – ср.р., бра – ср.р., такси – ср.р., манго – ср.р., пари, рагу – ср.р., кофе – м.р., пони – м.р., цунами – ср.р., какаду – м.р., какао – ср.р., фойе – ср.р , ООН – ж.р., Дели – м.р., хинди – ср.р., МВФ (Международный валютный фонд) – м.р., кенгуру – м.р., шимпанзе – м.р., «Таймс».

Упр. 24. Образуйте нормативные формы множественного числа следующих существительных.

Жемчуг 

Жемчуга, вексель  Векселя, инженер  Инженеры, купол  Купола, конструктор  Конструкторы, бухгалтер  Бухгалтеры, округ  Округа, слесарь  Слесари, приговор  Приговоры, ордер  Ордеры, паспорт  Паспорта, прожектор  Прожекторы, сорт  Сорта, выбор – Выборы, профессор  Профессора, трактор – Тракторы, цех – Цеха, орден – Ордена, директор – Директора, договор – Договоры, катер – Катера, сектор – Сектор, пальто – Пальто, рапорт – Рапорты, ректор – Ректоры, редактор – Редакторы, сват – Сваты, шофер  Шоферы.

Упр. 25. Выберите из скобок один из вариантов; подчеркните его.

1. Водить автобусы по горным дорогам могут только опытные (

шоферы – шофера). 2. (Выборы – выбора) губернатора области показывают, что политические противники заключают (договоры – договора) о сотрудничестве. 3. В летнее время у многих людей начинаются (отпуски – отпуска). 4. В кондитерском отделе всегда свежие (торта-торты). 5. Многие (цехи – цеха) завода переориентированы на выпуск новой продукции.

Упр. 26. От существительных образуйте формы родительного падежа множественного числа.

Апельсины –

Апельсинов , баклажаны – Баклажанов, башкиры – Башкир, блюдца – Блюдец, ботинки – Ботинок, будни – Будней, валенки – Валенок, вафли – Вафель, граммы – Граммов, гектары – Гектаров, грузины – Грузин, мандарины – Мандаринов, носки – Носков, полотенца – Полотенец, сапоги – Сапог, солдаты – Солдат, туфли – Туфель, чулки – Чулок, ясли  Яслей.

Упр. 27. Просклоняйте: 658 студентов, 395 тортов, 85-я школа. Пользуйтесь справочными материалами.

И.п. Шестьсот пятьдесят восемь студентовР.п. Шестисот пятьдесят восемь студентовД.п. Шестистам пятидесяти восьми студентамВ.п. Шестьсот пятьдесят восемь студентов Т.п. Шестьюстами пятидесятью восьмью студентамиП.п. Шестисот пятидесяти восьми студентах
И.п. Триста Девяносто Пять тортовР.п. Трёхсот девяноста пяти тортовД.п. Трёмстам девяноста пяти тортамВ.п. Триста девяносто тортовТ.п. Тремястами девяносто пятью тортамиП.п. Трехстах девяносто пяти тортах
И.п. 85-ая школа
Р.п. 85-ой школы
Д.п. 85-ый школе
В.п. 85-ую школу
Т.р. 85-ым школой
П.п. О 85-ом школе
Упр. 28. Запишите предложения, заменяя цифры словами.

1.К 2015 г. количество первоклассников в нашем городе значительно увеличится. К две тысячи пятнадцатому году количество первоклассников в нашем городе значительно увеличится. 2. Библиотека института ежегодно пополняется 300 – 400 книгами.

Библиотека института ежегодно пополняется тремя-четырьмя сотнями книг. 3. Вес третьего искусственного спутника Земли был равен 1326 кг. Вес третьего искусственного спутника Земли был равен одной тысяче тремстам двадцати шести килограммам. 4. Небольшой старинный город с 4675 жителями, красиво расположенный по обеим сторонам живописной реки, привлекает много туристов. Небольшой старинный город с четырьмя тысячами шестьюстами семьюдесятью пятью жителями, красиво расположенный по обеим сторонам живописной реки, привлекает много туристов.

5. Теплоход с 388 экскурсантами отправился в очередной рейс.

Теплоход с тремястами восьмьюдесятью восемью экскурсантами отправился в очередной рейс. 7. В 2000 г. в нашем институте был открыт новый факультет. В двух тысячном году в нашем институте был открыт новый факультет.

Упр. 29. Исправьте ошибки, вызванные неправильным образованием падежной формы количественных числительных.

1. Во время полета была выполнена широкая программа медико-биологических исследований, включавшая в себя около пятьдесят экспериментов.

Во время полета была выполнена широкая программа медико-биологических исследований, включавшая в себя около пятидесяти экспериментов 2. С семи русских кораблей успели выпустить по неприятелю более восемьсот снарядов. С семи русских кораблей успели выпустить по неприятелю более восьмисот снарядов. 3. Протяженность столичной подземной дороги достигла сто семьдесят три километра, а автобусных и троллейбусных линий четыре тысячи триста километров. Протяженность столичной подземной дороги достигла ста семидесяти трех километров, а автобусных и троллейбусных линий четырех тысяч трехсот километров. 4. Вместе с новыми тысяча двухсот тремя словами учебник немецкого языка будет насчитывать свыше четыре с половиной тысячи слов. Вместе с новыми тысячей двумястами тремя словами учебник немецкого языка будет насчитывать свыше четырех с половиной тысяч слов.

Упр. 30. Найдите и исправьте ошибки, связанные с неправильным употреблением числительных.

1. Они воспитали двух сыновей и двоих дочерей.

Они воспитали двоих сыновей и двух дочерей.

2. У обоих фирм есть долгосрочные обязательства перед партнёрами.

У обеих фирм есть долгосрочные обязательства перед партнёрами.

3. Пятеро студентов отправили домой.

Пятерых студентов отправили домой.

4. С докладом выступили трое академиков.

С докладом выступили тти академика.

5. На столе лежали четверо тетрадей и двое очков.

На столе лежали четыре тетради и двое очков.

Упр. 31. Образуйте от приведенных ниже глаголов формы 1 лица единственного числа настоящего времени; 3 лица единственного и множественного числа настоящего времени; повелительного наклонения.

Образец: люблю – любит – любят – люби.

Бегать –

бегаю – бегает – бегают – бегай

бежать –

бегу – бежит – бегут – беги

бриться 

бреюсь – бреется – бреются – брейся

ехать 

еду – едет – едут – езжай

ездить 

езжу – ездит – ездят – езди

класть 

кладу – кладет – кладут – клади

махать 

машу – машет – машут – маши

победить 

побеждаю – побеждает – побеждают – побеждай

сыпать 

сыплю – сыплет – сыплют – сыпь

хотеть 

хочу – хочет – хотят – хоти

Упр. 32. Раскройте скобки, заменив предложенные в них глаголы подходящими по смыслу описательными оборотами.

  1. Я (убедить) смогу убедить вас в правильности моих взглядов. 2. Если я не сдам экзамена, то (очутиться) окажусь в неприятном положении. 3. В данном положении я, скорее всего, (ощутить) почувствую неудобство. 4. Я уверен, что в этих соревнованиях я (победить) одержу победу.

Упр. 33. Определите, какие фамилии в данных примерах склоняются.

Елена Красных, Дмитрий Дурново, Олег Вулич, Анна Зегерс,

Артур Виденмаер, Евгения Сердюк, Алексей Шелепинь, Мария Пивень, Ольга Олигер, Петр Гура, Евгений Тамм, Эмиль Золя, Владимир Долгих, Клара Гурвич, Иван Франко, Бернард Шоу, Юлия Бурля, Людмила Лата, Иоганн Бобровски, Марк Твен, Александра Вейденкеллер, Ярослава Кулик, Леонид Сковорода, Джозеф Яблонски.
СИНТАКСИЧЕСКИЕ НОРМЫ

Синтаксические нормы – это правила построения словосочетаний и предложений. Ср.: Управляющий банком ( не банка), выйти из положения (не с положения), Вспоминая детство, представляю наш старый дом (не представляется).

Синтаксические нормы регулируют порядок слов в предложении, согласование подлежащего и сказуемого, определения с определяемым словом, некоторые случаи управления, построение простых предложений с однородными членами, причастными и деепричастными оборотами, связь частей сложного предложения.

Упр. 34. Найдите ошибки в употреблении следующих словосочетаний. Исправьте их.

1.Отчитаться в израсходовании полученного аванса.

Отчитаться в израсходовании полученного аванса. 2. Отзыв на статью. Отзыв о статье. 3. Уделять внимание на каждую мелочь. Уделять внимание каждой мелочи. 4. Превосходство новой техники перед старой. Превосходство новой техники над старой. 5. Примириться перед неумолимой судьбой. Смириться с неумолимой судьбой. 6. Уверенность в торжество справедливости. Уверенность в торжестве справедливости.

Упр. 35. Составьте словосочетания, поставив заключенные в скобках слова в нужном падеже.

Удивляться (результаты) –

Удивляться результатам

преклонение (талант) –

Преклонение перед талантом

уплатить (квартира) –

Уплатить за квартиру

упрекать (грубость) –

Упрекать за грубость

уверенность (победа) –

уверенность в победе

тормозить (развитие) –

тормозить развитие

поехать (Кавказ, Крым, Белоруссия) –

поехать на Кавказ, в Крым или Беларусь

выйти (автобус, троллейбус) –

выйти из автобуса или троллейбуса

платить (проезд) –

оплатить проезд

заведовать (отделение) 

заведовать отделением

управляющий (филиал) –

управляющий филиалом

уделить внимание (дисциплина) –

уделить внимание дисциплине

согласно (приказ, распоряжение) –

согласно приказу или распоряжению

Упр. 36. Исправьте ошибки в употреблении падежных форм и предлогов.

1. Об этом мы познакомим вас позже.

С этим мы ознакомим вас позже.

2. Докладчик подчеркнул о необходимости строительства.

Докладчик подчеркнул необходимость строительства.

3. Высказывается критика о том, что мы делаем.

4. Решимость парламента свелась лишь в предоставлении полномочий.

Решимость парламента свелась лишь к предоставлению полномочий.

5. Магазин, ориентированный специально для школьников.

Магазин, ориентирован специально на школьников.

6. Уверенность каждого за свой завтрашний день.

Уверенность каждого в своем завтрашнем дне.

7. Контролировать за ходом лечения.

Контролировать ход лечения.

8. Вчера брат приехал с Москвы.

Вчера брат приехал из Москвы.

9. В журнале опубликована рецензия о книге.

В журнале опубликована рецензия на книгу.

10. По окончанию переговоров представители делегаций подписали совместное заявление.

По окончании переговоров представители делегаций подписали совместное заявление.

Упр. 37. Исправьте ошибки в предложениях.

1. Он учился в школе хорошо, благодаря своих хороших способностей.

Он учился в школе хорошо, благодаря своим хорошим способностям.

2. Девушка очень тосковала по брате.

Девушка очень тосковала по брату.

3. Первый год по приезду его жизнь складывалась нормально.

Первый год с приезда его жизнь складывалась нормально.

4. Он выехал из города согласно предписания.

Он выехал из города согласно предписанию.

5. На этот вечер смогли приехать престарелые люди со всех районов города.

На этот вечер смогли приехать престарелые люди изо всех районов города.

6.В отделе критики и библиографии “толстых” журналов систематически публикуются обзоры о выходящих новинках художественной литературы.

В отделе критики и библиографии «толстых» журналов систематически публикуются обзоры выходящих новинок художественной литературы.

7. Представитель строительной организации вновь заверил заказчика о своей готовности выполнить работу в срок.

Представитель строительной организации вновь заверил заказчика в своей готовности выполнить работу в срок. 8. Факты, о которых изложил автор письма, при проверке полностью подтвердились. Факты, которые изложил автор письма, при проверке полностью подтвердились.

9.Студенты уделяют внимание на записи во время лекций.

Студенты уделяют внимание записям во время лекций.

10. Преподаватель указал о том, что в работе имеется ряд ошибок.

Преподаватель указал на то, что в работе имеется ряд ошибок.
СПРАВОЧНЫЕ МАТЕРИАЛЫ

ОРФОЭПИЧЕСКИЕ НОРМЫ

Табл. 1

Е [, э] Ё [, о]
атлЕт

афЕра

бытиЕ

гололЕдица

зЕв

метоимЕнный

одноврЕмЕнный отцвЕтший

опЕка

осЕдлый

преЕмник

разновремЕнный

безнадёжный

дарёный затёкший

манёвры

наёмник

никчёмный

новорождённый

одноимённый

остриё

оценённый

принёсший

платёжеспособный свёкла щёлка

Табл. 2

Согласный произносится твердо: Согласный произносится мягко:
антитеза [тэ] бактерия [тэ] бизнес [нэ]

генезис [нэ] декольте [дэ], [тэ] детектив [дэ], [тэ] де-юре [дэ] де-факто [дэ]

диспансер [сэ] идентичный [дэ]

интервью [тэ] коктейль [тэ] коллоквиум [ло]

консенсус [сэ]

коттедж [тэ] критерий [тэ]

майонез [нэ]

отель [тэ] орхидея [дэ]

патетика [тэ] претензия [тэ] сервис [сэ] тезис [тэ] тент [тэ]

тембр [тэ]

тенденция [тэ] [дэ] термос [тэ] тест [тэ] тет-а-тет [тэ]

тире [рэ]

шатен [тэ]

шоссе [сэ] штемпель [тэ]

экстерн [тэ]

эстет [тэ]

академия [д’э] берет [р’э] брюнет [н’э]

девальвация [д’э]

декада [д’э]

демон [д’э] кредо [р’э] кларнет [н’э]

крем [р’э]

кофе [ф’э]

музей [з’э]

Одесса [д’э]

патент [т’э]

пионер [н’э]

пресса [р’э]

пресс-конференция [р’э]

пресс-секретарь [р’э]

тема [т’э]

тенор [т’э]

термин [т’э]

фанера [н’э] шинель [н’э]

эксперт [п’э]

Допускается двоякое произношение
деканат [д’э], доп. [дэ]

темп [тэ] и [т’э]

конгресс [р’э], доп. [рэ]

консенсус [сэ] и [с’э]

менеджер [мэ], [нэ], доп. [м’э], [н’э]

сессия [с’э], доп. [сэ]

Табл. 3

Произносится [чн] Произносится [шн]
молочный

полуночный

прожиточный

пшеничный

горчичник

двоечник

Ильинична

конечно

нарочно

Никитична

прачечная

скворечник

скучно

скучный

шапочное знакомство

яичница

Допускается двоякое произношение
булочная [чн] и [шн]; копеечный [чн] и [шн]; порядочный [чн] и [шн];

сердечный [чн] и [шн] – смысловая дифференциация: серде[чн]ый удар и сер- де[шн]ый друг

Табл. 4

Нормы ударения

алкогОль

анАлог

асимметрИя

афЕра

баловАть

балОванный

блАга

бюрокрАтия

вероисповЕдание

газопровОд

гЕрбовый

двоЮродный

дефИс

диспансЕр

договОр

досУг

жалюзИ

звонИт, звонИшь

инАче

Искра

каталОг

квартАл

кладовАя

кОлледж

крапИва

красИвее

мАркетинг

мастерскИ

наркомАния

новорождЁнный

обеспЕчение

облегчИть

ободрИть

оптОвый

отчАсти

на похоронАх

придАное

принУдить

свЁкла

сиротА, сирОты

столЯр

углубИть

фенОмен

ходАтайство

шассИ

щавЕль

МОРФОЛОГИЧЕСКИЕ НОРМЫ

Табл. 5

Род имен существительных

Мужской род Женский род Средний род
Аэрозоль, зал, кофе (разг.: ср.р.),

погон, рельс, толь,

тюль, шампунь.

Бандероль, босоножка, вермишель, ласта, мозоль, мышь, плацкарта, тапка (разг. тапочка), туфля. Алиби, боа, бра, жюри,

какао, повидло, щупальце.

Равноправные варианты:

бакенбарда и бакенбард, клипса и клипс, ставень и ставня, манжет и манжета.

Табл. 6

Существительные в форме род. п. мн.ч.

пара ботинок, валенок, туфель, погон, сапог, чулок, носков

купить ананасов, апельсинов, мандаринов, помидоров, баклажанов, томатов

несколько гектаров, граммов, килограммов, будней, яслей

жить среди англичан, армян, башкир, болгар, грузин, турок, татар, цыган, туркмен, киргизов, негров, таджиков, узбеков

нет блюдец, полотенец, яблок, вафель

отряд солдат, партизан, минеров, саперов

Табл. 7

Окончания имен существительных в форме множественного числа

Окончание -Ы (-И) имеют слова: Окончание -А (-Я) имеют слова:
бухгалтеры

выборы

договоры

инженеры

инструкторы

кондукторы

конструкторы

лекторы

офицеры

приговоры

редакторы

режиссеры

ректоры

сваты

слесари

торты

фронты

шоферы

векселя

директора

доктора

жемчуга

катера

кителя

корма

кузова

купола

мастера

округа

ордера

отпуска

повара

поезда

профессора

сорта

сторожа

шелка

штемпеля

Равноправные варианты:

прожекторы  прожектора, секторы – сектора, тракторы  трактора, цехи  цеха

Табл. 8

Склонение числительных

Падеж 50 – 80 200 – 400 500 – 900
И.п. Пятьдесят Двести Пятьсот
Р.п. ПятИдесятИ ДвУХсот ПятИсот
Д.п. ПятИдесятИ ДвУМстАМ ПятИстАМ
В.п. Пятьдесят Двести Пятьсот
Т.п. ПятьЮдесятьЮ ДвУМЯстАМИ ПятьЮстАМИ
П.п. О пятИдесятИ О двУХстАХ О пятИстАХ

СИНТАКСИЧЕСКИЕ НОРМЫ

Табл. 9

Беспокоиться о ком-либо Тревожиться за кого-либо
Надеть что на что Одеть кого во что
Обращать внимание на что-либо Уделять внимание чему-либо
Отзыв о чем-либо Рецензия на что-либо
Превосходство над чем-либо Преимущество перед чем-либо
Препятствовать чему-либо Тормозить что-либо
Уверенность в чем-либо Вера во что-либо
Уплатить за что-либо Оплатить что-либо

Табл. 10

Благодаря (чему)

Согласно (чему)

Вопреки (чему)

Вследствие (чего)

В связи с (чем)

В зависимости от (чего)

Запомните: в книжной речи предлог ПО используется в значении «после» и сочетается с существительными в предложном падеже (по приезде из столицы, по окончании спектакля).

ЛИТЕРАТУРА

Словари

1. Иванова Т.Ф. Новый орфоэпический словарь русского языка. Произношение. Ударение. Грамматические формы / Т.Ф. Иванова. – М., 2006.

2. Крысин Л.П. Толковый словарь иноязычных слов / Л.П. Крысин. – М.: Эксмо, 2008.

3. Лопатин В.В. Учебный орфографический словарь русского языка / В.В. Лопатин, О.Е. Иванова, Ю.А. Сафонова. – М.: Эксмо, 2008.

4. Ожегов С.И. Толковый словарь русского языка / С.И. Ожегов, Н.Ю. Шведова. – М., 2012.

5. Правила русской орфографии и пунктуации: полный академический справочник / под ред. В.В. Лопатина. – М.: Эксмо, 2007.

6. Русский орфографический словарь / отв. ред. В.В. Лопатин. – М., 2007.
Учебные издания

1. Введенская Л.А. Русский язык и культура речи: учебное пособие / Л.А. Введенская. – Ростов-на-Дону, 2008 (и др. издания).

2. Нормы русского литературного языка: практ. материал к урокам / под ред. О.В. Загоровской.  М.: Просвещение, 2006.

3. Педагогическая риторика [электронный ресурс]: учебное пособие / под общей ред. проф. О.В. Загоровской. – Воронеж: Воронежский государственный педагогический университет, 2012. — < URL: http://www.vspu.ac/ru/download/lib/P/P1_2012_11.pdf>

4. Риторика: учебник для вузов / под ред. Н.А. Ипполитовой. – М.: Проспект, 2009.

5. Русский язык и культура речи. Нормы русского литературного языка: учебное пособие / под ред. О.В. Загоровской.  Воронеж: Воронежский государственный педагогический университет, 2003 (и др. издания).

6. Русский язык и культура речи. Педагогическая риторика: Пособие для самостоятельной работы студентов / О.В. Загоровская, О.В. Григоренко, О.Т. Косаренко, Г.Ф. Свиридова. – Воронеж, 2012. – 39 с.

7.Русский язык и культура речи: учебное пособие / под ред. В.И. Максимова. – М.: Юрайт, 2012.

8. Стернин И.А. Практическая риторика в объяснениях и упражнениях для тех, кто хочет научиться говорить / И.А. Стернин. – 6-е изд., испр. и доп. – Воронеж: Истоки, 2011.

9. Черняк В.Д. Русский язык и культура речи: учебник для бакалавров / В.Д. Черняк. – М.: Юрайт, 2012.

Русский язык представляет собой сложную
систему фонетических, лексических и
грамматических средств. Требование
правильности речи распространяется на
все уровни языка, и всюду требуется
соблюдение нормы. Языковая норма – это
центральное понятие речевой культуры.

Норма литературного языка
это общепринятое употребление языковых
средств, совокупность правил, определяющих
образцовое использование этих средств.
В соответствии с определением, нормативно
– не то, что широко распространено в
речи, а то, что обязательно, что
соответствует требованиям и рекомендациям
словарей и справочной лингвистической
литературы.

Языковые нормы не выдумываются учеными.
Они отражают закономерные процессы и
явления, происходящие в языке, и
поддерживаются речевой практикой. К
основным источникам языковой нормы
относятся произведения писателей-классиков
и современных писателей, анализ языка
средств массовой информации.

Характерные особенности нормы
литературного языка: общеобязательность,
относительная устойчивость и в то же
время историческая изменчивость.

В соответствии с основными уровнями
языка и сферами использования языковых
средств выделяются следующие типы норм:

  1. Орфоэпические
    (произношение), связанные со звуковой
    стороной литературной речи.

  2. Грамматические,
    связанные с правилами словоупотребления,
    употребления словосочетаний и
    синтаксических конструкций.

  3. Лексические,
    связанные с правилами словоупотребления,
    отбора и использования наиболее
    целесообразных лексических единиц.

Нормы помогают литературному языку
сохранять свою целостность и общепонятность.
Они защищают литературный язык от потока
диалектной речи, социальных и
профессиональных арго, просторечия.

Природа языковой нормы очень сложна.
Если бы нормы не были устойчивыми, если
бы они легко подвергались различного
рода воздействиям, языковая связь между
поколениями была бы нарушена. И потому
произведения А.С.Пушкина мы понимаем
свободно, хотя они созданы почти два
века тому назад. В то же время устойчивость
норм не абсолютна, а относительна. Норма,
как и все в языке, медленно, но непрерывно
изменяется под влиянием живой разговорной
речи, местных говоров, просторечья,
социальных и профессиональных жаргонов,
заимствований и т.д.

То, что было нормой в прошлом столетии
и даже 15-20 лет назад, сегодня может стать
отклонением от нее. Например, в начале
19 века нормативными были окончания
-И/-Ы во множественном числе имен
существительных: век – веки, дом –
домы, вино – вины
; окончание –ей в
родительном падеже множественного
числа: афиша – афишей, басня –
басней, капля – каплей
(современные
формы веки, дома, вина; афиш, басен,
капель).

Замена одной нормы другой происходит
очень медленно. Изменению норм предшествует
появление их вариантов, которые реально
существуют в языке.

Вариантность нормы проявляется очень
широко во всех областях языка. Например,
акцентологические варианты:
родился – родился, мышление – мышление;
фонетические варианты: будничный
– буднишний; варианты грамматических
форм: тракторы – трактора, чая – чаю,
искусствен – искусственен; морфологические
и словообразовательные варианты:
спазм – спазма, нагрев – нагревание,
лаконичный – лаконический; синтаксические
варианты: ждать поезда – ждать поезд,
исполненный отвагой – исполненный
отваги и т.п.

Орфоэпическая норма.
Произносительные нормы устной речи
изучает специальный раздел языкознания
– орфоэпия. Соблюдение единообразия в
произношении имеет важное значение.
Орфоэпические ошибки всегда мешают
воспринимать содержание речи: внимание
слушающего отвлекается различными
неправильностями произношения, и
высказывания во всей полноте и с
достаточным вниманием не воспринимается.
Произношение, соответствующее
орфоэпическим нормам, облегчает и
ускоряет процесс общения. Поэтому
социальная роль правильного произношения
очень важна. Особенно в настоящее время
в нашем обществе, где устная речь стала
средством самого широкого общения на
различных собраниях, конференциях,
съездах, симпозиумах.

Варианты норм отражаются в словарях
современного литературного языка.
Например, в «Орфоэпическом словаре»
как равноправные фиксируются акцентные
варианты таких слов: ржаветь и ржаветь,
тефтели и тефтели, штабы и штабы. Варианты
таких слов как нормировать и мышление
даются с пометой «доп.» (допустимо)
нормировать и мышление.

Помета «допустимо устаревающее»
(доп.устар.) показывает, что оцениваемый
вариант постепенно утрачивается:
сливочный – доп.устар. сливошный.

От колебаний нормы следует отличать ее
нарушения. В словарях ненормативные
варианты снабжены пометами «не
рекомендуется» (не рек.), «неправильно»
(неправ.), «грубо неправильно»: ассиметрия,
не рек. ассиметрия; корысть, неправ.
корысть; афера неправ. афёра; документ,
грубо неправ. документ.

Показатели различных нормативных
словарей дают основание говорить о 3
степенях нормативности:

— норма
1 степени – строгая, жесткая, не допускающая
вариантов;

— норма
2 степени – нейтральная, допускает
равнозначные варианты;

— норма
3 степени – более подвижная, допускает
использование разговорных, а также
устаревших форм.

Основные законы произношения
согласных:

— оглушение
и уподобление. В русской речи происходит
обязательное оглушение звонких согласных
в конце слова. Например: хле[п] (хлеб),
са[т] (сад), гора[т] (город) и т.д.

— в
сочетаниях звонкого и глухого согласного
(так же, как и глухого и звонкого) первый
из них уподобляется второму. Например:
ложка – ло[ш]ка, пробка – про[п]ка.

— перед
согласными л, м, н, р, не имеющими
парных глухих, и перед [в] уподобления
не происходит. Следует обратить внимание
на сочетание -ЧН, т.к. при его произношении
нередко допускаются ошибки. В произношении
слов с этим сочетанием наблюдается
колебание, что связано с изменением
правил старого московского произношения.

По нормам современного русского
литературного языка сочетание -ЧН обычно
так и произносится [ЧН], особенно это
относится к словам книжного происхождения
(алчный, античный, единичный, маскировочный).
В соответствии с нормами старого
московского произношения на месте
сочетания –ЧН- произносилось [ШН]:
грешневый, игрушешный, сказошник,
закусошная.

В современном языке можно говорить о
трех категориях слов в зависимости от
рекомендуемого в них произношении
сочетания –ЧН-:

1. Слова,
в которых –ЧН- произносится так, как
пишется: кулачный, уличный, библиотечный,
отличник, заочник.

2. Слова,
в которых допустимо двоякое произношение
–ЧН- и –ШН-: булочная и булошная, молочный
и молошный, порядочно и порядошно,
спичечный и спичешный, сливочный и
сливошный.

3. Слова,
в которых и сейчас орфоэпическая норма
требует произношения –ШН-, а
произношение –ЧН- недопустимо: конечно,
ску[шн]о, наро[шн]о, яи[шн]ица, скворе[шн]ик.
Произношение –ШН- вместо орфографического
–ЧН- в настоящее время требуется и в
женских отчествах на –ИЧНА-: Ильини[шн]а,
Никити[шн]а, Савви[шн]а, Фомини[шн]а,
Лукини[шн]а.

Заимствованные слова, как правило,
подчиняются орфоэпическим нормам
русского литературного языка и только
в некоторых случаях отличаются
особенностями в произношении. Наиболее
существенное из них – сохранение в
произношении звука [О] в безударных
слогах и твердых согласных перед гласным
[Е].

В безударном положении звук [О] сохраняется,
например, в таких словах, как м[о]дель,
м[о]дерн, [о]азис, б[о]а, [о]тель, ф[о]нема
и в иностранных собственных именах:
Фл[о]бер, В[о]льтер, Ш[о]пен, М[о]пассан.
В ряде слов иноязычного происхождения
твердость согласных перед [Е] сохраняется:
ш[тэ]псель, о[тэ]ль, с[тэ]нд, ко[дэ]кс,
[дэ]мпинг, ка[рэ], к[рэ]до и др.

Особенности русского ударения.
Снижает культуру устной речи не только
неправильное произношение, но и
неправильное ударение в словах.

Особенности и функции ударения изучает
раздел языкознания, который называется
акцентологией.

Ударение в русском языке свободно, что
отличает его от некоторых других языков,
в которых ударение закреплено за каким-то
определенным слогом. Например, в
эстонском, латышском, чешском, финском
языках ударным всегда бывает первый
слог. В польском, грузинском –
предпоследний; в армянском, французском
– последний. В русском языке ударение
может падать на любой слог, поэтому его
называют разноместным. Сопоставим
ударение в словах дерево, добыча,
голова, медикамент.
Ударение в русском
языке бывает подвижным и неподвижным.
Если в различных формах слова ударение
падает на одну и ту же часть, то такое
ударение является неподвижным
(берегу, бережешь, бережет, бережем,
бережете). Ударение, меняющее свое место
в разных формах одного и того же слова,
называется подвижным (прав,
правы, права; могу, можешь, могут).

Затруднение иногда вызывает ударение
в винительном падеже единственного
числа имен существительных женского
рода. С одной стороны, ботва – ботву,
десна – десну, стопа – стопу; с другой
стороны, гора – гору, доска – доску,
стена – стену. Конкуренция между
существительными и предлогами (до, из,
за, на, об, по, под) в перетягивании
ударения имеет давнюю историю. В начале
века ударение перетягивали на себя
предлоги: на воду, за ногу, под руки, из
лесу, без вести. В настоящее время для
литературного языка в целом допустимы
оба варианта ударения: по боку, под
боком, за бок, за ворот, на землю, на зиму.
Однако для повседневной речи более
характерны сейчас безударные предлоги:
на воду, год от года, из года в год.

Затруднения вызывает постановка ударения
во многих глаголах прошедшего времени,
имеющих ударение на основе во всех
формах, кроме формы единственного числа
женского рода, в которой оно переносится
на окончание. Например: брал, брала,
брало, брали; взял, взяла, взяло, взяли;
задал, задала, задало, задали; занял,
заняла, заняло, заняли; прожил, прожила,
прожило, прожили и др. То же явление
наблюдается и у многих кратких
страдательных причастий прошедшего
времени. Например: взят, взята, взято,
взяты; начат, начата, начато, начаты;
продан, продана, продано, проданы и др.

Особенно часто нарушается норма ударения
в таких словах как оптовый, начать,
алкоголь, кухонный, жалюзи, задолго,
агент, каталог, намерение
(неправ.
оптовый, начать, алкоголь, кухонный,
жалюзи, задолго, агент, каталог, намерение).

Развитие языка влияет также на место
ударения в слове. Это приводит к
нормативным колебаниям. Для русского
ударения существует понятие вариантности,
означающее наличие в некоторых словах
вариантов ударения, применяемых в разных
ситуациях общения.

Целый ряд вариантов ударения связан с
профессиональной сферой употребления.
Есть слова, специфические ударения в
которых традиционно принято только в
узкопрофессиональной среде. В любой
другой среде такое ударение воспринимается
как ошибочное. Например: шприцы,
эпилепсия (у медиков
шприцы, эпилепсия);
компас, боцманы
(
у моряков – компас, боцмана;
флейтовый (у музыкантов
– флейтовый);
добыча (у горняков
– добыча)
и др.

Ударение в русском языке играет важную
смыслоразличительную роль. С его помощью
различаются омографы (парные
слова, совпадающие по написанию, но не
по произношению): видение – видение,
хлопок – хлопок, атлас – атлас, мука –
мука, проклятый – проклятый.

Об ударении в словах можно справиться
в «Орфоэпическом словаре русского
языка», в «Словаре ударений для работников
радио и телевидения», а также в
орфографических, толковых словарях
русского языка, в различных
словарях-справочниках, в пособиях по
культуре речи.

Грамматические нормы
это правила использования морфологических
форм разных частей речи и синтаксических
конструкций.

Наиболее часты грамматические ошибки,
связанных с употреблением рода имен
существительных. Категория рода, как
правило, весьма стабильна. Однако нередко
наблюдается неправильное отнесение
имен существительных к грамматическому
роду. Так, нарушая литературную норму,
говорят железнодорожная рельса
(вместо железнодорожный рельс),
ароматная шампунь (вместо ароматный
шампунь
), заказной бандероль (вместо
заказная бандероль), лакированный
туфель
(вместо лакированная туфля).
Правильно говорить туфля с ударением
на первом слоге: нет одной туфли; купил
красивые туфли; рада новым туфлям; много
осенних туфель.

— Грамматический род сложносокращенных
слов, составленных из первых букв – СНГ
(эсэнгэ), РТС (эртээс) – или звуков –
ИТАР – определяется по основному слову.
Поэтому говорят: СНГ возник; РТС
заключила договор; СПИД опасен; ИТАР
сообщило.
Однако иногда аббревиатура
получает род, как обычное слово, по
формальному показателю и относится к
мужскому роду в случае нулевого окончания
(ЖЭК, несмотря на то, что это
жилищно-эксплуатационная контора), к
среднему роду в случае окончания –О-
(РОНО, хотя это районный отдел народного
образования).

— Несклоняемые имена существительные,
обозначающие профессию, должность,
звание, традиционно относятся к мужскому
роду: компетентный атташе, молодой
рефери..

— Род несклоняемых существительных,
обозначающих географические названия,
определяется по общему родовому
наименованию. Например: старый Тбилиси
(город); полноводная Миссисипи (река);
Капри спал (остров).

— Несклоняемые существительные,
обозначающие неодушевленные предметы,
относятся к среднему роду. Например:
новое депо, золотое пенсне, разноцветное
кашне ароматное желе.
Исключение
составляет слово кофе (муж.род).
Газеты сообщают о падении цен на
колумбийский кофе.

— В русском языке не склоняются фамилии
на –аго, -яго, -ово, -ых, -их: Живаго,
Дубяго, Дурново, Седых, Чутских.
Не
склоняются также украинские фамилии
на –ко. Например: Стихи Тараса
Шевченко.

— Не склоняются нерусские фамилии,
оканчивающиеся на согласный звук, если
они относятся к женщинам: романы Анны
Зегерс,
но балет Арама Хачатуряна,
игра гроссмейстера Ботвинника, романы
Жюля Верна, история любви Фредерика
Шопена и Жорж Санд.

Для официального наименования рода
занятий, профессий женщин предпочтительнее
существительные мужского рода (аспирант,
геолог, лаборант
). Наименованиям на
–ша, -иха типа инженерша, врачиха,
библиотекарша
присущ сниженный,
иногда подчеркнуто-пренебрежительный
оттенок.

При выборе падежных форм типа в цехе
– в цеху, в отпуске – в отпуску, на холоде
– на холоду
следует исходить из того,
что формы на –Е- присущи литературному
языку, а формы на –У- разговорному.

Соответствуют литературной норме
следующие формы Им.п. множ.числа
существительных муж.рода:

а) с
ударным окончанием –а, -я: адреса,
вечера, глаза, голоса, директора, доктора,
профессора, мастера, номера, ордена,
острова, паспорта;

б) с
безударным окончанием –ы, -и: инженеры,
шоферы, бухгалтеры, редакторы, пекари,
лекторы, тракторы, договоры, приговоры,
выборы, порты, торты.

В ряде случаев наблюдаются вариантные
колебания: инспекторы – инспектора,
джемперы – джемпера, прожекторы –
прожектора, слесари – слесаря, тракторы
– трактора, крейсеры – крейсера.

Следует иметь в виду, что последние
более свойственны бытовой и профессиональной
речи. Некоторые вариантные формы
различаются значением: образы
(художественно-литературное) – образа
(иконы); пояса (части одежды) –
поясы (географические); пропуска
(документы) – пропуски (недосмотры);
цветы (растения) – цвета (окраска)
и т.п.

1. В родительном падеже множественного
числа в литературном языке приняты
следующие формы существительных мужского
рода (одни без окончания, другие с
окончанием):

а)
отдельные названия парных предметов:
ботинок, валенок, погон, сапог, чулок
(но носков);

б)
отдельные названия лиц по национальной
принадлежности: армян, бурят, башкир,
грузин, осетин, лезгин, турок, цыган,
румын, татар
(но: киргизов, арабов,
монголов, таджиков, узбеков, якутов
);

в)
отдельные названия единиц измерения
при указании на их количество: ампер,
аршин, ватт, вольт, герц, ом, микрон,
рентген
(но: граммов, килограммов,
джоулей
).

В случае колебаний форм с нулевым
окончанием и с –ов- первые свойственны
разговорной речи, а последние – строго
литературному языку: (много) сектор –
секторов, апельсин – апельсинов, помидор
– помидоров, томат – томатов.

2. Слова женского рода имеют следующие
окончания в родительном падеже
множественного числа: барж, басен,
вафель, кочерег, свадеб, усадеб, простынь,
яблонь; долей, килей, пригоршней, свечей.

3. Слова среднего рода: верховьев,
доньев (дно – донья), низовьев, устьев,
побережий, коленей, яблок; блюдец,
одеялец, полотенец.

Рекомендуются следующие краткие формы
имен прилагательных на –ен и –енен
(хотя обе формы являются литературными):
бездействен, безнравствен, бессмыслен,
бесчислен, бесчувствен, величествен,
двусмыслен, естествен, злокачествен,
искуствен, легкомыслен
(формы:
бездейственен, безнравственен и
т.д. более свойственны книжной речи).

Синтаксическая норма – это
правила построения словосочетаний и
предложений.

Важную роль в построении предложения
играет правильное согласование
подлежащего со сказуемым.

1. При подлежащем, выраженном собирательным
существительным, имеющим количественное
значение (ряд, большинство, меньшинство,
часть и т.д.) в сочетании с родительным
падежом множественного числа, сказуемое
обычно ставится во множественном числе,
если речь идет о предметах одушевленных
или если подчеркивается активность
действия, и в единственном числе, если
подлежащее обозначает предметы
неодушевленные. Например: Большинство
членов бригады систематически
перевыполняют нормы. Ряд новых домов
стоял в конце деревни.

Такая же форма согласования наблюдается
в тех случаях, когда подлежащее выражено
так называемым счетным оборотом, т.е.
сочетанием количественного числительного
или другого счетного слова (например,
несколько) с существительными в
родительном падеже множественного
числа. Например: Несколько человек
ходили взад и вперед по коридору. Засеяно
сто двадцать гектаров.

2. Если при подлежащем – счетном обороте
имеются слова все, эти, то сказуемое
ставится во множественном числе.
Например: Все пять мальчиков сидели
вокруг костра. Эти восемь книг куплены
недавно.
Наоборот, при наличии слов
всего, только, лишь сказуемое, как
правило, ставится в единственном числе.
Например: На экскурсию пошло всего 15
человек. В секции занимается только 8
студентов.

3. При сложных названиях, состоящих из
двух слов разного грамматического рода,
сказуемое согласуется с тем из них,
которое выражает более широкое понятие
или конкретное обозначение предмета:
витрина-стенд помещена в вестибюле;
книга-справочник переиздана; плащ-палатка
лежала в свернутом виде; кресло-кровать
стояло в углу
и т.п.

4. При существительных мужского рода,
называющих профессию, должность, звание,
но обозначающих женщину, сказуемое в
книжном стиле чаще ставится в форме
мужского рода, а в разговорном – чаще
в женском роде. Например: Ректор
университета регулярно выступал(а)
перед сотрудниками.
Согласование
определений в женском роде с такими
существительными носит разговорный
характер. Например: наша референт

5. При подлежащем, выраженном сочетанием
нарицательного и собственного имени,
сказуемое согласуется с последним:
Референт Иванова помогла директору
составить отчет. Методист Васильева
выдала справку.

6. Часто в предложениях при двух или
нескольких управляющих словах имеется
общее дополнение. Такие конструкции
вполне правильны, если управляющие
слова требуют одинакового падежа и
предлога. Например: читать и
конспектировать книгу; подбирать и
готовить кадры.
Но неверны предложения,
в которых общее зависимое слово имеется
при словах, требующих разного управления.
Например: Трест организовал и руководил
предприятиями
(организовал что?,
руководил чем?); подбор и наблюдение
над фактами
(подбор чего?, наблюдение
над чем?) и т.п. Обычный способ правки в
подобных случаях – добавление ко второму
управляющему слову местоимения,
заменяющего существительное при первом
слове. Например: подбор фактов и
наблюдение над ними.

7. Следует различать конструкции со
словами, близкими по значению или
однокоренными, но требующими различного
управления. Например: беспокоиться о
ком-нибудь,
но тревожиться за
кого-нибудь; обращать внимание на
что-либо,
но уделять внимание
чему-либо; опираться на что-либо,
но
базироваться на чем-либо; отзыв о чем-либо
(отзыв о книге), но рецензия на
что-либо
(рецензия на книгу);
превосходство над чем-либо
(превосходство
над старым), но преимущество перед
чем-либо
(преимущество перед старым).

Нарушение грамматических норм нередко
связано с употреблением в речи предлогов.
Различие предлогов благодаря + Дат.п.
и из-за + Род.п. заключается в том,
что первый используется для обозначения
причины, положительно влияющей на
что-либо (Благодаря помощи населения
последствия аварии были быстро
ликвидированы)
, а второй – для указания
причины, отрицательно влияющей на
что-либо (Из-за отсутствия помощи
населения последствия аварии не могли
быть ликвидированы быстро)
. Имеются
различия и при употреблении целевых
предлогов. В нейтральной речи употребляются
предлоги для, за: Для реализации
программы необходимы соответствующие
ресурсы. Зайди в библиотеку
за
книгами
. Последний предлог
используется только после глаголов
движения. В книжной речи употребляются
предлоги в целях, с целью. Первый
сочетается только с отглагольными
существительными (в целях реализации,
уточнения)
, а второй – с отглагольными
существительными (с целью реализации,
уточнения)
и с инфинитивом (с целью
реализовать, уточнить).

Лексическая норма. Под лексическими
нормами понимают правильность
словоупотребления. Слово должно
использоваться в том значении (в прямом
или переносном), которое оно имеет и
которое зафиксировано в словарях
русского языка. Нарушение лексических
норм приводит к искажению смысла
высказывания.

Нарушение лексических норм приводит к
искажению смысла высказывания. Нарушение
лексических норм порой связано с тем,
что говорящие путают слова, близкие по
звучанию, но различные по значению.
Например, не всегда правильно употребляются
глаголы предоставить и
представить. Иногда мы слышим
неверные выражения типа: «Слово
представляется Петрову», «Разрешите
предоставить вам доктора Петрова».

Глагол предоставить означает
«дать возможность воспользоваться
чем-либо» (предоставить слово, отпуск,
квартиру, права и т.д.), а глагол представить
имеет значение «передать, дать, предъявить
что-либо кому-либо» (представить отчет,
справку, факты, доказательства; представить
к награде, к ордену, к званию и т.д.).

Приведенные выше предложения с этими
глаголами правильно звучат так: «Слово
предоставляется Петрову», «Разрешите
представить вам доктора Петрова».

Порой неверно употребляют такие паронимы:
исполнительный – исполнительский
(исполнительный – старательный,
имеющий своей целью осуществление
чего-либо: исполнительный орган;
исполнительский – относящийся к
исполнителю: исполнительская манера);
логический – логичный (логический
– относящийся к логике: логическое
мышление;
логичный – правильный,
разумный, последовательный: логичное
рассуждение
); отчетный – отчетливый
(отчетный – относящийся к отчету:
отчетный период; отчетливый – хорошо
различимый: отчетливый звук) и т.п.

Лексика языка обогащается постоянно.
Под влиянием тех процессов, которые
произошли в нашем обществе в перестроечный
и постперестроечный периоды наш язык
стал интенсивно пополняться новыми
словами.

Среди общественно значимых политических
терминов, рожденных перестройкой, можно
назвать такие: новое политическое
мышление, общеевропейский дом, всеобщая
система международной безопасности,
либерализация, поименное голосование,
общечеловеческие ценности, арендаторство,
демонополизация, договорная цена, единое
экономическое пространство, зона
свободного предпринимательства, дельцы
теневой экономики, «деревянные» рубли
и т.д.

Особую трудность составляет для говорящих
незнание или неточное знание значений
заимствованных слов. Например: аквапарк,
андерграунд, анимационный, армрестлер,
армрестлинг, аудит, аудитор, аэрогриль,
толинг, толингер, дилер, дистрибьютер,
клиринг, ноу-хау, паблисити, маркетинг,
лизинг.

Говоря о лексических нормах русского
языка наших дней нельзя не указать на
увлечение словами и словечками,
находящимися далеко за пределами
культурного общения, независимо от
степени их новизны в языке. К сожалению,
средства массовой информации захлестнул
поток просторечных, жаргонных, а нередко
и откровенно непристойных слов и
выражений. Для уточнения лексических
норм современного литературного языка
рекомендуется использовать толковые
словари русского языка, специальную
справочную литературу.

Таким образом, говоря о новых словах в
русском языке и трудностях их употребления,
следует еще раз подчеркнуть необходимость
более точного знания семантики
новообразований, особенно терминов,
умения пользоваться заимствованиями,
что требует особой осторожности. Следует
быть также внимательными при употреблении
наименований, имеющих обиходное и
правовое значение и не смешивать их.

Нужно также иметь в виду, что многие
слова изменили свои оценочные свойства
и стилистическую окраску, сферу
употребления, приобрели необычайную
частотность, даже стали «модными», или
утратили былую употребляемость. Наконец,
постарайтесь в своей речи избавиться
от просторечных слов и жаргонизмов.

Контрольные вопросы:

  1. Дайте
    определение понятия «норма литературного
    языка». Перечислите характерные
    особенности нормы.

  2. Расскажите
    о вариантах норм литературного языка.

  3. Охарактеризуйте
    орфоэпические нормы. Расскажите об
    особенностях русского ударения.

  4. Охарактеризуйте
    грамматические нормы. Употребление
    каких морфологических форм вызывает
    у вас затруднения?

  5. В
    чем состоит наибольшая трудность в
    употреблении новых слов?

  6. Как
    вы относитесь к словам, которые находятся
    за пределами русского литературного
    языка (инвективная и нецензурная
    лексика)? Употребляете ли вы их и в каких
    случаях?

Практические
задания

Задание
1.
Какой звук
([о] или [э]) произносится под ударением
в следующих словах? Там, где необходимо,
поставьте две точки над Е. Пользуйтесь
справочными материалами.

Опека, безнадежный,
блеклый, наемник, атлет, маневр, никчемный,
одновременный, разновременный,
одноименный, преемник, бытие, затекший,
острие, обнесший, принесший, местоименный,
новорожденный, оседлый, платежеспособный,
афера, дареный, свекла, наперченный,
щелка, зев, отцветший.

Задание
2.
Найдите
ошибки, которые появились в связи с
неправильным образованием слов. Исправьте
предложения.

1. Змеи относятся
к животным класса пресмыкающих.

2. При гололеде
будьте внимательны на дорогах, чтобы
не подскользнуться и не упасть прямо
на проезжей части.

3. Эта история
может стать самым смотрибельным сериалом.

4. У него был
вспыхчивый характер.

  1. Его предсмертельные
    слова произвели большое впечатление.

Задание
3.
Исправьте
ошибки, вызванные неправильным
образованием падежной формы количественных
числительных.

1. Библиотека
института ежемесячно пополняется триста
книгами. 2. Во время полета была
выполнена широкая программа
медико-биологических исследований,
включавшая в себя около пятьдесят
экспериментов. 3. С семи русских кораблей
успели выпустить по неприятелю более
восемьсот снарядов. 4. Протяженность
столичной подземной дороги достигла
сто семьдесят три километра, а автобусных
и троллейбусных линий четыре тысячи
триста километров. 5. Вместе с новыми
тысяча двухсот тремя словами учебник
немецкого языка будет насчитывать свыше
четыре с половиной тысячи слов.

Задание
4.
Выберите
из скобок один из вариантов; подчеркните
его.

  1. Водить автобусы
    по горным дорогам могут только опытные
    (шоферы – шофера).

  2. (Выборы – выбора)
    губернатора области показывают, что
    многие политические противники заключают
    (договоры – договора) о сотрудничестве.

  3. В летнее время
    у многих людей начинаются (отпуски –
    отпуска).

  4. В нашем кондитерском
    отделе всегда свежие (торты – торта).

  5. Многие (цехи –
    цеха) завода переориентированы на
    выпуск новой продукции.

Задание
5.
Образуйте
от приведенных ниже глаголов формы
1-го лица единственного числа настоящего
времени; 3 лица единственного числа
настоящего времени.

Образец:
люблю – любит – любят.

Бегать, бежать,
бриться, ехать, ездить, класть, махать,
победить

сыпать, хотеть

Задание
6.
Назовите
и запишите слова, употребляющиеся в
русском языке со следующими
интернациональными словообразовательными
элементами.

Авиа…(лат.avis
— птица), авт(о)… (греч.
autos
— сам), агр(о)…(греч.
agros
— поле), аква…(лат.
aqua
— вода), анти…(греч.
anti
— против), био…(греч.
bios
— жизнь), дем(о)… (греч.
demos
— народ), интер…(греч.между), …крат
(греч.
kratos
— власть), микр(о)…(греч.
mikros
-малый), мон(о)…(греч.
monos
— один), орф(о)…(греч.
orthos
— прямой, правильный), поли…(греч.
poli
— много), …скоп (греч.
skopeo
— смотрю), …тек(а) (греч.
theke
— вместилище), тип…(греч.
typos
— отпечаток, образ), ультра…(лат.
ultra
— сверх), фил(о)…(греч.
philos
— люблю), экстра…(лат.
extra
-вне).

Задание
7.
Знакомы
ли вам значения этих иностранных слов?
Употребите их в предложениях.

Абракадабра,
авантюризм, ажиотаж, бедлам, блеф,
вакансия, варьировать, виртуальный,
вульгарный, дебютант, деликатес,
анонимный, конфиденциальный, экстренный,
экстремальный.

Задание
8.
Установите
различия в экспрессивно-стилистической
окраске синонимов. Составьте предложения
с каждым из данных слов.

Сообщать,
возвещать, доводить до сведения,
выкладывать; вкушать, есть, уплетать,
принимать пищу; глаза, зенки, очи, глазки;
уста, губы, губки; книга, книжечка,
фолиант, книжонка; послание, письмо,
письмецо; зачинатель, инициатор, зачинщик.

Задание
9.
Установите
разницу в эмоционально-экспрессивной
окраске слов.

Старец – старик;
ложе – постель; поведать – сообщить;
возложить – положить; стяг – знамя; дар
– подарок; кончина – смерть; дерзание
– стремление.

Задание
10.
Найдите
ошибки в употреблении следующих
словосочетаний. Исправьте их.

  1. Отчитаться в
    израсходовании полученного аванса.

  2. Отзыв на статью.

  3. Уделять внимание
    на каждую мелочь.

  4. Превосходство
    новой техники перед старой.

  5. Примириться
    перед неумолимой судьбой.

  6. Уверенность в
    торжество справедливости.

Задание
11.
Исправьте
ошибки в употреблении падежных форм и
предлогов.

1. Об этом мы познакомим вас позже.

2. Докладчик
подчеркнул о необходимости строительства.

3. Высказывается
критика о том, что мы делаем.

4. Решимость
парламента свелась лишь в предоставлении
полномочий.

5. Магазин,
ориентированный специально для
школьников.

6. Уверенность
каждого за свой завтрашний день.

7. Контролировать
за ходом лечения.

8. Озабоченность
за безопасность границ.

Задание
12.
Исправьте
ошибки в предложениях.

1. Он учился в школе хорошо, благодаря
своих хороших способностей.

2. Девушка очень
тосковала по брате.

3. Первый год по
приезду его жизнь складывалась нормально.

4. Он выехал из
города согласно предписания.

5. На этот вечер
смогли приехать престарелые люди со
всех районов города.

6. В отделе критики
и библиографии «толстых» журналов
систематические публикуются обзоры о
выходящих новинках художественной
литературы.

7. Представитель
строительной организации вновь заверил
заказчика о своей готовности выполнить
работу в срок.

  1. Факты, о которых
    изложил автор письма, при проверке
    полностью подтвердились.

9. Студенты уделяют
внимание на записи во время лекций.

10. Преподаватель
указал о том, что в работе имеется ряд
ошибок.

Понравилась статья? Поделить с друзьями:
  • Принимать к исполнению ошибка
  • Примириться перед неумолимой судьбой исправить ошибку
  • Приметный жест лексическая ошибка
  • Примеры умения признавать свои ошибки
  • Примеры проявления ошибки фундаментальной атрибуции